Stephen Wilde: The Myth of ‘Backradiation’

Posted: February 25, 2012 by tallbloke in atmosphere, climate, Energy, general circulation, Ocean dynamics

The Myth of ‘Backradiation’.

Stephen Wilde

1) The so called ‘consensus’ theory of the Greenhouse Effect.

At some point over the past twenty years or so it has come to be believed that the physics of radiation is the primary driving process for planetary atmospheric temperatures.

I do not know how or when that happened but it is contrary to the settled science that I grew up with some 40 years ago.

The idea is that certain gases in the atmosphere known as Greenhouse Gases absorb and emit more radiation than the other, majority, gases such as Oxygen and Nitrogen which make up the vast bulk of Earth’s atmosphere.

Consequently it is proposed that such Greenhouse Gases block radiation emanating from the Earth’s surface from escaping to space and re- radiate a proportion of such upward radiation back to the surface which then becomes warmer than it otherwise would have done.

That, in essence is the Greenhouse Theory of radiative gases and that underlies the entire proposition that human emissions of CO2 and other similar gases such as Methane are capable of heating up the Earth so as to change climates and ultimately cause devastation.

2) The original theory of the Greenhouse Effect.

When I was being educated back in the 1960’s the so called Greenhouse Effect was widely regarded as a consequence of atmospheric pressure. That was the ‘consensus’ view in those days.

It was often stated that the high pressure at the surface of Venus was the cause of the heat at the surface.

The atmosphere of Venus is around 90 times the mass of that of Earth and it was accepted that that was the cause of the surface temperature. Nobody suggested in those days that the higher temperature at the surface of Venus was anything to do with the fact that the atmosphere is comprised of 95% of CO2 as against the 0.04% of CO2 in Earth’s atmosphere.

In fact it was pretty obvious that CO2 had nothing to do with it because the atmosphere of Mars is also around 95% CO2 yet the surface temperature is low so it isn’t anything to do with the simple proportions of the atmospheric concentrations of CO2.

It was widely accepted that atmospheric pressure was the critical factor in accordance with the established science of the Gas Laws details of which can be found here:

http://en.wikipedia.org/wiki/Lapse_rate

Thus:

While most often applied to Earth’s atmosphere the concept can be extended to any gravitationally supported ball of gas”

and:

this lapse or reduction in temperature is normal with increasing distance from the conductive source”.

So there it is, out in the public domain, the fact that surface temperature for a planet with an atmosphere has a temperature at the surface higher than the temperature at the top of the atmosphere and the lapse rate describes the rate at which that surface temperature declines with height.

The temperature of the Earth at the surface is completely explained by the Gas Laws and the Adiabatic Lapse Rate with no need to consider the radiative characteristics of the individual components of the atmosphere.

Furthermore it has been shown that the temperature of Venus is much the same as that of Earth at the same atmospheric pressure subject only to an adjustment for distance from the sun. CO2 clearly has nothing to do with the surface temperature.

3) Which Greenhouse Theory is correct ?

Huge confusion has been caused by that ‘consensus’ theory because we have an entire generation (possibly two) of scientists, students and members of the public who think that molecules of Greenhouse Gases are drifting around in the atmosphere like little mini radiators directing heat back down to the surface and warming it up potentially catastrophically. Indeed I did read a newspaper report describing the phenomenon in just that way.

Fortunately, it is utter nonsense. There are no little radiators in the sky heating us up until we descend into oblivion by destroying our environment.

The truth is that it is a matter of pressure (induced by the gravitational field of the planet and atmospheric mass) plus the level of solar input that determines the temperature at the surface of ANY planet.

Those factors and those factors alone determine the amount of energy that any given planet can hold on to before any excess is lost to space.

The temperature of the surface of any planet is limited by gravity, atmospheric mass and solar input alone as per the Gas Laws and if any change in the composition of the planet or its atmosphere tries to increase that temperature then the atmospheric circulation simply reconfigures itself to eject the excess faster to space by a variety of radiative and non radiative processes.

Further evidence in support of that is the simple fact that ALL atmospheric gases are at the same ambient temperature at any given height in the atmospheric column. Thus (relatively) non radiative gases such as Oxygen and Nitrogen participate fully in the effect, no doubt from conduction and convection which would have warmed those gases up in the earliest days of the formation of our atmosphere.

Once one involves ALL atmospheric gases in the warming effect then a little more CO2 from human emissions becomes utterly trivial and not worth consideration especially when one includes our oceans as part of our atmosphere as we should.

4) An example of confusion.

I recently became involved in a blog discussion involving, amongst other things, the fact that where a table is left out in a garden overnight there will be condensation on the top of the table, condensation on the grass all around but little or no condensation on the grass under the table.

There were all sorts of convoluted radiative solutions considered including radiation from the ground to the underside of the table and from the underside of the table to the ground then from the top of the table upwards and from the sky downwards. None of it was satisfactory because the radiation down from the underside of the table would be insufficient to prevent condensation below the table and if radiation were coming down from the sky then the table top being insulated to some degree from the ground should not have been so wet from condensation when it should have been warmed by the sky above. It just couldn’t be made to fit the observations if there is significant downward radiation from the sky as proposed by AGW theory.

However, if one applies the Gas Laws, ignoring radiation downward from the atmosphere the answer becomes clear. The presence of the table top blocks radiation from the ground which remains warmer than it would have been so less condensation occurs on the ground beneath the table. The top of the table radiates freely to space and becomes cold enough to reduce the air above it to below the dew point and so receives condensation.

5) A better concept.

The Gas Laws explain perfectly well why the surface is the warmest place (excluding geothermal activity) for a planet with an atmosphere. Due to gravity there are more molecules of the atmosphere per unit of volume at or just above the surface so when solar irradiation comes along there are more molecules per unit volume for it to react with either directly via radiation or indirectly via conduction from the surface and greater heat is generated at the surface declining with height. That is very simple and very obvious.

It is a mistake to then regard the atmosphere as radiating down to the surface because the atmosphere always radiates upward. Instead we should regard the atmosphere as being heated by energy scattering from the surface upward such that it is the temperature of the atmosphere just at or above the surface that passes energy back to the ground and not the sky as proposed by AGW theory.

Everything makes much more sense on that basis.

6) Conclusion.

An atmosphere of ANY composition will heat the surface below in accordance with the Gas Laws which dictate that the only relevant factors determining surface temperature are surface pressure (from gravity and atmospheric mass) and solar energy input. Those factors, and those factors alone, control surface temperature and indeed energy content for the system as a whole.

A change in composition for any part of the system will only alter the rate at which energy flows through the system so that for example IF more GHGs do indeed slow down the throughput of energy then the system will provide another way of speeding it up again for a zero net effect. All that happens is that the balance changes between one type of energy transfer and another. The system has radiation, evaporation, conduction, convection and the water cycle to freely manipulate as necessary.

As it is, GHGs radiate upwards as much as they radiate downwards and I have seen no evidence that the two directions of emission do not cancel out. Since the vast majority of the atmosphere consists of non GHGs then GHGs actually provide a mechanism for cooling that non GHGs cannot provide so unless demonstrated otherwise I assume that the net effect of more GHGs is zero.

Be that as it may, if it were to be shown that the net effect of more GHGs is to make the atmosphere somewhat warmer then the only effect would be to accelerate other processes such as evaporation, conduction, convection, outward radiation and a faster or larger water cycle. Otherwise, the Gas Laws would be breached.

To achieve that faster throughput of energy the surface air pressure distribution and the position of the climate zones would need to shift a fraction but I have discussed elsewhere how vastly larger such shifts already occur naturally from variations in the behaviour of oceans and sun.

On the basis of the above, the radiative theories behind Anthropogenic Global Warming would appear to be incorrect as regards the addition of any extra energy to the whole Earth system but MAY be correct (if the amount of energy sent downward exceeds the value of energy sent upward) in terms of a miniscule climate effect from a change in the surface pressure distribution and a consequent shift in the climate zones.

Separately, it appears that the proposed positive effect from more water vapour in the air as a result of more GHGs is likely to be incorrect since there has been no sign of increasing global humidity during the late 20th century period of a slightly warming troposphere.

If anything, the Earth system response to more GHGs would appear likely to be negative as I have proposed.

A positive water vapour response to more GHGs was always implausible because more water vapour would lead to more warming and more water vapour then more warming ad infinitum. In the past there has been much more CO2 and much more warmth without that outcome.

There are also problems with the surface temperature measuring system as a result of compromised sensor locations and urban heat island effects plus numerous ‘adjustments’ to past data which seem not to have been fully recorded or justified.

How it must work in practice.

We need to get away from the idea that no work is being done where a gas which is receiving solar energy is situated within a gravitational field. Roy Spencer has made a big mistake in saying that there is a once and for all compression with no work done thereafter. Lots of other highly qualified scientists say just that.

They seem to forget that molecules of gas that are receiving solar energy vibrate, moving in ALL directions, by virtue of their kinetic energy which is why they are in gaseous form and not a solid.

Adding solar energy is what makes them zip about to form a gas in the first place and the more solar input the more they zip about.

The power source is the solar input. Switch it off and gravity succeeds in pulling down all the molecules so that the gas forms a solid on the surface once more.

The sun adds the necessary power, work is done as the molecules vibrate and infrared heat energy becomes highest where gravity has placed most molecules available to participate in the process, at the surface. The weight of the whole atmosphere places greatest pressure and greatest molecular density at the surface.

So, at that point we have solar input, the surface pressure and the surface temperature all in balance. Change any one of them and the point of equilibrium changes as per the Gas Laws.

However, on a planet with an open sky there is also the volume of the atmosphere to consider. That volume will be related directly to the total system energy content.

As soon as one changes system energy content by however small an amount the volume of the atmosphere will change instantly to keep the other numbers the same UNLESS one also alters the pressure at the surface AND/OR the level of solar input. Those two factors will change the equilibrium temperature because they increase the total amount of interaction going on between air molecules and insolation at the surface.

GHGs alter system energy content a fraction so the volume of the atmosphere changes by a fraction but because there is no increase in surface pressure and no increase in solar input then the change in atmospheric volume instantly cancels the effect of the GHGs on surface temperature by shifting the entire atmosphere upward leaving less molecules near the surface to participate in the interaction with solar energy.

The difference is that GHGs only affect the timing and location of the throughput of energy and NOT the total amount of interaction between air molecules and solar energy.

GHGs slow down energy loss to space but the increase in atmospheric volume simply dissipates it instantly and in the process the global circulation of the air shifts a miniscule fraction as a result of the increased volume of the atmosphere.

The same thing happens in response to ANY change other than increased surface pressure or increased solar input.

Hence the Venus/Earth observations and the curve fit shown by N & Z.

Summary.

Neither the Earth nor the Earth’s atmosphere can be treated as black bodies:

http://en.wikipedia.org/wiki/Black_body

To give black body status to Earth you have to take a point beyond the atmosphere as the ‘surface’ and only then apply the relevant equations for ascertaining surface temperatures:

http://en.wikipedia.org/wiki/Stefan%E2%80%93Boltzmann_law

Furthermore, treating Earth and its atmosphere as two black bodies separated by a vacuum is wholly inappropriate because the Earth and its atmosphere are a single unit interacting primarily via non radiative processes which is where the Gas Laws come in.

Due to the governing factors of surface pressure and solar input the only relevant transfer of energy to and from atmosphere and surface involves the non radiative process of conduction plus a tiny amount of radiation from the GHGs just at or above the surface.

Conduction from the solar irradiated surface supplies the energy for convection and evaporation which together facilitate the transfer of energy from surface to space, in the process being well able to negate any radiative effects from changes in atmospheric composition.

Additional support is supplied by horizontal air mass movements such as the development and equatorward migration of features such as the ‘Mobile Polar Highs’ described by Marcel Leroux. Atmospheric events such as those have a substantial impact on sea surface / surface air temperatures.

The radiative characteristics of GHGs are neutralized within the system by:

i) The ability of GHGs to radiate directly out to space and

ii) The ability of the atmosphere to reconfigure itself by altering the surface pressure distribution thereby adjusting the rate of energy flow from surface to space.

If the radiative characteristics of GHGs were not neutralised then the Gas Laws would not apply and the concept of the Standard Atmosphere would not work. I suggest that readers Google the Standard Atmosphere and acquaint themselves with its characteristics. Aviation, in particular, relies on it being valid.

So, for bodies separated by a vacuum, apply the Stefan Bolzmann equations but only at a point outside any atmospheres where radiative processes do indeed dominate exclusively.

For bodies not separated by a vacuum, such as a planet and its atmosphere, apply the Gas Laws because non radiative processes dominate by holding the balance of the net energy flow within their control.

Comments
  1. Richard111 says:

    Very nice. Thank you. I will print this out for my coffee table talking points. One quibble, you mention the sun as the power source, but no mention of how “greenhouse gases” will shield the surface. That incoming sunlight has the full radiation range whereas the outgoing nightside radiation is strictly limited to the far infrared. There is a huge difference in performance there. Is this ever taken into account?

  2. Crom says:

    Am I missing something, or does all of this text basically boil down to: “I was taught something 50 years ago, so it must be true…”

    I would suggest that Mr. Wilde’s intuition about how the atmosphere works is probably not a good substitute for actual science.

    [Reply] Actual science begins with descriptive formulation and qualitative thinking. This work is the basis of theory construction, not the end product. This site is about collaboration to develop and refine ideas. Measured and polite criticism is part of the process. Point scoring and inflammatory rhetoric is not. Get less snarky, or get lost.

  3. Ray Tomes says:

    Article states: “Due to gravity there are more molecules of the atmosphere per unit of volume at or just above the surface so when solar irradiation comes along there are more molecules per unit volume for it to react with either directly via radiation or indirectly via conduction from the surface and greater heat is generated at the surface declining with height. That is very simple and very obvious.” which is quite correct except that it equates heat to temperature. More molecules absorb more heat, but NOT more heat per molecule. Temperature is heat per molecule. The argument is unsound.

  4. tallbloke says:

    Hi Ray, quite right, but consider the dynamic situation. The point is that there is constant replenishment as long as the sun is shining and constant loss as bodies radiate and conduct. So a given volume of denser air is able to conduct or radiate (if humid) more heat to an adjacent cooler mass per second than thin air before it suffers a significant drop in temperature itself. The convective processes which will ameliorate that effect take time, so denser air will raise the temperature of a cold body faster than less dense air, even though the molecules might be at the same temperature. Being a mountaineer, I’m well aware of this through experience, though there may be something else going on I haven’t thought of.

    Do you agree with my line of thought?

  5. Roger Clague says:

    Steven Wilde says

    “”vacuum, apply Stefan-Boltzmann Law” and “planet and atmosphere, apply Gas Laws”.

    The consensus in climate science is to apply radiative laws and Gas Laws together to the Earth and its atmosphere. Why is this wrong?

    Steven says

    1. Separating heat and radiation correctly calculates laspse rate and surface temperatures.
    2. At TOA, radiation in = radiation out, therefore in the atmosphere as a whole the effects neutralise each other and cancel out.

    I point out that radiation energy depends on frequency of EMR, heat on mass. They are so different they can and must be analysed separarely.

  6. Markus Fitzhenry says:

    “However, if one applies the Gas Laws, ignoring radiation downward from the atmosphere the answer becomes clear. The presence of the table top blocks radiation from the ground which remains warmer than it would have been so less condensation occurs on the ground beneath the table. The top of the table radiates freely to space and becomes cold enough to reduce the air above it to below the dew point and so receives condensation.”

    Thank you Stephen, when Hans, who I have unwavering respect for, said ‘I can assure you that a cloud cover in midwinter in Stockholm area (59 latitude north) means it is about 5-10C warmer than if the sky is clear. At the same time I am absolutely sure there exists nothing such as +300 W/m^2 back radiation in the real world.’ I was worried I was going down the wrong track about ‘back radiation’, and I came to a halt with my kids book.

    Your table explanation has shone some light for me. Thanks.

  7. Joe's World says:

    Stephen,

    Volume of gases make no difference nor does type of gases?
    Of course not in a controlled laboratory with limited parameters and no motion.

    We can’t even accurately measure atmospheric pressure.
    When your ears “pop” traveling…does that not tell you there have been pressure changes?

  8. Stephen Wilde says:

    Ray Tomes said:

    “More molecules absorb more heat, but NOT more heat per molecule. Temperature is heat per molecule. The argument is unsound.”

    Molecules absorb more photons from the sun either directly or via conduction from the surface. The additional energy in the molecules causes increased vibration of those molecules. That vibration is known as kinetic energy. That kinetic energy generates IR longwave which is recorded as heat in a sensor designed to display the level of heat as what we call temperature.

  9. Stephen Wilde says:

    Roger Clague said:

    “The consensus in climate science is to apply radiative laws and Gas Laws together to the Earth and its atmosphere. Why is this wrong?”

    I’m not convinced that that is correct. Can you demonstrate it ?

    “1. Separating heat and radiation correctly calculates laspse rate and surface temperatures.
    2. At TOA, radiation in = radiation out, therefore in the atmosphere as a whole the effects neutralise each other and cancel out”

    I do not see that as an accurate summary of my words.,

  10. Mydogsgotnonose says:

    It’s quite easy to prove ‘back radiation’ is imaginary. All you need to know is that radiation is emitted and absorbed by sites matched exactly to the incoming photon, then you must understand the concept of a photon was invented by Planck to teach radiation physics to people who find the thing bloody hard; there’s no such thing!

    Imagine two bodies in radiative equilibrium in a vacuum. Radiation flux each way is identical. When it reaches the body, each photon is absorbed and merges into a group with a certain probability of being ejected or converted to internal kinetic energy [heat]. Because the bodies are at constant temperature, those probabilities are equal. Therefore, no net heat flux enters or leaves either body.

    However, there remains equal energy transmission in either direction but it cancels out. This is called Prévost Exchange Energy [Law of 1791]. Trenberth would have you believe it is added to the energy leaving each body, doubling the S-B constant.

    What happens if you reduce the temperature of body 2? Less energy is emitted by it yet the energy coming to it from Body 1 is unchanged. Using ‘1.2’ to denote ‘from 1 to 2’, the net energy transfer is [Q1.2 –Q2.1] and the Prévost Exchange Energy has fallen to Q2.1. In the limit when Body 2 is at absolute zero, there is no Prévost Exchange Energy.

    What oxymoronic climate scientists do is to shield a radiometer detector pointing at body 2 from Q1.2 so it measures Q2.1 and they call this ‘back radiation’. They are quite prepared to accept that net energy transfer is Q1.2-Q2.1 but they double count energy flow and by setting Q1.2 to the S-B equation for T1, claim that total radiative flux is the same as in a vacuum. They then add to this convection and evapo-transpiration. The end result is that they artificially create positive feedback in the iteration process.

    In reality, Prévost Exchange Energy is a standing wave which connects at the speed of light the radiative density of states on the two bodies, in effect the measure of temperature, absorptivity and emissivity needed to give Kirchhoff’s Law of Radiation, also a natural control system. The act of blocking one part of it reveals the hidden remaining half as an apparent energy source. How convenient for people trying to scare public and politicians about imaginary CAGW!

  11. I have posted a new article on my blog,

    “The True Energy Balance of the Earth+Atmosphere”

    demonstrating a decisive quantitative verification of my views on fundamental atmospheric warming, including verifying the true radiating temperature of the Earth+Atmosphere system (NOT the 255K of the consensus):

    ——————–
    And Ray Tomes said:

    “More molecules absorb more heat, but NOT more heat per molecule. Temperature is heat per molecule. The argument is unsound.”

    ————- Molecules under greater pressure (with greater depth, in the hydrostatic troposphere) absorb more heat per molecule, thus attain a higher temperature with depth.

  12. mkelly says:

    All heat transfer depends on a difference in temperature only. The atmosphere gets its heat from the surface of the earth and is under normal conditions “cooler” than the surface ergo it cannot transfer heat to the surface. I have asked Joel, Willis E., W.Connolly among others to write a radiative heat transfer equation showing how back radiation can heat the surface and have not gotten one response. Further, they don’t grasp the significance of if T1=T2 no heat is transfer occurs but radiation can still be taking place.

  13. Mydogsgotnonose says:

    The scam is based on ‘peer-reviewed’ but incorrect science’. Thus Arrhenius got IR warming [a mistake by Tyndall] wrong and mistakenly assumed the S-B equation predicts that a solid surface in contact with the atmosphere emits radiation according to S-B for its temperature in parallel with convection. In reality, the sum of the two has to equal the incoming SW energy.

    That ‘climate science’ pushes provably incorrect science is what defines it as a cult with dogma.

  14. Magic Turtle says:

    I also went to school in the ’60s but the question of what makes planetary temperatures what they are didn’t occur to me until the ’70s, by which time Carl Sagan was already promoting the idea of CO2 causing a runaway greenhouse effect on Venus and we’d all better watch out or we’ll make CO2 do the same thing on Earth. You can watch a video of him and Steven Hawking promoting that emotive idea here: http://www.youtube.com/watch?v=mu1PicT0TMU .

    That idea is quite false of course. Even if there is a greenhouse warming effect from CO2 it could not possibly produce a runaway greenhouse effect on Earth so long as the surface is not completely covered in clouds (like the Venusian surface is in fact). The reason for this is that clouds reflect sunlight so if CO2 causes warming at the planet’s surface, evaporation rates increase, the oceans produce more clouds, which reflect away more sunlight and thereby cause the Earth to cool. Neat, eh? It’s called the ‘Iris effect’ because it acts like the iris of the eye as a negative feedback mechanism on the incoming sunlight to keep the Earth’s surface at a constant energy-level. And the Earth’s energy-level is its global mean temperature.

    Sagan and all the other scientists who promoted the myth of the man-made terrestrial runaway greenhouse effect from CO2 were sufficiently intelligent and informed to have known about the Iris effect and that the idea which they were promoting was false and impossible. Therefore one can say that they must either have been deluded or else have been lying deliberately. Either way, their effect has been to generate massive confusion and fear in the minds of the general public.

    I think the fear only makes it more difficult to clear up the confusion and misunderstanding that they have created about the greenhouse effect. The fear gives us a subconscious reason to deny the existence of the greenhouse effect and to try to formulate logical rationalisations for our denial of it. I am not talking about ‘denial’ here in the same sense as man-made climate change zealots use that term as an insulting label for anyone who does not subscribe to their alarmist doctrine. As Prof. Lindzen recently made clear in his talk at the British Parliament, one does not have to deny the existence of the greenhouse effect in order to be able to deny, on rational, scientific grounds, the exaggerated claims that the alarmists are making for it.

    In my view rational denial of the principle of the greenhouse effect is not possible because it follows logically from the known laws of physics. Likewise, I believe the principle of back-radiation cannot be denied rationally either for the same reason. Let me try to prove my beliefs with an imaginary example.

    The Earth’s climate system is exceedingly complex so let us reduce the problem to its simple essence in our minds by imagining a planet that is the proverbial ‘ball of rock’ in space and orbiting a star. To start off, let us give this hypothetical planet an atmosphere that is totally transparent to all wavelengths of electromagnetic radiation or ‘light’. (Since the surface is a dry rock, there can be no clouds in its atmosphere to complicate the picture.) Because of its perfect transparency, the atmosphere cannot be warmed by direct absorption of any radiation coming in from space or from beneath the planet’s surface and therefore the only possible ways in which it can receive energy to warm it are by conduction and convection from the surface. But the atmosphere cannot be warmed above the temperature of the surface by these means because that would require heat to flow spontaneously from the cooler body (atmosphere) to the warmer one (the solid planet) which would violate the 2nd law of thermodynamics.

    We may note that the planet’s gravitational field is causing a temperature-lapse to arise in the atmosphere because it is causing the individual air-molecules to accelerate constantly in the downward direction towards the planets surface. But gravity cannot increase the atmospheric temperature at the surface above that of the surface itself because the temperature is the kinetic energy of the molecules and that can only be as great as the amount that is given to it by the surface in the first place. The situation is like a juggler who is keeping many balls in the air at the same time. No matter how high the juggler may throw the balls, they will land back in his hands with only the same kinetic energy that he gave them when he threw them up. So there is no gravitational heating of the planet’s surface and the Gas laws of physics are irrelevant here.

    We may also note that, because the atmosphere has been warmed above absolute zero by the surface, it is spontaneously radiating energy in all directions like all other material bodies in the universe in accordance with Planck’s law and the Stefan-Boltzmann (S-B) law. The radiation that is emitted in the direction of the surface, called ‘back-radiation’, actually reaches the surface because the atmosphere is perfectly transparent and at least some of it is absorbed there, thereby increasing the temperature at the surface by a definite amount. This heating effect by back-radiation may be very small but it is real nonetheless and if we knew the average temperature and emissivity of the atmosphere we could calculate it precisely with some hairy mathematics. But because it is back-radiated energy that came from the surface originally, whereby the surface temperature would have been diminished by an even greater amount than the back-radiation (the rest being radiated away ultimately into outer space) it is unable to restore the surface temperature to what it would have been if the surface had not warmed the atmosphere initially.

    So in this simple hypothetical example of a rocky planet with a perfectly transparent atmosphere, inevitably there is back-radiation but no gravitational warming of the surface. But since the back-radiation here does not raise the energy-level of the surface above its initial value as determined by the energy it receives from in-coming sunlight, it does not constitute a ‘greenhouse effect’ as such and it produces no ‘global warming’ whatsoever.

    Now let us imagine that a group of advanced extraterrestrial geoengineers discover the planet and decide that it would be just right for their occupation if only it was somewhat warmer and so they blast holes in the planet’s surface to release some of the CO2 and other ‘greenhouse gases’ that are stored inside the crust. When the dust has settled and the planet has returned to energetic equilibrium the atmosphere is in a new stable state that is fundamentally different to the old one. The presence of the greenhouse gases in the atmosphere means that it is no longer perfectly transparent on all wavelengths now but that it is absorbent of radiation in infrared (IR) waveband. This in turn means that the atmosphere is receiving energy in two new ways in addition to the conductive and convective warming by the surface that it had before. These are, namely, by absorbing IR-radiation directly from the in-coming solar irradiance and from the out-going radiation emanating from the surface. This new absorption of energy warms the atmosphere above its earlier temperature and so places it onto a new, higher energy-level.

    What happens to this new energy that is entering the atmosphere in these new ways? First, because the new atmosphere is now warmer than the old one, it is radiating energy itself at a faster rate in all directions and so the amount of energy carried by the ‘back-radiation’ has increased. Secondly, the portion of the absorbed energy that is not radiated away becomes mechanical energy of the individual molecules, ie. the energy of material force and motion, or potential and kinetic energy. The gravitational field acts to redistribute the new absorbed energy in such a way that it becomes pure potential energy at the top of the atmosphere and pure kinetic energy at the bottom. This extra kinetic energy at the bottom of the atmosphere (ie. at the surface) constitutes global warming. No violation of any physical laws is occurring here.

    So we can see then that both principles of global warming – ie. back-radiation and gravitational, or ‘adiabatic lapse’ heating – are valid physical principles that are not mutually exclusive or incompatible but are natural complements to each other. They are both in operation in all planetary atmospheres that contain greenhouse gases.

    Yet planet Earth is more complex still and it contains yet more original ways of heating and cooling its atmosphere. Probably the biggest complicating factor on Earth is the water cycle, which transforms water through various phases in various ways and patterns that all have different energetic characteristics and properties that affect the distribution and balance of energies in different forms throughout the world. But even that is only one factor among many more, including the Earth’s interactions with its electrical and magnetic environments and with cosmic rays, and then there are biological factors too. We are just at the infantile stage of beginning to take the first primitive steps in understanding this awesomely complex natural system and are nowhere near able to foresee how the global mean temperature may change at any time in the future. Those people who pretend to know how the world’s climate system works and how it will respond to anything that we humans are doing do not appreciate the abysmal depths of their own vast ignorance and foolishness in my view.

  15. greg elliott says:

    In a gravitational field, if a gas is in free fall, then the molecules will travel in straight lines, and the kinetic energy (conduction) will be equivalent in all directions.

    However, on a planet such as earth this is not what is happening. The gas is not in free fall. Rather, it is being accelerated by the force of gravity, at 9.8 m/s^2 “upwards”.

    As a result, molecules that are moving “down” towards the surface will on average have a higher kinetic energy than molecules that are moving “up”. As well the motion of molecules that are moving laterally will not follow straight lines. They will have parabolic trajectories downward, which will tend to favor collisions in the downward direction.

    These two factors taken together, the increased kinetic energy in a downward direction, and the parabolic trajectories favoring downward collisions, suggests that conduction is directional in an accelerating frame of reference.

    That over time, gravity though conduction will increase the kinetic energy of the molecules towards the surface, at the expense of the molecules further away from the surface. As temperature varies directly with kinetic energy, this will give rise to a dry air lapse rate due to conduction.

    The assumption that without GHG there would be no lapse rate – this is based on a 150 year old assumption that was not confirmed experimentally. Rather it was decided politically, based on majority rule.

    It is interesting to note that in 150 years since this question was decided, it has never been experimentally tested, except recently by one amateur scientists. In that time a large body of self -reinforcing science has been developed, based on that one assumption.

    However, a conclusion is only valid if the premise is valid. And the one experiment performed, what does it say? It says that the premise is wrong. That for 150 years science has marched down the wrong road.

  16. Paul says:

    Random thoughts:

    How would you explain historical temperature changes such as MWP and LIA? Must these be due to changes in atmospheric pressure? What would be the mechanism for that?

    If we’re adding carbon atoms to the atmosphere, we’re also making it heavier, so that should also have some effect on the pressure and therefore temperature. But isn’t that change too small to explain the change in temperatures over the last 70 years or so?

  17. Magic Turtle says:

    Mydogsgotnonose said (Today 3:24 PM):

    “It’s quite easy to prove ‘back radiation’ is imaginary. All you need to know is that radiation is emitted and absorbed by sites matched exactly to the incoming photon,…”

    This is not a limiting condition in the case of the Earth’s atmosphere and solid/liquid surface. If substances in the Earth’s atmosphere are able to absorb IR-energy emitted from its surface then the corresponding quantum-resonance site-matches between emitters and absorbers already exist.

    ‘Back-radiation’ is a fact. It is simply a consequence of the fact that all matter in the universe is radiating energy in all directions all of the time. This implies that the Earth’s atmosphere is radiating in all directions too. That portion of the atmosphere’s radiation which is directed towards the planetary surface is called ‘back-radiation’. That’s all there is to it. However, please see my post above for a slightly more extensive explanation.

  18. greg elliott says:

    It is interesting to note that the theory that GHG gives rise to the lapse rate directly contradicts that theory that GHG gives rise to surface warming.

    For GHG to give rise to the lapse rate, GHG must be removing energy from the atmosphere. 1/2 of that energy will radiate outwards to space, and 1/2 of that energy will radiate towards earth in the form of “back radiation”.

    This 1/2 of the energy from the atmosphere that is being removed to space by GHG to create the lapse rate is energy that would otherwise be available to warm the night side of the earth, and warm the earth towards the poles. Thus, GHG in creating the lapse rate has the net effect of reducing the energy available to warm the surface, leading to a net reduction of the surface temperature.

    The the same time, the AGW theory says that GHG leads tosurface warming by blocking outgoing radiation. However, to block outgoing radiation, GHG needs to absorb energy which will cause the atmosphere to increase in temperature (the atmospheric hotspot).

    However, this is directly contradicted by the GHG caused lapse rate, which says that GHG causes the atmosphere to cool!!

    So, we are left with the contradiction, that adding GHG to the atmosphere will cause atmospheric cooling, leading to the lapse rate, and adding GHG to the atmosphere will lead to atmospheric warming, leading to surface warming.

    Since the lapse rate is confirmed by observation, (while the atmospheric hotspot has not been confirmed by observation), if GHG gives rise to the lapse rate, then GHG cannot cause surface warming.

  19. Magic Turtle says:

    Greg Elliott said (Today 7:31 PM):

    “The assumption that without GHG there would be no lapse rate – this is based on a 150 year old assumption that was not confirmed experimentally.”

    I quite agree. The adiabatic lapse is an effect of the gravitational field, not of greenhouse gases.

  20. Steve Fox says:

    Hi Stephen
    Thanks for your clear presentation of this. I have no science background, but am trying to get a grasp of the principles involved.
    I read recently that heat could not be radiated into space since there are no molecules present to carry it. Therefore also that any notion of space being cold (or hot for that matter) was mistaken.
    Is this incorrect? It seems to conflict with the GHGs radiating heat outwards from the planet.
    I would much appreciate any clarification you could offer.

  21. greg elliott says:

    Magic Turtle says:
    February 25, 2012 at 7:40 pm
    ‘Back-radiation’ is a fact. It is simply a consequence of the fact that all matter in the universe is radiating energy in all directions all of the time. This implies that the Earth’s atmosphere is radiating in all directions too.

    Radiation and conduction both occur from hot objects to cold and from cold objects to hot at the molecular level. It is quite easy to show that a cold molecule (low kinetic energy) can warm a hot molecule (high kinetic energy) through collision, further reducing the kinetic energy of the already cold molecule.

    This does not violate thermodynamics, because thermodynamics does not apply at the molecular level. Thermodynamics is a statistical affect that operates on averages over many molecules.

    Thus, while a “cold” molecule can warm a “hot” molecule, the probability this will happen is lower than the probability that a “hot” molecule will warm a “cold” molecule. Thus, thermodynamics tells us that as a result of statistical averaging that heat only flows from warm to cold objects.

    Thus, at the molecular level, radiation and conduction are two sides of the same coin.

  22. Richard111 says:
    February 25, 2012 at 8:09 am
    “One quibble, you mention the sun as the power source, but no mention of how “greenhouse gases” will shield the surface. That incoming sunlight has the full radiation range whereas the outgoing nightside radiation is strictly limited to the far infrared. There is a huge difference in performance there. Is this ever taken into account?”

    Is an interesting question as far as current GHE theory (but irrelevant to N&Z). There is also the disc to sphere ratio to consider when calculating power and the red shift caused by the different radiating temperatures. Not easy to tie down without measurements in the overlap area of both spectra. Perhaps CERES data might produce the necessary information.

  23. Richard111

    Sorry. I should have added that I believe GHE theory omits this effect.

  24. RKS says:

    Thank you Stephen,

    You’ve reinforced my understanding of the subject and provided some extremely useful information.

    I’m still waiting for N&Z’s work to be falsified and look forward to their Part 2.

  25. Stephen Wilde says:

    Acouple of contributors say that backradiation is a fact.

    It depends on what you call backradiation. AGW proponents say that backradiation is sent down to the surface by the entire atmosphere. It is that approach which I regard as a myth.

    Instead there is a dynamic energy exchange by all available processes between the surface and the air molecules directly above it. Part of that exchange involves air molecules near the surface radiating back to the surface. I accept backradiation if it is described in that manner but that is not what is meant by the AGW theory of backradiation.

    I don’t think the downward radiation from higher up makes any difference because the volume of the atmosphere simply changes in response to it so as to keep the surface temperature the same.

    Harry Hoffman takes the view that the atmosphere is independently heated directly by the incoming solar energy rather than significantly from upward conducton or radiation from the surface. I regard that as a separate issue which is not really the subject of this article.

    However the atmosphere is heated the fact is that the maximum concentration of molecules is placed at the surface by gravity and the highest temperature will be found at the surface for any given level of solar energy input.

    Steve Fox asks:

    “I read recently that heat could not be radiated into space since there are no molecules present to carry it. Therefore also that any notion of space being cold (or hot for that matter) was mistaken.”

    Hot and cold are relative concepts. The temperature of space is just 2 to 3 C above absolute zero. I think that is cold. There are widely separated molecules floating about in space so it is not a perfect vacuum.

    Energy can be radiated to space from the Earth’s surface and from GHGs in the air. Non GHGs do have some radiative capability but not a lot.

    Magic Turtle sad:

    “So we can see then that both principles of global warming – ie. back-radiation and gravitational, or ‘adiabatic lapse’ heating – are valid physical principles that are not mutually exclusive or incompatible but are natural complements to each other. They are both in operation in all planetary atmospheres that contain greenhouse gases.”

    I agree that both exist but I do distinguish them in my article. The gravitational component sets the lapse rate which cannot vary unless the gravitational field changes and when combined with solar input and atmospheric mass it defines the total energy that the system is capable of retaining.

    On the other hand, if any other factors (such as more/less GHGs, air circulation changes or variations in energy transfer from oceans to air) try to alter the total system energy content then, instead, atmospheric volume changes to alter the speed of enrgy transfer through the system to negate the effect on surface temperature.

    The change in atmospheric volume would be revealed by a change in the atmospheric heights such as the height of the tropopause and the effective radiating height.

  26. Mydogsgotnonose says:

    Hi magic turtle. I do not dispute the existence of radiated energy from a colder body to a warmer body. However, it can do no thermodynamic work because that part intercepted by the warmer body is a standing wave communicating information between the emitter/absorber states on both bodies.

    When the cooler body is at absolute zero, the Prevost exchange energy to give it its proper name is zero. When the temperatures are equal, it is the same as the radiation emitted by either body.

    However, it can still do no thermodynamic work and it can only be detected by blocking the energy from the warmer body to the colder body. By counting ‘back radiation’ with the energy emitted by the warmer body, Trenberth is increasing the S-B constant by a factor between 1 and 2. Think about it. It is fantasy physics.

  27. Stephen Wilde says:

    Mydogsgotnonose said:

    “I do not dispute the existence of radiated energy from a colder body to a warmer body. ”

    I don’t think anyone here does.

    However the S-B equations only apply as regards two separate bodies with a vacuum in between.
    S-B is a radiative only calculation.

    The AGW proponents treat the Earth and the atmosphere as two separate black bodies which is not right as I say in my article.

    The simple netting out of the two radiative flows cannot explain anything because of all the non radiative energy transfers going on between surface and air and within the air.

    Then there is that freely variable atmospheric volume to consider. Any additional energy in the air from more GHGs results in increased atmospheric volume which reduces the number of molecules per unit area at the surface. When that happens there are less molecules per unit area at the surface to interact with incoming solar energy which generates less heat so the temperature at the surface remains just the same as before.

  28. Mydogsgotnonose says:

    SW: what I think you’re trying to do is to analyse coupled convection and radiation. To do this mechanistically is very difficult and engineering texts make it dimensionless then give empirical effective heat transfer coefficient data.

    Your idea about molecules on the surface interacting directly with incoming solar radiation is wrong. An adsorbed gas molecule is ejected with extra energy from the kinetic energy of the solid. That kinetic energy is increased by incoming photons being absorbed in vibrational states then that energy is thermalised to kinetic energy.

  29. dp says:

    This can be picked at in several places but this one has me curious because it seems so simple:

    You say: ” However, if one applies the Gas Laws, ignoring radiation downward from the atmosphere the answer becomes clear. The presence of the table top blocks radiation from the ground which remains warmer than it would have been so less condensation occurs on the ground beneath the table. The top of the table radiates freely to space and becomes cold enough to reduce the air above it to below the dew point and so receives condensation.”

    The table blocks the radiation coming from the ground. Fine – but that doesn’t happen until that radiation has left the ground. Where does that energy go after it reaches the table? Back to the ground as back radiation? What about all that angular radiation from the ground that hits the table at an angle or misses the table entirely? And if it is the case that a table can produce back radiation then what whim of physics says CO2 cannot?

    What is particularly confusing is you almost seem to imply that the presence of the table prevents (blocks) the radiation from leaving the ground in the first place, and that is simply not possible. I’m going to assume that is not your intended explanation.

  30. Ray Tomes says:

    Hi Talbloke, I agree that there are effects that make higher air colder than lower air. It is just the particular argument given that I take issue with.

    I do find the gravitational component argument interesting. If a molecule falls, then it gains energy, if it goes upwards it loses energy due to gravity. That must make a temperature gradient. In fact, if molecules fell from infinity to Earth (or even a moderate distance) they achieve a velocity of 11.2 km/s (or significant part thereof). The molecules in air are only doing 0.5 km/s, so we should expect a much greater temperature gradient. Presumable it is much less because of losses due to radiation and convection. Regards, Ray

  31. Stephen Wilde says:

    “Your idea about molecules on the surface interacting directly with incoming solar radiation is wrong”

    Air molecules at or just above the surface either acquire extra kinetic energy from photons being absorbed by them them directly (GHGs) or in the case of non GHGs by conduction from the surface or conduction from GHGs that they might collide with.

    How is that wrong ?

    I am not analysing coupled radiation and convection at this stage, merely pointing out why the specific version of backradiation promulgated by AGW theory is not correct.

  32. Ray Tomes says:

    Stephen Wilde quotes me and then advances an argument. I see nothing in what you say that in any way attacks what I said. All those processes can apply equally to a molecule in a dense or rare medium.

  33. Stephen Wilde says:

    dp said:

    “What is particularly confusing is you almost seem to imply that the presence of the table prevents (blocks) the radiation from leaving the ground in the first place, and that is simply not possible”

    The radiation leaves the ground but is blocked from escaping to space by the table.

    There will be radiation bouncing to and fro between ground and table but it cannot escape to space except by being diverted sideways which does of course happen to some extent but the air under the table remains warmer than the air open to the sky and therefore the surface beneth the table will acquire less condensation (not necessarily none).

    “And if it is the case that a table can produce back radiation then what whim of physics says CO2 cannot?”

    CO2 and other GHGs can but at any significant distance from the ground the additional energy goes to increasing the volume of the atmosphere rather than causing a net decrease in the radiative flow from the ground.

    When the volume of the entire atmosphere increases (vertically) the number of molecules per unit of volume at the surface decreases which reduces the surface temperature to offset the warming of the surface that would have otherwise occurred from more energy in the air.

    The Gas Laws work both ways. The pressure may remain the same at the surface but if the atmosphere expands the surface will cool and if the atmosphere contracts the surface will warm.

    Add energy to the air and the atmosphere will expand and there will be no increase in surface temperature. The space under a table would not be large enough to make much difference but even there the warming effect of the table above would be somehat diminished by the warmer air beneath it containing a few less molecules per unit area than the colder air above the ground around the table.

    On a global scale it appears that the expansion of the atmosphere is enough to produce the observations reported by Harry Dale Hoffman and Nikolov & Zeller.

    To rebut my contentions you must first invalidate their data.

  34. Stephen Wilde says:

    Ray,

    Then I seem to have missed your point. Repeat it in clearer language and I’ll try again.

  35. Mydogsgotnonose says:

    SW: the great majority of molecules in air are N2 and O2 and there’s no way that SW or IR can interact with them. The mechanism of transfer of heat energy is to adsorbed molecules which then leave the surface.

    Ever seen a Crookes’ Radiometer?

  36. dp says:

    If the affect of energizing the atmosphere by ground radiation is to cause it to increase its volume then it becomes more or less sparse (molecules per unit if volume). This will modify the extinction range of the energizing radiation which is a measurable quantity. This N-Z hypothesis should be very trivial to prove – simply show that the extinction range correlates with changes in the specific GHG being tested for change.

    Back to the table above the grass analysis – how far above the grass does the table have to be before the back radiation becomes insignificant? What happens if the table is actually a parabolic reflector aimed at the earth? My curiosity is honest as explaining this adequately explains the rest.

    Somewhat ot, but there is quite a lot of money to be made exploiting the night sky radiation energy loss – the trick being to create a reservoir to lose heat during the night and to reheat it during the day. http://solarwall.com/posts/night-radiation-cooling-with-roof-mounted-solarwallr-panels21.php?p=10&g=3

    Does N-Z predict the air temperature at the bottom of a dry well will be colder or warmer than air at the surface? Pueblo Indian kivas are very much cooler than the surface air. At a glance that seems unlikely if N-Z is correct. These are the daytime form of your table on the grass. They provide relief from down welling radiation.

  37. sergeiMK says:

    Stephen Wilde says: February 25, 2012 at 12:04 pm
    Molecules absorb more photons from the sun either directly or via conduction from the surface. The additional energy in the molecules causes increased vibration of those molecules. That vibration is known as kinetic energy. That kinetic energy generates IR longwave which is recorded as heat in a sensor designed to display the level of heat as what we call temperature.
    ===========
    Come-on you cannot have it both ways – you are saying the kinetic energy of molecules including O2 and N2 create IR.
    But you say it is only GHGs that radiate to space cooling the planet.
    These are not compatible statements.

  38. Ray Tomes says:

    Stephen, the point is that having more heat (or more photons) because there are more molecules does not change the temperature. The temperature is a measure of heat per molecule.

  39. Stephen Wilde says:

    Ray Tomes said:

    “the point is that having more heat (or more photons) because there are more molecules does not change the temperature”

    More molecules per unit area results in more reactions with incoming solar photons within that area and so generates more kinetic energy (heat) within that area. THat increases the temperature.

    sergeiMK said:

    “Come-on you cannot have it both ways – you are saying the kinetic energy of molecules including O2 and N2 create IR.
    But you say it is only GHGs that radiate to space cooling the planet”

    O2 and N2 acquire the same ambient temperature as the GHG molecules around them at each level of the atmosphere. They therefore have kinetic energy. Most energy transfer within the atmosphere is to and from GHGs and non GHGS by conduction. The GHGs effect the vast majority of radiation to space from within the atmosphere.

    dp said:

    “If the affect of energizing the atmosphere by ground radiation”

    I was actually discussing the energising of the atmosphere by GHG molecules at varying heights.

    “how far above the grass does the table have to be before the back radiation becomes insignificant? ”

    Depends on what is meant by insignificant. On a still night maybe 10 feet or so.

    “Does N-Z predict the air temperature at the bottom of a dry well will be colder or warmer than air at the surface? ”

    Other effects dominate in small areas such as shelter from direct sunlight and exposure to air movement and local humidity.How much pressure difference would you expect to see in a dry well from top to bottom ?

    Mydogsgotnonose said:

    “The mechanism of transfer of heat energy is to adsorbed molecules which then leave the surface.”

    That would be conduction and convection which I mentioned.

    These questions are not helpful. Please read my article properly and make constructive comments. I’m sure it isn’t yet perfect. So far no one has queried a fundamental point that I have not covered.

  40. Chris M says:

    A couple of questions please for Stephen, Mdgnn and/or others:

    The first is the relative contribution of heat energy transfer via surface conduction to N2 and O2 molecules vs. collision of energized water vapour and CO2 with N2 and O2 molecules in the air, which is also a form of conduction if I understand correctly. I imagine that because the atmospheric portions of water vapour and CO2 are very small compared to N2 and O2 that the former would greatly predominate?

    The second relates to the reverse direction: If heat energy is transferred from an N2/O2 molecule to a H2O/CO2 molecule, is the energy thus acquired emitted as longwave infrared, and wouldn’t that then have a net cooling effect due to eventual release of LWIR to space?

    Lastly, how does incoming near-infrared (SW) radiation absorbed by water vapour contribute to atmospheric warming? Is this also heat energy, and is the energy re-emitted at the same wavelength, or as lower energy LWIR?

    Thanks for clarifying, with apologies in advance for any naivete displayed.

  41. tchannon says:

    Mydogsgotnonose,

    “Crookes’ Radiometer?”

    Oops. A little while ago I posted a link to an old text about the bizarre device which had and has many believing how it works, nope. This is I think a good illustration of how hard the subject can be.

    For a start why doesn’t it work if there is a hard vacuum in there?

    Here is the link, from Usenet Physics FAQ
    http://math.ucr.edu/home/baez/physics/General/LightMill/light-mill.html

  42. Stephen Wilde says:

    ChrisM:

    i) I don’t know the relevant proportions. What I do know is that somehow O2 and N2 have acquired the same ambient temperature as the GHG molecules around them at any given height and so must be participating fully in the so called Greenhouse Effect. Furthermore that implies that they acquired their kinetic energy by mostly non radiative means which suggests that the Gas Laws are more significant than radiative physics in setting up the Greenhouse Effect in the first place.

    ii) Yes and I think one must set that effect against the propensity to send energy downward. I have seen no evidence as to the net balance. For the purpose of my article I assumed that the net outcome was warming and suggested that even if that is correct the expansion of the atmosphere and the rise in the effective radiation height would prevent surface warming.

    iii) The creation of water vapour has a very substantial cooling effect at the surface. I suspect that it negates any downward IR from the molecules near the surface. If any IR were to reach the water from higher up it would negate that too.The latent heat is re emitted at a higher level when condensation occurs. The clouds then formed radiate it straight up as lower energy LWIR. I read somewhere that all outgoing radiation fromthe Earth is as lower energy LWIR. The whole planet converts incoming solar shortwave to outgoing LWIR.

  43. sergeiMK says:

    Stephen Wilde says: February 26, 2012 at 1:25 am
    O2 and N2 acquire the same ambient temperature as the GHG molecules around them at each level of the atmosphere. They therefore have kinetic energy. Most energy transfer within the atmosphere is to and from GHGs and non GHGS by conduction. The GHGs effect the vast majority of radiation to space from within the atmosphere.
    =============
    A very confused statement – do Non ghg molecules or ghg molecules, transfer the heat from the ground?
    However, you only aver sat that GHGs radiate to space. But they radiate in all directions somewhat less than 50% will hit the ground. somewhat more than 50% will radiate to space. so less than the total heat transferred upward (by whatever mechanism) will be lost.

    In your theories you currently state that GHGs cool the atmosphere not warm it. But you never state what this with respect to. No ghgs and the radiation escapes from the ground unrestricted with ghgs the radiation exiting the atmosphere is somewhat more than 50% of that leaving the ground. Surely this makes the ground warmer?

  44. Magic Turtle says:

    Hi Stephen

    You wrote (today 1:25am):
    “So far no one has queried a fundamental point that I have not covered.”

    OK then, let’s see if I can do that. 😉

    As you may know the current global mean temperature at the surface is approximately 15ºC.
    This implies that the Earth’s surface is radiating power at the mean rate of about 390 W/sq.m.
    But the mean solar irradiance at the surface amounts to only about 237 W/sq.m.
    Thus there is a shortfall in the surface energy/power-balance of about 153 W/sq.m.
    How do you imagine nature is making up this huge shortfall so as to maintain the mean surface temperature at ~15ºC?
    (The greenhouse theory explains it as recycled energy/power from the surface.)

  45. Ray Tomes says:

    Stephen wrote: “More molecules per unit area results in more reactions with incoming solar photons within that area and so generates more kinetic energy (heat) within that area. THat increases the temperature.”

    More kinetic energy shared between more molecules does NOT increase the temperature. Please go and check the physics. I say (for the last time) that temperature is a measure of energy PER molecule.

  46. Mydogsgotnonose says:

    rchannon: Crookes’ radiometer works because of the differential rate of accumulation of heat energy on the black side of the paddle compared with the silver side.

    This energy is transferred to adsorbed gas molecules and when more of these are ejected with higher momentum from the blackened side, it creates a differential force.

    My point was to show the other contributor how energy is transferred to gas molecules, heterogeneously. I did no claim the device worked in a vacuum.

    [ it’s a fun device. Sorry if I came across as attacking you, wasn’t my intention, which was alerting people to the strangeness surrounding radiation. –Tim]

  47. Richard111 says:

    @Ray Tomes

    My understanding is that any given VOLUME of gas can have a measurable temperature. I don’t see how you can define the temperature of a single molecule. When you talk about the temperature of a gas you refer to the AVERAGE temperature of ALL the molecules in that volume. Statisically, there will be a few molecules MUCH HIGHER than the measured temperature as well as a few molecules much lower than the measured temperature.
    http://galileo.phys.virginia.edu/classes/252/kinetic_theory.html

  48. Stephen Wilde says:

    Ray Tomes said:

    “More kinetic energy shared between more molecules does NOT increase the temperature”

    You forget the role of the incoming solar shortwave. It clearly raises the temperature and does so more where there are more molecules per unit volume than where there are less. You are assuming that the same amount of kinetic energy is being shared amongst more molecules which would result in a lower temperature but in fact the continuing solar irradiation results in more kinetic energy accumulating within the unit of volume containing a larger number of molecules. So more energy in total AND more energy per molecule because the closeness of the molecules to each other retains the energy in the parcel of air for longer during which time there are more molecular collisions.

    sergeiMK says:

    “In your theories you currently state that GHGs cool the atmosphere not warm it.”

    No I don’t. I say that IF they warm it then the negative system response of an expansion of the atmosphere will cancel it out. I have said that for all anyone seems to know the actual effect might be net cooling because of the upward radiation sending energy to space faster than non GHGs would have achieved it. That accords with what you are suggesting.

    “do Non ghg molecules or ghg molecules, transfer the heat from the ground?”

    Both transfer energy from the ground by conduction. GHGs transfer it effectively by radiation as well. Non GHGs do transfer some by radiation but not much. GHGs and non GHGs also transfer it back to the ground in the same ways hence the dynamc energy exchange betweeen surface and air.

    Magic Turtle said:

    “How do you imagine nature is making up this huge shortfall so as to maintain the mean surface temperature at ~15ºC?
    (The greenhouse theory explains it as recycled energy/power from the surface.) ”

    I say the same thing but say that the cause is the dynamic energy exchange between the surface and the lowest molecules of the air as explained by the Gas Laws and not from GHGs higher up as suggested by AGW theory.

  49. mkelly says:

    Magic Turtle says:”The radiation that is emitted in the direction of the surface, called ‘back-radiation’, actually reaches the surface because the atmosphere is perfectly transparent and at least some of it is absorbed there, thereby increasing the temperature at the surface by a definite amount.”

    You clearly do not know radiative heat transfer equations. If you think you do please write an equation showing how this takes place. Further since only heat or work can cross boundaries please write first law equations showing why the atmosphere does not decrease U to accomplish the heat transferred to the surface.

  50. Magic Turtle says:

    Stephen

    You wrote (25th Feb. 9:15pm):
    “Any additional energy in the air from more GHGs results in increased atmospheric volume which reduces the number of molecules per unit area at the surface.”

    Actually, that’s not true. The average number of molecules per unit area is simply the total number of molecules in the atmosphere divided by the earth’s total surface area. This is unaffected by changes in atmospheric volume.

    From your last comment above:
    “I say that IF (GHGs) warm (the atmosphere) then the negative system response of an expansion of the atmosphere will cancel it out.”

    I don’t think it will. The atmosphere’s expansion does not cause cooling. It is an effect caused by the atmosphere warming. All that it causes is the rising of the top of the atmosphere.

    “I have said that for all anyone seems to know the actual effect might be net cooling because of the upward radiation sending energy to space faster than non GHGs would have achieved it.”

    If the GHGs send radiation to space faster than non-GHGs, why wouldn’t they send radiation back to the surface faster than non-GHGs too?

    “Magic Turtle said:
    ‘How do you imagine nature is making up this huge shortfall so as to maintain the mean surface temperature at ~15ºC?
    (The greenhouse theory explains it as recycled energy/power from the surface.)’
    I say the same thing but say that the cause is the dynamic energy exchange between the surface and the lowest molecules of the air as explained by the Gas Laws and not from GHGs higher up as suggested by AGW theory.”

    I’m still not understanding you, I’m afraid. Why do you think the GHGs higher in the atmosphere cannot irradiate the surface with some of the excess energy that they exclusively have absorbed from the surface radiance? What, in your mind, is stopping this natural process from taking place?

    Let me state the basic greenhouse theory again as I see it to make sure that we are talking about the same thing. To my mind the greenhouse theory is saying that GHGs in the atmosphere are selectively absorbing IR-radiation emitted from the planet’s surface and are re-radiating the energy that it has absorbed both to outer space and back to the earth’s surface which warms as a result of having received this recycled radiation. Why shouldn’t that happen?

  51. mkelly says:

    Magic Turtle says: February 26, 2012 at 4:20 pm Why shouldn’t that happen?

    First you have been by several folks that 1. Thermal energy transfer only takes place because of a difference in temperature (a gradient) 2. If the atmosphere gives up energy to the surface then it must have less energy by the same amount. 3. A thing cannot heat itself and that is what you propose. 4. Radiation and ‘heat are not the same thing. 5. A ‘hotter object does not absorb ‘cooler’ object radiation.

  52. sergeiMK says:

    Stephen Wilde says: February 26, 2012 at 10:40 am
    “In your theories you currently state that GHGs cool the atmosphere not warm it.”

    No I don’t. I say that IF they warm it then the negative system response of an expansion of the atmosphere will cancel it out. I have said that for all anyone seems to know the actual effect might be net cooling because of the upward radiation sending energy to space faster than non GHGs would have achieved it. That accords with what you are suggesting.

    “do Non ghg molecules or ghg molecules, transfer the heat from the ground?”

    Both transfer energy from the ground by conduction. GHGs transfer it effectively by radiation as well. Non GHGs do transfer some by radiation but not much. GHGs and non GHGs also transfer it back to the ground in the same ways hence the dynamc energy exchange betweeen surface and air.
    ==============
    “If they warm it” Must have an answer on that – do ghgs radiate heat towards the earth and warm it (which is then cancelled by radiation to space).

    A word of warning here. for negative fb to work there has to be a change in the thing that is being negated. e.g. in an operational amplifier trying to keep its inverting input the same as its non inverting input there is always a difference required to generate the negative feedback – insignificant in high input impedance high open loop gain (200k) devices .. However the open loop gain of the earth system is not likely to be great. So your feed back (e.g. increased cloudiness) will require the temperature to increase until e.g. cloudiness albedo negates the additional energy input and then stay there.

    If you agree that GHGs radiate in all directions and some of that hits the earth then you should also realise that if the earth is getting 1kw/sqm from the sun and 0.1kw/sqm from ghg it will be warmer than if it was just receiving 1kw/sqm from the sun

    in a non ghg world the air will only be warmed by the ground by conduction, the ground will only be warmed by the sun. The non GHG cannot cool the planet but neither can it heat it above the value set by the albedo and TSI – it does not absorb energy from TSI directly. The ground receives 100% of the TSI and if at equilibrium Will radiate 100% tsi to space(at a lower temperature spectrum). A GHG will take some of the LWIR and “reflect” it to ground. The ground does not “know” where this extra energy comes from – it is simply an increase in total received irradiance.

    Perhaps filters and mirrors are a good way to visulise this.

    Pass sunlight through a visual light filter so only the visual part of the spectrum reaches a black body in a vacuum (BB) – this is energy so the absorbed part will warm the BB. The BB will radiate with a spectrum dependent on temperature. Place a mirror which reflects 50% of only LWIR and passes visual light in both directions, around the BB. The incoming radiation has not changed the BB has not changed, the vacuum has not changed. But now the BB is receiving 50% of its emitted LWIR back. This can only cause the BB to increase in temperature

    Are you suggesting that conduction is the only way for ghgs to warm the ground?

  53. RKS says:

    Magic Turtle says:
    February 26, 2012 at 4:20 pm>>>>

    Your views, as with many trained climate scientists, appears to be skewed by the radiative transfer concept.

    Have you by any chance studied the theory by Nikolov and Zeller advancing surface warming being due to atmospheric pressure and insolation alone?

  54. dp says:

    Lets try this again, different perspective. We know as a matter of science that certain frequencies of light have a measurable extinction distance. That is to say if we construct a laser that is centered on the absorption band of CO2 and turn it on, there is a distance at which it is undetectable.

    We can then conclude that all the energy in that laser beam has been absorbed by molecules (CO2 by design) that are absorbent at that frequency and that those molecules can and will re-radiate some of that energy at about the same frequency as absorbed, and that they will also transfer some of that energy by contact with other molecules.

    We also know that the extinction range is a shorter distance than the atmosphere is tall. This allows us to conclude that all IR energy that is generated at the surface is absorbed by the atmosphere and that it is distributed in a gradient, vertically, with the greatest energy given to those molecules closest to the surface.

    We also know from the science that the molecules, on average, re-radiate the absorbed energy in all directions. As a personal note – the CO2 molecules, being rather (American) football shaped, do not radiate equally in all directions, but the collection of molecules are arranged randomly which provides the 360º distribution. Anyway – some of the radiation goes down and is re-radiated from the surface. Some goes above the horizon. That which goes up energizes molecules beyond the extinction range of surface radiation because, well, they are above the surface. For molecules lower than the extinction range from the surface they are also heating all the in range CO2 molecules down to the surface as well as those above. Think about what that means. Down welling radiation and upwelling radiation are warming the intervening molecules concurrently. Now think about what that means.

    Further – the extinction range for these above surface molecules is greater as altitude increases than for the surface because the atmosphere becomes less dense with altitude (the horizontal extinction range at the surface is less than the vertical extinction range, by implication). There comes an altitude at which the extinction range is greater than the distance to TOA and that is the altitude at which energy at the frequency of absorption leaves the system for the space between the stars.

    Because of the symmetry of the re-radiation from the CO2 molecules and the described relaying of energy by radiation to the TOA and beyond, molecules above the extinction range of surface radiation will relay heat back to the surface by this same relaying. All this means is that energy makes several trips up and down before it finally escapes the system. This is lag, for those of you still awake, and it is this lag that creates the heat reservoir in the atmosphere.

    There is another interesting thing that happens as the atmosphere heats from the surface. It expands. Because of inertia in the atmospheric mass it cannot do this immediately – it takes time to propagate expansion – things have to move out of the way. That produces compression and a compression wave. What happens when you compress a gas?

  55. Stephen Wilde says:

    “Any additional energy in the air from more GHGs results in increased atmospheric volume which reduces the number of molecules per unit area at the surface.”

    That should read ‘ molecules per unit volume at the surface’.

  56. greg elliott says:

    Here is an simple thought experiment that shows the atmospheric dry air lapse rate is a result of gravity.

    Picture a square frame of reference with an single molecule of ideal gas inside. Place the molecule of gas at the center of the frame and at rest. This corresponds to a molecule of gas with no kinetic energy and its temperature is 0K.

    Now add gravity. The “bottom” of the frame is the surface of the planet. The molecule of gas will accelerate towards the planet, gaining kinetic energy as it falls. Its temperate – relative to our frame of reference – will increase. The temperature of the gas molecule will continue to increase until it hits the bottom of the frame (the surface).

    The molecule of ideal gas is perfectly elastic, so it will rebound from the surface with the same velocity it reached when it struck the surface. The molecule of gas will decelerate away from the planet, losing kinetic energy as it rises. Its temperate – relative to our frame of reference – will decrease. The temperature of the gas molecule will continue to decrease until its reaches its original location and temperature.

    This contradicts the notion that a column of air in a gravitational field will be iso-thermal. It also contradicts the notion that GHG is the cause of the dry air lapse rate.

    Another way to look at the problem is to consider Einstein and relativity. A column of air on earth is equivalent to a column of air inside a rocket that is accelerating at 9.8 m/s. That acceleration will push the molecules of air away from the direction of the acceleration, increasing the density and pressure of molecules towards the tail of the rocket.

    As happens with sound energy and light energy, there will also be the equivalent of a “Doppler Shift” in the energy of the molecules. They will not follow straight lines, they will follow parabolic trajectories that will increase their kinetic energy (temperature) towards the tail of the rocket and decrease their kinetic energy (temperature) to wards the nose of the rocket.

    If the acceleration is great enough, the molecules will never develop sufficient kinetic energy to reach the nose of the rocket. They will be cooled to 0K before they reach the nose and fall back towards the tail, leaving a vacuum inside the rocket at the nose and dense, hot air towards the tail.

    The key point is to recognize as did Einstein with Relativity, that the column of air exists within an accelerating frame of reference. Because we live on this frame of reference, we typically ignore this acceleration in our physical calculations, leading to incorrect results.

  57. dp says:

    “The key point is to recognize as did Einstein with Relativity, that the column of air exists within an accelerating frame of reference. Because we live on this frame of reference, we typically ignore this acceleration in our physical calculations, leading to incorrect results.”

    The other way of looking at this is that there is no acceleration (there isn’t) but that time is moving (it is). The length of a second at the surface is different than the length of a second half way to the moon.

  58. Magic Turtle says:

    Mydogsgotnonose

    I’m not sure what calling back-radiation ‘Prevost exchange energy’ adds to our enlightenment. Back-radiation is not a special kind of radiation; it is just that part of the atmosphere’s normal thermal radiation in all directions that happens to land on the surface. Isn’t it? Or have I got hold of the wrong end of the stick?

    As such, I don’t see why it shouldn’t be capable of doing thermodynamic work. As I understand it, when the back-radiation is absorbed at the earth’s surface it raises electrons from lower energy-levels to higher ones, which is a kind of thermodynamic work, I think you could say. And in raising the temperature of the earth it increases the earth’s ability to do thermodynamic work, albeit ever so slightly perhaps.

    Mkelly (Today 1:32pm):

    “You clearly do not know radiative heat transfer equations. If you think you do please write an equation showing how this takes place.”

    Please show how radiative heat transfer equations are relevant to the case in point before asking me to demonstrate my knowledge of them.

    “Further since only heat or work can cross boundaries please write first law equations showing why the atmosphere does not decrease U to accomplish the heat transferred to the surface.”

    If you have a point to make please make it in your own way and leave me to make my points in my way. I do not jump through hoops.

    mkelly (Today 5:16 pm):

    “Magic Turtle says: February 26, 2012 at 4:20 pm Why shouldn’t that happen?
    First you have been by several folks that 1. Thermal energy transfer only takes place because of a difference in temperature (a gradient)….”

    People tell me all sorts of things but I don’t necessarily believe them. Why should I believe this statement of yours that seems to conflict with well-known laws of physics? Thermal energy is transferred from cooler bodies to warmer ones by radiation. It is only the net flow that is always in the warmer –>cooler direction. But in the case that we are concerned with here, thermal energy is being transferred in the warmer –>cooler direction anyway because the radiation is being emitted by excited (ie. ‘hot’) molecular sources in the atmosphere and absorbed by unexcited (ie. ‘cold) molecular receivers at the earth’s surface. Where’s the problem in that?

    “2. If the atmosphere gives up energy to the surface then it must have less energy by the same amount.”

    Yes, I’ve already acknowledged that.

    “3. A thing cannot heat itself and that is what you propose.”

    No it isn’t. I never proposed, or even suggested any such thing.

    “4. Radiation and ‘heat are not the same thing.”

    I am well aware of that. But in energetic terms they are the same and their differences are not necessarily relevant to the case in hand, as here.

    “5. A ‘hotter object does not absorb ‘cooler’ object radiation.”

    Yes it does if the object is composed of bulk matter, ie. where the constraints of quantum-states have been overcome by the multiplicity of particles making up the object. The rule that you have cited applies only to individual molecules, atoms and particles, not to bulk matter.

    mkelly (Today 5.35 pm):

    “Have you by any chance studied the theory by Nikolov and Zeller advancing surface warming being due to atmospheric pressure and insolation alone?”

    No. And I can see no reason to do so either. I have already explained at length why I accept that the greenhouse effect is substantially mediated by the earth’s gravitational field (ie. by ‘pressure’). What I am trying to understand is why Stephen and some others here reject the idea of back-radiation as another possible mediating mechanism of the greenhouse effect. I am not claiming that back-radiation is large, or that it is even detectable with state of art technology. But we are dealing with theoretical matters here, not matters of practical experience, and as a theoretical postulate back-radiation appears perfectly sound and in accordance with long-established physics to me. So far no-one has shown me how it is unsound.

  59. greg elliott says:

    Here is a simulation on youtube that shows the dry air lapse rate, and warming of the surface due to increased kinetic energy of the molecules closer to the surface. Something that climate science says is caused by GHG.

  60. Stephen Wilde says:

    “What I am trying to understand is why Stephen and some others here reject the idea of back-radiation as another possible mediating mechanism of the greenhouse effect.”

    It is a matter of definitions.

    The Greenhouse Effect proposed by AGW enthusiasts suggests downward radiation from the entire atmospheric column from all the GHGs between surface and tropopause. That is what they mean by backradiation.

    The Greenhouse Effect according to the Gas Laws is a consequence of pressure and insolation alone. The maximum temperature occurs at the surface with no need for downward IR from GHGs up in the atmosphere. There is an energy exchange between the surface and the molecules at or just above the surface and that includes some downward IR from air to surface. That is backradiation in a limited sense but it is not what AGW proponents mean by it.

    The radiation from GHGs higher up in the atmosphere stays in the atmosphere because the atmosphere simply expands to accommodate the extra energy without warming the surface. That is implicit in the Gas Laws.

    Any warming of the surface beyond the normal energy exchange at the surface is prevented because the atmospheric expansion reduces the density of molecules just above the surface and that has a cooling effect as per the Gas Laws which offsets any warming effect from the GHGs.

    However, given that GHGs radiate upward as well I’m not convinced that there is any warming effect. I just say that if there were it would be negated anyway.

  61. Stephen Wilde says:

    Thanks for that video, greg, it makes the point nicely.

    There is a Grenhouse Effect but it is induced by pressure and insolation and is unaffected by downward IR from GHGs in the atmosphere.

  62. sergeiMK says:

    Magic Turtle says February 26, 2012 at 9:38 pm
    … why Stephen and some others here reject the idea of back-radiation as another possible mediating mechanism of the greenhouse effect. I am not claiming that back-radiation is large, or that it is even detectable with state of art technology…..
    =====================
    I have given links to documents that use many instruments to measure back radiation (the documents concern calibration of the instruments):

    Click to access LongWaveIrradianceMeas.pdf

    Click to access Marty2003_IPASRCII_JGR.pdf

    From the first document is this measurement made over a few days:

    The radiation on occasion is the same day and night! A considerable quantity that N&Z ignore.

    Any response to my previous post Mr Wilde?

  63. sergeiMK says:

    Stephen Wilde says: February 26, 2012 at 10:41 pm
    …The Greenhouse Effect proposed by AGW enthusiasts suggests downward radiation from the entire atmospheric column from all the GHGs between surface and tropopause. That is what they mean by backradiation.
    =============
    Radiation in the atmosphere
    No one suggests that photons starting at the top of atmosphere or even 1 meter above reach the earth. Each photon has a very small flight time before encountering another GHG molecule, being absorbed and almost instantly re-emitted in a random direction. It is the net flux that allows back radiation to exist. More than half up and less than half down.

  64. RKS says:

    Where does all this talk about the effects of back radiation fit in with the fact that on Venus, with an atmospheric CO2 concentration of 97%, the surface temperature is unaffected by CO2 with no boiling off of the atmosphere due to global warming.

    Along with several other planetary bodies, the observed temperature fits in precisely with the theory that surface temperature is caused by atmospheric pressure plus insolation.

  65. dp says:

    Somebody said:
    “The radiation from GHGs higher up in the atmosphere stays in the atmosphere because the atmosphere simply expands to accommodate the extra energy without warming the surface. That is implicit in the Gas Laws”

    That hardly seems likely given the IR radiation travels at the speed of light, 3 dimensions of freedom, and nothing to retain it. Radiated energy follows a relentless path up and out of the system. If it did not the planet would indeed fry us all. The path is indirect, of course, but like a Pachinko game the balls eventually fall through: http://www.youtube.com/watch?v=cMmm61D3P00

  66. kuhnkat says:

    dp,

    Stephen was talking about the downward directed radiation, not the upward radiation when he said it stayed in the atmosphere. He was talking about the fact that the density of GHG’s and atmosphere is the largest near the ground and not much LW gets past this area upwards or downwards.

  67. dp says:

    A molecule does not radiate in a single direction – they go off like a flash bulb radiating in all directions at once. The extinction range is not very far so the next molecule set is irradiated and re-radiates almost immediately in the same spherical pattern. There is a better than 50% chance the radiation will be above the horizon because the surface covers less than 180º even at low altitudes because the world is not flat even in a skeptical universe. The energy can’t not go up if it is radiative.

  68. jjthom says:

    dp says: February 27, 2012 at 5:07 am
    A molecule does not radiate in a single direction – they go off like a flash bulb radiating in all directions at once. The extinction range is not very far so the next molecule set is irradiated and re-radiates almost immediately in the same spherical pattern
    ===========
    I think that the radiation only gets emitted in one direction for each photon. This direction is in any direction at each emission. So not quite like a flash bulb!

  69. mkelly says:

    Magic Turtle says:”The radiation that is emitted in the direction of the surface, called ‘back-radiation’, actually reaches the surface because the atmosphere is perfectly transparent and at least some of it is absorbed there, thereby increasing the temperature at the surface by a definite amount.”

    Magic Turtle also says: “Please show how radiative heat transfer equations are relevant to the case in point before asking me to demonstrate my knowledge of them.”

    You say in the first quote that back radiation “The radiation…increasing the temperature at the surface by a definite amount” . So to answer your second quote “how radiative heat transfer equations are relevant” because you said they were as you attribute the increase in surface temperature to radiation (back radiation). Can you now demonstrate your knowledge of them?

    Mr. Turtle the language of science is mathematics. Asking you to write equations to demonstrate or prove your point is so I can fully understand what you are talking about. If you cannot I understand.

  70. Magic Turtle says:

    Those people who subscribe to the theory that global mean temperatures can be explained wholly by atmospheric pressure plus insolation are forgetting about the 1st law of thermodynamics. As I have mentioned above, there is a power-discrepancy of some 153 W/sq.m at the Earth’s surface between the amount supplied by insolation and the amount that the surface is actually radiating. Who or what is making up the shortfall? Mere pressure alone cannot be doing it because static pressure is not a power-source.

    The discrepancy is even greater on the surface of Venus where insolation accounts for only about 65 W/sq.m of the 16,730 W/sq.m actually radiating from the surface.

    It is this observed super-abundance of power at the surfaces of planets that demands some form of the greenhouse theory to explain it (as atmospherically recycled power from the surface). This gives rise to the question of how the required recycling is being achieved. There are two fundamental processes that I can see. One is back-radiation; the other is gravitation. They are distinct and separate processes, but they both depend upon the absorption of out-going surface radiation by GHGs in the atmosphere for their operation.

    I think subscribers to the “insolation + pressure” theory may simply have half-grasped the gravitational aspect of the power-recycling process and missed seeing how it relies on GHGs for its operation. Without GHGs there would be nothing to absorb the out-going radiant power from the surface and no power for the atmosphere to recycle. In that case there would only be the power of insolation at the surface and that cannot account for the super-abundance of radiant power that we actually find there.

    (Note to RKS: The reason for the Venusian atmosphere not boiling away with the heat of global warming is that although the Venusian surface is indeed blisteringly hot the top of its atmosphere from which the imagined boiling away would occur is very cold – a long way below 0ºC.)

  71. Magic Turtle says:

    Mkelly (Today 3:19pm)

    You say “…the language of science is mathematics”. I would say that mathematics is only one of the languages of science, which include plain English, and underlying them all is the universal language of logical thought. However, you appear to be insisting that I explain my thinking in the language of your choice and that is not reasonable. This is an English-speaking forum and so it is the language that I am using here. Not everyone speaks math but we can all be logical.

    Furthermore, the specific language that you have chosen for me to speak in, namely the language of radiative heat-transfer equations, does not seem (to me) appropriate to the situation that I was describing in English, because it did not just involve radiative heat-transfers. That is why I asked you to show how radiative heat-transfer equations would be relevant before asking me to try and prove my point with them.

    As I said, I don’t jump through hoops.

  72. Stephen Wilde says:

    “there is a power-discrepancy of some 153 W/sq.m at the Earth’s surface between the amount supplied by insolation and the amount that the surface is actually radiating. Who or what is making up the shortfall? ”

    That has been dealt with several times already.

    The surface is constantly exchanging energy with the air above via all types of energy transfer. About 153 Wm2 goes from surface to air and about 153Wm2 from air to surface constantly.

    Heat builds at the surface until the air molecules at or just above the surface are warm enough to match the flow from surface to air.

    The composition of the atmosphere is irrelevant save that if there were no GHGs the surface/air exchange would be almost entirely conductive.

  73. mkelly says:

    Magic Turtle says: “That is why I asked you to show how radiative heat-transfer equations would be relevant before asking me to try and prove my point with them.”

    Magic Turtle says: “The radiation…increasing the temperature at the surface by a definite amount” .

    You sir are the one who says the temperature increase of the surface is via radiation. So that is why radiative heat transfer equations are relevant.

    I have shown you why the equations are relevant per your request so please honor mine.

  74. jjthom says:

    Steven Wilde

    sergeimk proposed a simile above:

    Perhaps filters and mirrors are a good way to visulise this.

    Pass sunlight through a visual light filter so only the visual part of the spectrum reaches a black body in a vacuum (BB) – this is energy so the absorbed part will warm the BB. The BB will radiate with a spectrum dependent on temperature. Place a mirror which reflects 50% of only LWIR and passes visual light in both directions, around the BB. The incoming radiation has not changed the BB has not changed, the vacuum has not changed. But now the BB is receiving 50% of its emitted LWIR back. This can only cause the BB to increase in temperature

    Do you agree with that?

    I would then suggest that an encircling non GHG be added to the BB.
    With no ir mirror TSI in will equal black body irradiance (BBI) out at equilibrium There is no option here otherwise it is not at equilibrium. The non GHG cannot change the IR Flux out or in it does not absorb radiation.
    The gas will conduct heat to and from the BB at the point of contact but it cannot alter the flux in or out. The BB temperature remains the same as if it were not there – it just takes a bit longer to reach equilibrium because of the non GHG mass.

    Add in the 50 % ir reflector and again the BB is receiving 50% of the BBI +100% of the TSI. Its temperature will rise until again the emissions to space will equal the input.

    Note that there is nothing for the non GHG to conduct to other than the BB I just cannot see where it changes anything in this system.

    Your thoughts please

  75. Magic Turtle: I have not followed the entire proceedings to this point, and I only come here to inform, not to interminably discuss theory. I also only monitor comments directed at me, on the specific subjects of my posts, on my blog. But for what it’s worth, while I’m here, your statement that there is a shortfall of 153 W/m^2 supplied to the Earth’s surface is wrong on more than one account. First, the Earth’s surface cannot be taken to radiate as a blackbody–see my recent response to this question on my blog, at:

    Why not S-B at Earth’s surface?

    Also, using the numbers provided in the Kiehl-Trenberth Earth energy budget (1997), with a mean incident solar intensity of 342 W/m^2, 107 W/m^2 reflected by atmosphere and surface, and 67 W/m^2 directly absorbed by the atmosphere, then 168 W/m^2 are absorbed by the surface (and of the latter, 24 W/m^2 are convected back up as thermals, 78 W/m^2 removed by evapo-transpiration, and 40 W/m^2 transmitted directly to space through the atmospheric window–leaving 26 W/m^2 from the surface supposedly absorbed by the atmosphere). Since the surface is NOT a blackbody radiator, the huge loop of energy between surface and atmosphere known as the greenhouse effect is a chimera–and it is simply an empirical fact, from my Venus/Earth comparison at the first link above, that there is no greenhouse effect at all, of increasing temperature with increasing carbon dioxide.

    And you need to remember that 67 W/m^2 of incident solar energy directly absorbed by the atmosphere–the atmosphere is NOT transparent to all incoming solar radiation, as you seem to believe and as consensus climate scientists want the world to believe, as nearly 20% is directly absorbed, and that is what fundamentally warms the atmosphere and sustains the lapse rate structure, not heating from the surface.

    If you read my article at the Kiehl-Trenberth link above, you can see the most obvious error involved in the greenhouse effect and the supposed effects of backradiation: violation of the conservation of energy (that “huge loop” I mentioned involves more than the incident mean solar energy, and would require an energy pump, that does not exist). Another way to see the error is to note that the energy supposedly imparted to the surface by “backradiation”, from the cooler atmosphere, and re-emitted by the surface, would be re-emitted at the higher temperature of the surface, which again, would require a free energy pump.

    I don’t subscribe yet to any theory, only to the simplest and most definitive empirical and logical facts. I prefer to correct climate science from the ground up, not in one swell foop. You ought to be interested in my latest post,
    The True Energy Balance of the Earth+Atmosphere”

  76. “It is a mistake to then regard the atmosphere as radiating down to the surface because the atmosphere always radiates upward.”

    “As it is, GHGs radiate upwards as much as they radiate downwards and I have seen no evidence that the two directions of emission do not cancel out.”

    First, how can these two conflicting statements be reconciled? Second, how can the “two directions of emission” cancel out? One is to space, the other to the ground, and from the same source. The only cancelling out that might arguably occur is between atmosphere (GHGs) and the surface.

  77. Chris M says:

    http://theendofthemystery.blogspot.com.au/2012/02/true-energy-balance-of-earthatmosphere.html

    Tallbloke, someone with the moniker “margolin” has posted this comment on Harry Huffman’s site:

    “margolinFeb 25, 2012 08:15 PM

    Tallbloke says you effectively set albedo equal to zero. Which is unrealistic.”

    Agent provocateur?

    As far as I can tell HDH’s calculations are consistent with what N & Z are saying.

  78. Magic Turtle says:

    Stephen (5:10 pm):

    “ ‘there is a power-discrepancy of some 153 W/sq.m at the Earth’s surface between the amount supplied by insolation and the amount that the surface is actually radiating. Who or what is making up the shortfall?’
    That has been dealt with several times already.”

    I’m afraid it hasn’t. I think your explanation is overlooking the fact that solar irradiance is only providing ~237 W/sq.m, which is just enough to warm the surface to a temperature of about –19ºC (called by convention its ‘black body temperature’). In fact the Earth’s actual mean surface temperature is slightly less than +15ºC and therefore it radiates with a mean intensity of ~390 W/sq.m, whereby the power-discrepancy arises. But we could express the power-discrepancy in terms of temperature if that means more to you and say that the Earth has been raised above its baseline black body temperature by about 34ºC.

    So then, if the Earth had a perfectly transparent atmosphere (ie. with no absorptivity because no GHGs, except for clouds which we’ll ignore for now so as not to overcomplicate the picture) the Sun could only warm its surface to about –19ºC. Now, your argument appears to be that by some unspecified interactions between the surface and the bottom-most layer of the atmosphere that is in direct contact with it, it somehow raises its own temperature to +15ºC without the aid of the greenhouse effect. I think that idea violates the 1st law of thermodynamics as well as the 2nd.

    If this mysterious process of self-heating does not involve the greenhouse effect from GHGs, what does it involve? Surely we are left with just the basic unidirectional mechanisms of net heat-transfer of conduction, convection and radiation. But these are all processes which remove heat from the surface and reduce its temperature. There are none that can raise it any higher than it would be by virtue of solar irradiance by itself, ie. the ‘black body temperature’ of about –19ºC

    “The surface is constantly exchanging energy with the air above via all types of energy transfer. About 153 Wm2 goes from surface to air and about 153Wm2 from air to surface constantly.”

    I quite agree, but that is in an Earth-atmosphere that contains GHGs to provide the greenhouse effect. If the GHGs weren’t there, where could the 153 W/sq.m come from?

    “The composition of the atmosphere is irrelevant save that if there were no GHGs the surface/air exchange would be almost entirely conductive.”

    Conductive heat transfer alone only enables the surface to be warmed up to the level it started off at, ie. the black body temperature of –19ºC. The shortfall would still exist.

  79. Magic Turtle says:

    mkelly (Today 5:25 pm):

    “You sir are the one who says the temperature increase of the surface is via radiation. So that is why radiative heat transfer equations are relevant.”

    I don’t see how radiative heat transfer equations become relevant to my argument just because I mentioned the word ‘radiation’ in describing it. What questions would they answer? What doubts would they clear up? What new information would they provide? “None” is the only answer to all three of those questions that I can see.

    But if you have some objection to my argument based on radiative heat transfer equations I wish you would make it straightforwardly instead of demanding that I re-express my argument in those highly-technical terms apparently just to gratify your wish that I do so.

    “I have shown you why the equations are relevant per your request so please honor mine.”

    No you haven’t and as I said before, I don’t jump through hoops. Just because you ask me to do something it does not mean that you have the right to expect it of me.

  80. tchannon says:

    MagicTurtle,

    There is for Earth the addition of asymmetric heat transfer because the earth has day and night. I deliberately cooked that up to throw confusion into the zero ghg arguments. It is also real.
    Planet with gravity, transparent atmosphere, few posts back.

  81. Stephen Wilde says:

    Sorry, magic turtle but I must give up on you. Your imaginative reordering of reality is beyond my abilities to correct.

  82. Tenuc says:

    Magic Turtle says:
    February 27, 2012 at 8:18 pm
    “…solar irradiance is only providing ~237 W/sq.m, which is just enough to warm the surface to a temperature of about –19ºC (called by convention its ‘black body temperature’). In fact the Earth’s actual mean surface temperature is slightly less than +15ºC and therefore it radiates with a mean intensity of ~390 W/sq.m, whereby the power-discrepancy arises. But we could express the power-discrepancy in terms of temperature if that means more to you and say that the Earth has been raised above its baseline black body temperature by about 34ºC…”

    Don’t know where you got the information from, MT, but it is clearly wrong. For starters it is pointless using SB equation at the surface of the Earth as our Earth is a thick 3-D conducting sphere surround by a layer of mixed gases at high pressure. It is 70% covered by water and the remaining 30%, which is land, has varying emissivity, depending on its physical composition and whether it happens to be wet or dry e.t.c. The other major problem is that it is exposed to a climate system driven by spatio-temporal chaos and this dynamic non-linear system ensures that temperature and energy in or out are never in equilibrium, rather they oscillates over a number of different overlapping quasi-cycles.

    Read Harry Huffman’s post above if you want to understand how to measure the radiative effect on our planet and others in the solar system to circumvent some of the above problems.

    Composition of atmosphere has no effect on planetary long-term temperatures.

  83. Magic Turtle says:

    Hi tchannon

    “There is for Earth the addition of asymmetric heat transfer because the earth has day and night.”

    Yes, I did read your post and I am aware of this issue. It seems to make the power-discrepancy at the surface bigger too. Thanks for reminding me of it.

  84. D J Cotton says:

    If the AGW conjecture is to be overturned the truth must be proven empirically and promulgated emphatically. I suggest it runs along these lines …

    The reason radiation from a cooler object slows down the radiated heat transfer to itself from a warmer body is not because there is a compensating transfer of thermal energy back to the warmer body, because such would violate the Second Law. Rather it is because a standing wave is established which is represented by all the area under the Planck curve for the cooler body. This area represents the frequencies and intensities that are common to both the warm and cool objects.

    The atmosphere (with over 50 gases and water vapour) does not radiate everything that a true blackbody would, but water vapour does help fill the area under that curve. So there is a standing wave, but its total power is not as much as a true blackbody. This is why some radiation escapes directly through the atmospheric window.

    The standing wave has no thermal effect because none of its energy is ever converted to thermal energy. It just sends information back to the warmer body and a part of the warmer body’s radiation goes into the standing wave. The energy radiated by the warmer body which is represented by the area between the curves does get converted to thermal energy because it cannot resonate and thus contribute to the standing wave. The calculations of course agree with accepted physics, but the mechanism is not a two-way transfer of heat, as many appear to have supposed.

    But there is no build up of the effect of carbon dioxide due to multiple repetitions of the capturing and re-emitting process envisaged in the IPCC energy diagrams and models. Each carbon dioxide molecule can only play a single role in a very limited sub-section of the total standing wave. Its contribution per molecule would be no more than a molecule of water, and so its total overall effect is comparable with its relative proportion to WV and other emitters in the atmosphere – insignificant.

    Furthermore, there must be a compensating effect for reduced radiation by way of additional evaporation, diffusion etc because the very stable temperatures not far underground will be reflected in the close thermal equilibrium at the surface / atmosphere interface.

  85. sergeiMK says:

    Steven Wilde
    Could scientifically criticise my model (extended by jjtoms) repeated below
    Perhaps filters and mirrors are a good way to visulise this.

    Pass sunlight through a visual light filter so only the visual part of the spectrum reaches a black body in a vacuum (BB) – this is energy so the absorbed part will warm the BB. The BB will radiate with a spectrum dependent on temperature. Place a mirror which reflects 50% of only LWIR and passes visual light in both directions, around the BB. The incoming radiation has not changed the BB has not changed, the vacuum has not changed. But now the BB is receiving 50% of its emitted LWIR back. This can only cause the BB to increase in temperature

    Do you agree with that?

    I would then suggest that an encircling non GHG be added to the BB.
    With no ir mirror TSI in will equal black body irradiance (BBI) out at equilibrium There is no option here otherwise it is not at equilibrium. The non GHG cannot change the IR Flux out or in it does not absorb radiation.
    The gas will conduct heat to and from the BB at the point of contact but it cannot alter the flux in or out. The BB temperature remains the same as if it were not there – it just takes a bit longer to reach equilibrium because of the non GHG mass.

    Add in the 50 % ir reflector and again the BB is receiving 50% of the BBI +100% of the TSI. Its temperature will rise until again the emissions to space will equal the input.

    Note that there is nothing for the non GHG to conduct to other than the BB, I just cannot see where it changes anything in this system.

  86. Magic Turtle says:

    Harry Dale Huffman (27th Feb. 6:42 pm):

    “Magic Turtle: I have not followed the entire proceedings to this point, and I only come here to inform, not to interminably discuss theory.”

    Same here. However I find myself having to discuss theory apparently interminably because I am being confronted by objections to my arguments by a number of people who appear to want to rewrite the physics textbooks without having read them first.

    “But for what it’s worth, while I’m here, your statement that there is a shortfall of 153 W/m^2 supplied to the Earth’s surface is wrong on more than one account. First, the Earth’s surface cannot be taken to radiate as a blackbody–….”

    Yes it can. It is not a perfect black body, but then nothing is. However it is as close to being one as anything gets in the real world on infra-red wavelengths, which are the wavelengths with which we are concerned here.

    I have visited the web-sites to which you referred me and seen your explanation of why you think the Stefan-Boltzmann law should not be applied to the Earth’s surface and I have to say that I think your argument is flawed. The S-B law applies to all macroscopic bodies independently of their thermodynamic relationships with other bodies and not just at equilibrium either. It is simply a statement of the mathematical relationship between the absolute temperature of a body and its radiant intensity, said radiation occurring exactly the same irrespective of the radiations emanating from other bodies in its environment.

    “Also, using the numbers provided in the Kiehl-Trenberth Earth energy budget (1997), with a mean incident solar intensity of 342 W/m^2, 107 W/m^2 reflected by atmosphere and surface, and 67 W/m^2 directly absorbed by the atmosphere, then 168 W/m^2 are absorbed by the surface (and of the latter, 24 W/m^2 are convected back up as thermals, 78 W/m^2 removed by evapo-transpiration, and 40 W/m^2 transmitted directly to space through the atmospheric window–leaving 26 W/m^2 from the surface supposedly absorbed by the atmosphere).”

    I am familiar with that paper. I regard it as pseudoscience because the figures that you quote from it as their empirical findings were in fact the products of computer model simulations, not physical observations of the real world as the unsuspecting reader would assume.

    “Since the surface is NOT a blackbody radiator, the huge loop of energy between surface and atmosphere known as the greenhouse effect is a chimera–and it is simply an empirical fact, from my Venus/Earth comparison at the first link above, that there is no greenhouse effect at all, of increasing temperature with increasing carbon dioxide.”

    I can only say that you appear to me to be rejecting the validity of some basic laws and findings of physics here. According to modern physics, the Earth’s surface is effectively a black body radiator and I have not seen any convincing arguments to the contrary. I’m sorry.

    However, I do not dispute your empirical findings regarding the Venusian CO2-greenhouse. On the contrary, from my own calculations from basic physics, I have found the CO2-greenhouse effect to be wholly inadequate to explain the vast radiative-power discrepancy at the Venusian surface. Likewise on Earth I have found – again, from basic physics – that the greenhouse effect from CO2 is negligible, as well as being at variance with the effect predicted from the IPCC’s logarithmic equation for CO2 climate-forcing. In fact, I cannot find – anywhere in the solar system – any demonstration of CO2’s proclaimed global warming potency that the entire world is being turned upside down in order to mitigate. That really is a chimera to my mind.

    Nevertheless, there is another Venusian greenhouse besides CO2 constituted by the thick and dense cloud-layer of complex sulphurous oxides dissolved in water that cover the entire planet’s surface like a million-tog duvet. These larger kinds of molecules, containing many atoms, possess far larger IR-absorption wavebands than CO2 so we should not be surprised if they have a correspondingly larger greenhouse effect too. We cannot yet assume that there is no greenhouse effect on Venus just because we have found that CO2 does not fit the bill.

    “the atmosphere is NOT transparent to all incoming solar radiation, as you seem to believe”

    I don’t know where you can have got the impression that I believe this, unless it was from a misreading of my original post here in which I postulated a hypothetical planet which has an atmosphere that is perfectly transparent on all wavelengths for purposes of comparison when making my original argument.

    “If you read my article at the Kiehl-Trenberth link above, you can see the most obvious error involved in the greenhouse effect and the supposed effects of backradiation: violation of the conservation of energy (that “huge loop” I mentioned involves more than the incident mean solar energy, and would require an energy pump, that does not exist). Another way to see the error is to note that the energy supposedly imparted to the surface by “backradiation”, from the cooler atmosphere, and re-emitted by the surface, would be re-emitted at the higher temperature of the surface, which again, would require a free energy pump.”

    I’m sorry, but your arguments make no sense to me. They appear to be based upon a kind of physics that I never learned, was never taught and which I cannot find in any standard reference works. According to the physics that I know, the greenhouse effect is a logical consequence of the laws of radiative and gravitational energy-exchanges in accordance with the laws of thermodynamics, not in conflict with them. Your proposition of the greenhouse effect as a chimera seems to be a chimera itself to me. You say that the Earth cannot be treated as a black body on IR-wavelengths but accepted standard physics says it can. You say that we cannot apply the S-B law to the planet’s surface, but accepted standard physics says we can. You say that there is no discrepancy between the black body temperature and the actual temperature at the planetary surface, but accepted standard physics says there is. Sorry, but I’m sticking with the physics that I know and understand unless or until I see good reason to change my mind about some part of it. And so far I haven’t seen any.

    “I don’t subscribe yet to any theory, only to the simplest and most definitive empirical and logical facts. I prefer to correct climate science from the ground up, not in one swell foop.”

    Fine, but where is the ‘ground’? I don’t think you can correct climate science by knocking out the accepted laws of physics on which it rests from under its feet.

  87. wayne says:

    Can someone please point me to a pre-IPCC era physics book, maybe specific in atmospheric physics, that defines ‘back-radiation’? The earliest reference I can find on the web of this new term is here:
    http://www.ipcc.ch/publications_and_data/ar4/wg1/en/ch3s3-4-4-2.html
    “Of course, back radiation from greenhouse gases and clouds operate in the opposite direction (Philipona and Dürr, 2004).”

    (Stephen: some how I missed this entire post but your doing fine from what I have read so far, I may have some surprising info on N&Z’s relation but it’s taking a while to assimilate. I’ve even found a surprising way to look at SB in relation to pressure and density and via “temperature”. I, for one, will never look at “temperature” the same)

  88. Stephen Wilde says:

    “violation of the conservation of energy (that “huge loop” I mentioned involves more than the incident mean solar energy, and would require an energy pump”

    There is no violation.

    The energy exchange between air and ground at the surface of any planet with an atmosphere is dictated by pressure at the surface.

    It can be anything from zero (no atmosphere) to a temperature high enough to provoke fusion reactions (a star).

    There is no reason why it should be limited to the incident mean solar energy.

  89. Nick Stokes says:

    wayne says: February 28, 2012 at 3:01 am
    “Can someone please point me to a pre-IPCC era physics book, maybe specific in atmospheric physics, that defines ‘back-radiation’?”

    It’s been known, and measured, for a very very long time. Here, starting about p 75, is a description of measurement by Pouillet, 1837. He’s doesn’t have S-B, and he talks in temperatures – I think he’s assuming flux is proportional to temp. But he’s measuring it.

  90. ferd berple says:

    Does anyone know of a good simulator that will model an ideal gas with gravity?

    Playing around with what I’ve found quickly on the internet, it seems quite obvious that the exchange of kinetic and potential energy explains the dry air lapse rate.

    In the limited models I’ve played with at the bottom of the gravity well there are many fast moving balls, while at the top of the gravity well there are a few slow moving balls. This would appear to duplicate what we see in the atmosphere. High pressure and temperature at the bottom, low pressure and temperature at the top.

    What I find fascinating is that for 150 years science has assumed that the temperature of a column of air in a gravity field without GHG will show no lapse rate. Except for one amateur scientist it appears that this has never been experimentally tested. A computer simulation would appear to form the basis for a paper, as it appears to predict that a column of air in a gravity well will be colder at the top than bottom, even without GHG.

    It reminds me of the assumption that heavier objects fall faster than light objects. As an interesting side-note, this also explains the heating of a star towards the core before ignition. Not as a result of the compression of gas – which many rightly point out requires the continued and ongoing collapse of the star. Rather, as the increase in kinetic energy of the molecules toward the center of mass due to the acceleration of gravity.

  91. Magic Turtle says:

    Stephen Wilde wrote (Feb. 27, 8:58 pm):

    “Sorry, magic turtle but I must give up on you. Your imaginative reordering of reality is beyond my abilities to correct.”

    You must do it? Really? Are you not making a free-willed choice?

    Your statement suggests that you have felt a responsibility to correct my view of reality. I think that burden would have been an unnecessary one for you to carry. My views are and always have been my own responsibility to correct, not yours. I have no wish for you to continue carrying that self-imposed burden, so if you are now choosing to give it up that’s fine by me.

    But how do you know that my view of reality needs any correcting as your statement implies that you do? Do you even know what my view of reality is? Surely it is only your own view of reality that you can know and judge as being ‘correct’ or ‘incorrect’. Are you sure that your inability to correct my views is not an unconscious displacement onto me of an inability on your part to correct your own?

    Of course, I cannot correct your views either and since I already knew this from the start I haven’t been trying to correct them. I have only tried to point out where I see them being at variance with the findings, implications and laws of modern physics and to leave you to correct your view of reality wherever and however you see it needing to be corrected.

  92. PaulK says:

    “The idea is that certain gases in the atmosphere known as Greenhouse Gases absorb and emit more radiation than the other, majority, gases such as Oxygen and Nitrogen which make up the vast bulk of Earth’s atmosphere.

    Consequently it is proposed that such Greenhouse Gases block radiation emanating from the Earth’s surface from escaping to space and re- radiate a proportion of such upward radiation back to the surface which then becomes warmer than it otherwise would have done.”

    Gases have characteristic spectra and same do absorb IR radiation more effectively than others, water vapour especially. The downwelling radiation of the atmosphere is just the result of the air temperature near the surface. I have no idea, why some people want to call it “backradiation”. Anyway it is not the cause of the surface getting warmer with GHGs. The reason is that GHGs do block a large part of the IR radiation of the Earth surface from disappearing to the space directly.

    You say:
    “As it is, GHGs radiate upwards as much as they radiate downwards and I have seen no evidence that the two directions of emission do not cancel out.”

    You are obviously considering a thin layer of air. It does radiate as much upwards and downwards. However, the pressure and the temperature in the atmosphere near the surface are much higher than at the top of the troposphere. That is why the atmosphere as a whole radiates more downwards than upwards.

    This difference makes also the GHG effect work. GHGs hamper also their own radiation. The more GHGs there is the lower the “backradiation” comes from and the higher the GHGs radiation gets out to the space. Since it is colder and the air is thinner higher in the atmosphere, the GHGs radiate less effectively (I ~ T^4) high. So, when the concentration of GHGs increase, because of the lapse rate also the surface has to get warmer and not just the upper troposhere before the Earth is in the energy balance again.

  93. mkelly says:

    Magic Turtle says:
    February 27, 2012 at 8:52 pm
    But if you have some objection to my argument based on radiative heat transfer equations I wish you would make it straightforwardly instead of demanding that I re-express my argument in those highly-technical terms apparently just to gratify your wish that I do so.

    As to this I did I said a thing may not heat itself, but you failed to grasp that. If you could write out an heat transfer equation for radiation you would see why you are wrong. I also said that all heat transfer must have a temperature gradient. But you fail to grasp that, too.

    So thank you. We now know three things about you. You are not knowledgable, you are not honorable, and you are poking your head from the wrong end of your shell.

  94. Magic Turtle says:

    mkelly (Today 9:19 pm):

    “ ‘But if you have some objection to my argument based on radiative heat transfer equations I wish you would make it straightforwardly instead of demanding that I re-express my argument in those highly-technical terms apparently just to gratify your wish that I do so.’
    As to this I did I said a thing may not heat itself, but you failed to grasp that. If you could write out an heat transfer equation for radiation you would see why you are wrong.”

    I am not wrong. You are, because you misread my comment and got hold of the wrong end of the stick about what I was saying. The paragraph from which you have been quoting related to the situation of a hypothetical planet that has a perfectly transparent atmosphere on all wavelengths (ie. no GHGs). The specific sentence that you quoted as the one with which you have a problem was:

    “The radiation that is emitted in the direction of the surface, called ‘back-radiation’, actually reaches the surface because the atmosphere is perfectly transparent and at least some of it is absorbed there, thereby increasing the temperature at the surface by a definite amount.”

    But if you had read on to the end of that paragraph would have found:

    “But because it is back-radiated energy that came from the surface originally, whereby the surface temperature would have been diminished by an even greater amount than the back-radiation (the rest being radiated away ultimately into outer space) it is unable to restore the surface temperature to what it would have been if the surface had not warmed the atmosphere initially.”

    As you could have seen if your mind had been open to seeing it, I was not suggesting that a thing could heat itself; on the contrary, I was suggesting that it couldn’t and was explaining why in this particular case it couldn’t. So the problem that you were seeing did not exist. Your mind invented it and misread it into what I said. If you would care to construct an energy-balance equation to represent the situation yourself, I think you will see that I have been correct in all that I have said.

  95. Stephen Wilde says:

    “The reason is that GHGs do block a large part of the IR radiation of the Earth surface from disappearing to space directly”

    Applying the Ideal as Law it is more likely that they cause an expansion of the atmosphere, a change in the environmental lapse rate and no change in surface temperature.

  96. Stephen Wilde says:

    That should be ‘Ideal Gas Law’.

  97. curious george says:

    Stephen – your garden table experiment has another possible interpretation: Water condenses mostly from above. Take a rain (or a drizzle) instead of dew, would it still be surprising that the top of the table is wet and the bottom and the grass underneath are not?

    I have serious doubts about climate models. I believe they are not good enough to make any reliable prediction – where is a good 100-hour weather forecast? But we should counter AGW crowd with arguments clearly better that their own.

  98. sergeiMK says:

    Steven Wilde

    I would be really interested in your comments on my “model” mentioned above.:

    Pass sunlight through a visual light filter so only the visual part of the spectrum reaches a black body in a vacuum (BB) – this is energy so the absorbed part will warm the BB. The BB will radiate with a spectrum dependent on temperature. Place a mirror which reflects 50% of only LWIR and passes visual light in both directions, around the BB. The incoming radiation has not changed the BB has not changed, the vacuum has not changed. But now the BB is receiving 50% of its emitted LWIR back. This can only cause the BB to increase in temperature

    Do you agree with that?

    I would then suggest that an encircling non GHG be added to the BB.
    With no ir mirror TSI in will equal black body irradiance (BBI) out at equilibrium There is no option here otherwise it is not at equilibrium. The non GHG cannot change the IR Flux out or in it does not absorb radiation.
    The gas will conduct heat to and from the BB at the point of contact but it cannot alter the flux in or out. The BB temperature remains the same as if it were not there – it just takes a bit longer to reach equilibrium because of the non GHG mass.

    Add in the 50 % ir reflector and again the BB is receiving 50% of the BBI +100% of the TSI. Its temperature will rise until again the emissions to space will equal the input.

    Note that there is nothing for the non GHG to conduct to other than the BB, I just cannot see where it changes anything in this system.

    I have been talking with a friend about solar water heating. When it came to evacuated glass tubes we suddenly connected them with N&Z theory.

    Each tube is like a mini airless world with a black body absorber and a vacuum. The heat transfer fluid in these can actually blow up the tubes when solar radiation causes it to boil at greater than 100C. Now these are placed at ground level so the TSI input is the same as the earth receives. But with zero air pressure What would N&Zs equation predict as the absorber temperature?

    [ not an N&Z matter, is purely about flux, an energy flow, if impeded energy then dissipates power, constraint 5500K, that’s my view but I am not really taking part here –Tim co-mod]

  99. PaulK says:

    PaulK: “The reason is that GHGs do block a large part of the IR radiation of the Earth surface from disappearing to space directly”

    Steven Wilde: “Applying the Ideal Gas Law it is more likely that they cause an expansion of the atmosphere, a change in the environmental lapse rate and no change in surface temperature.”

    So, you agree that if there is no change in the lapse rate, my text above is correct?

    The lapse rate varies in time and location, but the avarage is considered guite stable. The lapse rate is a result of gravity and the properties of air. When there is practically no change in either, how do you make the lapse rate change?

  100. Stephen Wilde says:

    “So, you agree that if there is no change in the lapse rate, my text above is correct?”

    You must distinguish between the adiabatic lapse rate set by gravity and the environmental lapse rate set by composition.

    The energy flows through the system are juggled between the two until solar shortwave in matches longwave out.

    When GHG quantities increase the atmosphere expands a bit so that the thermal effect of the reduction of density at the surface offsets the thermal effect of a rise in effective radiating height so as to leave surface temperature unchanged but a slight change in atmospheric circulation due to an altered slope for the environmental (not adiabatic) lapse rate.

    Only the sun irradiating the surface alters surface temperature.

    Warmer air doesn’t alter surface temperature. It just alters the effective radiating height.

    Apply the Ideal Gas Law for any thermal phenomena within the atmosphere.

    Only apply the S-B Law from a point outside the atmosphere.

    The effective radiating height has already been affected by atmospheric composition and so cannot be used as the ’surface’ for the purpose of applying the S-B Law.

  101. Magic Turtle says:

    Stephen

    The S-B law is conceived as being universal. This means that it is held to apply just as much inside the Earth’s atmosphere as outside it. Your decision to restrict its jurisdiction to space outside of the atmosphere appears arbitrary, unjust and a denial of nature’s sovereignty to me.

    The penalty for this offence is to be confined to the blogosphere for the rest of your natural life and to be harassed interminably by unscientific thinkers like yourself who have rejected the rule of natural law and whose minds are in chaos because of it.

    Best wishes
    MT

  102. Stephen Wilde says:

    “The S-B law is conceived as being universal”

    Of course it is but one has to apply it correctly.

    Within the atmosphere another universal law is in command namely the Ideal Gas Law.

    How do you reconcile their competing claims ?

  103. PaulK says:

    Stephen Wilde: “You must distinguish between the adiabatic lapse rate set by gravity and the environmental lapse rate set by composition.”

    They both are set by gravity and by composition and you didn’t get my point. I have tried to understand your text.

    Stephen Wilde: “When GHG quantities increase the atmosphere expands a bit so that the thermal effect of the reduction of density at the surface offsets the thermal effect of a rise in effective radiating height so as to leave surface temperature unchanged but a slight change in atmospheric circulation due to an altered slope for the environmental lapse rate”

    But I cannot. How and why the reduction would cancel the effect of the rise in effective radiating height? How and why would the lapse rate change? My previous point was that it is quite stable.

  104. Stephen Wilde says:

    PaulK

    I don’t see that the gravity induced lapse rate for a theoretical ststic atmosphere is affected by composition. Can you provide evidence that it is ?

    The actual real world lapse rate varies greatly as a result of composition such as in the stratosphere where ozone reverses it.

    The actual real world lapse rate is variable due to changes in the composition and movement of air parcels.

    According to the Ideal Gas Law the surface temperature is in part dependent on gas density at the surface which is itself a function of molecules per unit of volume.

    Greater density for a higher temperature and less density for a lower temperature.

    If a given input of energy to a parcel of air causes it to expand upward then the density at the surface will decline because the same number of molecules is being distributed across a larger volume.

    The decline in density will be directly related to the amount of expansion so that the thermal effect of the two changes(density and volume) cancels out.

    Normally a higher effective radiating height would imply a hotter surface if one applies the S-B Law but if the density at the surface decreases the Ideal Gas Law says the surface temperature will be lower.

  105. Magic Turtle says:

    Stephen (29th Feb. 8:57 pm)

    ‘ “The S-B law is conceived as being universal”

    Of course it is but one has to apply it correctly.’

    Indeed one does, but that’s not the same as prohibiting its application as you appear to be doing. You cannot just arbitrarily suspend a law of nature and you are exceeding your authority by trying to do so. 😉

    Banning the S-B law is not to ‘apply it correctly’. It is a very powerful law which enables us to determine the radiative intensity of a body from its known temperature and vice versa with unfailing reliability and accuracy, independently of all extraneous factors and considerations. It can be applied to all bodies which radiate and all bodies which have temperature in all situations and at all levels of universal structure above the quantum level. In short it can be applied to all macroscopic bodies in one way or the other.

    Applying it directly to the measured solar-power that is absorbed at the Earth’s surface yields the result that solar power alone can heat the planetary-surface only to a global mean temperature of about –18C. Applying it directly (but in the opposite direction) to the measured actual global mean surface temperature of about +15C yields the result that the Earth must be radiating power with a global mean intensity of about 390 W/sq.m.

    Now the 1st law of thermodynamics demands that an additional source of power must be found to account for the 33C elevation of the surface above its –18C solar baseline temperature. It does not specify what form that extra power must come in, whether by conduction, convection, radiation, or some other form, but only that it must come from somewhere and in accordance with all the other laws of nature too. The greenhouse theory attempts to answer that demand by proposing that the required extra power comes from GHGs in the atmosphere that intercept and absorb outgoing IR-radiation being emitted from the Earth’s surface, which the atmosphere then recycles back to the surface in ways that have not yet been fully defined although with radiation playing some part. No inconsistencies appear to have been found between this theory and the existing laws of physics up to now and no other, alternative sources of extra power to the surface appear to have been found.

    “Within the atmosphere another universal law is in command namely the Ideal Gas Law.”

    From the standpoint of modern science the Ideal Gas Law is not in total command of the atmosphere. It describes only the ways in which pressure, volume and temperature will vary with respect to one another when energy is introduced to, or removed from the gas in question when any constraints on the system containing the gas are known. But it is not in command of the processes whereby energy may be introduced to, or removed from the system. Those are under the command of other laws whose jurisdiction extends beyond the boundaries of the gas.

    “How do you reconcile their competing claims ?”

    From the standpoint of science they are not making competing claims. On philosophical grounds, science makes the fundamental assumption that the laws of nature are one and perfectly harmonious and coherent (unlike man’s laws which are multitudinous and largely incoherent). The individual laws of science are simply human attempts to formulate, however imperfectly, the different ways in which the One Perfect Law of nature applies to different situations. So all of the formulated laws of nature apply everywhere and to every situation in principle (because the Law is One) and no competition or conflicts between them can ever possibly arise.

  106. Stephen Wilde says:

    Everything that you say GHGs must be doing is in fact being done by the physics of the Ideal Gas Law.The only difference is that you think it is GHGs alone whereas the IGL says it is every molecule in the atmosphere.

    Whether the GHE is caused by GHGs or by pressure at the surface via the IGL all your objections apply to both so in arguing against the role of the IGL you are also arguing against the role of GHGs and in arguing for the role of GHGs you are also arguing for the role of the IGL.

    You are facing both directions at the same time but do not realise it.

    The role of the IGL in fixing surface temperature for a planet with an atmosphere is settled science predating your radiative theory.

    The S-B Law is neither contradicted nor denied. The IGL simply deals with a situation that the S-B Law was never designed to deal with. It is supplementary to the S-B Law for application only where there is an atmosphere.

  107. BenAW says:

    Magic Turtle says:
    March 1, 2012 at 5:11 am

    “Applying it directly to the measured solar-power that is absorbed at the Earth’s surface yields the result that solar power alone can heat the planetary-surface only to a global mean temperature of about –18C. Applying it directly (but in the opposite direction) to the measured actual global mean surface temperature of about +15C yields the result that the Earth must be radiating power with a global mean intensity of about 390 W/sq.m.”

    The avg. 390 W/m^2 isn’t lost. In the end the atmosphere reduces the energy flow to 240W/m^2 at the TOA. The avg 240W/m^2 is lost to space, but compensated by avg. 240 W/m^2 incoming solar.
    System Earth isn’t cooling or warming if these figures are correct.
    The surface temp can easily be explained by the temp. of the oceans, ~275K, and higher at the surface, where the sun heats the upper layers.

    See https://tallbloke.wordpress.com/2012/02/23/ben-wouters-how-the-earths-surface-maintains-its-temperature/

  108. D J Cotton says:

    It is not so much a case of the S-B Law giving any wrong result as it is a case of misinterpreting that, when it gives the right result for, say, two plates radiating at each other, there is supposed to be two-way transfer of thermal energy. This is not the case because the cold to hot “way” would violate the Second Law of Thermodynamics. There is only one-way transfer from hot to cold represented by the area between the Planck curves for the two plates, because standing waves account for all the “transfer” from cold to hot and an equivalent amount from hot to cold..

    As you know, I deny that carbon dioxide “matters.” But I do consider it important to understand the physics which makes it very clear why it has totally insignificant warming effects, but more significant cooling effects due to absorbing solar-IR and sending it back to space.

    What it does not do is capture photons and fire them back at the surface where they crash land and warm us all up a bit. CO2 might seem to have the ability to do that much faster than water vapour, so, even though there’s less than 5% as much of it, it is supposed to have a comparable effect.

    Well as I have explained in other posts and on my site, the process is totally different. These so-called GH molecules are setting up standing waves with the surface, and WV can do so just as effectively as CO2. In fact, it can do it more effectively because it can radiate in a far wider frequency range, so it supports many more standing waves in all these extra frequencies. The end result may be that all the CO2 has less than 0.5% of the effect of all the WV, but don’t quote me on that because I have not done accurate calculations.

    But doing calculation like this is also pointless because, even though radiation from the atmosphere can slow the rate of cooling of the surface (but never warm it) the other processes – evaporative cooling, diffusion,, chemical processes – will start to cool at a faster rate. (Reasons for this are on my ‘Explanation’ page.)

    It is my hope that scientists who are against AGW will focus on the false physics in the IPCC “explanations” and models, rather than continually argue about temperature trends. Such trends are following a (roughly) 1,000 year roughly sinusoidal cycle which probably has another 0.4 to 0.9 degrees of rising to do before reaching a maximum in perhaps 150 to 250 years from now. In any data extending for over a century you should see a trend of about half a degree per century, but with a slight decline in the gradient as in the yellow line in the plot at the foot of my Home page http://climate-change-theory.com

  109. Magic Turtle says:

    Stephen Wilde (Today 9:56 am)

    “Everything that you say GHGs must be doing is in fact being done by the physics of the Ideal Gas Law.”

    Really? How?

    You seem to be talking nonsense to me. The Ideal Gas Law cannot make up the power shortfall at the surface created by the deficit of solar-input without the aid of greenhouse gases to absorb outgoing surface radiance first. I think you are disregarding the rigours of the 1st law of thermodynamics here. The non-GHG component of the atmosphere is not a source of power that modern physics can recognise. It can only serve to transmit the power that the atmosphere has first received from a source outside itself. The GHGs are what the greenhouse theory proposes as the means of enabling the atmosphere to receive the required extra power to begin with.

    Another scientific law that you appear to me to be ignoring is the Beer-Lambert law of optics, which deems that no matter how slight the downward radiance from the upper atmosphere and how high up it might be, some of it will get through to the surface and be absorbed, thereby constituting the ‘backradiation’ which you have decided does not exist.

    “You are facing both directions at the same time but do not realise it.”

    Really? What ambivalence is there in my position. I say that there is a decisive effectual difference between GHGs and non-GHGs (while you say there is not) and while my position is supported by the accepted laws of physics yours is not. That seems unequivocal to me.

    “The role of the IGL in fixing surface temperature for a planet with an atmosphere is settled science predating your radiative theory.”

    It is not ‘my’ radiative theory. I don’t own it, I wasn’t the first to conceive it and it became ‘settled science’ way back in the 19th and early 20th centuries. I am not interested in debating the historical precedents, but although the ‘Ideal Gas Law’ as you call it is ‘settled science’ your theory of planetary surface-heating by means of it, without the aid of GHGs to capture outgoing surface radiance, is not. On the contrary, it is your proposition that is the revolutionary one here, not mine. I’m just trying to represent the status quo.

    “The S-B Law is neither contradicted nor denied.”

    Excuse me? Wasn’t it you who wrote (Feb 29, 11:26 am) “Only apply the S-B Law from a point outside the atmosphere.”? You seem to be denying your denial of the S-B law to me. And who is it that has contradicted the results of applying the S-B law consistently inside the atmosphere by saying that they are invalid and proposing that the results of applying the Ideal Gas Law instead should be taken in their place? That sounds like doubletalk to me.

    “The IGL simply deals with a situation that the S-B Law was never designed to deal with.”

    That is complete rot, you know. I think you’ve got things the wrong way around here. The S-B law is not only more comprehensive than the IGL but it also more fundamental too. It is perfectly suited to the task of determining temperatures on the basis of radiative energy-flows and vice versa within the total climate-system, of which the atmosphere is just one component.

    “It is supplementary to the S-B Law for application only where there is an atmosphere.”

    No-one is denying the validity of the gas laws within the atmosphere. But the gas laws in themselves are not a power-source. And that is what is needed to satisfy the requirements of the 1st law of thermodynamics at the surface. And the gas laws cannot do that. At least, that is what the conventional scientific thinking is on the matter. But then, what would the conventional scientific community know about it? It only originated the gas laws in the first place!

  110. RKS says:

    Stephen Wilde says:
    March 1, 2012 at 9:56 am>>>>>>>>>>>>

    I think you might like to look at the current discussion on WUWT which classes people with your ‘anti consensus’ views as ‘Deniers’

    "Climate Deniers" Are Giving Us Skeptics a Bad Name

    No sensible explanation of where the extra 33K came from or how the Power generated at the surface [in terms of Wm^2] is 40% greater than Total Insolation.

    Somehow GHG’s ‘Generated it’ or acted as a magic generating blanket.

    It HAS to be GHG’s or your a “Denier” according to Mr. Singer and chums.

  111. RKS says:

    I know, I spelled you’re wrong again!!

  112. Stephen Wilde says:

    There is no additional power source. There is only a delay in transmission through the system which allows energy to build up at the surface.

    Your viewpoint is fatally flawed.

  113. RKS says:

    Stephen Wilde says:
    March 1, 2012 at 7:16 pm

    There is no additional power source. There is only a delay in transmission through the system which allows energy to build up at the surface.

    Your viewpoint is fatally flawed.>>>>>>>>>>>

    I was actually paraphrasing the opinions on WUWT, they were not my own.

    As with yourself, I believe the extra energy at the surface is due to kinetic energy in the troposphere as a result of atmospheric pressure.

    Sorry if I didn’t make myself clear.

  114. Magic Turtle says:

    Stephen Wilde (1st Mar. 7:16 pm):

    “There is no additional power source. There is only a delay in transmission through the system which allows energy to build up at the surface.”

    I am not concerned about a build up of energy at the surface. I am concerned about the build up of power there. This is what your theory does not explain (consistent with the 1st law of thermodynamics).

    “Your viewpoint is fatally flawed.”

    I don’t think so. My viewpoint accepts, accommodates and accords with all of the established laws of physics. Yours does not, specifically with respect to the S-B law, the 1st and 2nd laws of thermodynamics and the Beer-Lambert law. I think it is your viewpoint that is fatally flawed.

  115. D J Cotton says:

    Stephen is right in saying it’s not the Ideal Gas Law slowing the rate of cooling. No significant amount of thermal energy is ever going to transfer from the cooler atmosphere to the warmer surface in violation of the Second Law by any means, radiation or otherwise.

    Yet our schools “teach” young students (brainwash them in fact) that the greenhouse up there in the sky keeps us all warm like a big comfy blanket that mankind has made to be too thick.

    Young students don’t have the background in physics to be able to “think critically” about the issues and processes involved.

    Try giving your students a thought experiment like that below. Nothing less than this type of “thinking critically” is going to lead to valid conclusions in the climate debate. Not even the AGW proponents have been able to think critically enough like this example which, if you do think critically, will demonstrate why the greenhouse conjecture is false ….

    Consider a metal plate enclosed on one side with a “perfect” insulator. It is dangling out of a satellite and collecting the full blast of the Sun on its uninsulated side. Say it is between the Sun and Earth and its plane is perpendicular to the line between Sun and Earth. Assume it is not affected by radiation from the satellite.

    Say the Sun warms it to 330K at equilibrium. Now pull it into the shade of the satellite and turn it 90 degrees so it faces space and not the Earth or satellite. It will radiate virtually all the amount represented by all the area under its Planck curve, like a blackbody. So it “wants” to do this. You time how long it takes to cool to, say, 280K then to 200K

    Now repeat the experiment, but when it is in the shade of the satellite face it towards the Earth.

    We know the radiation from the Earth has the same flux (close enough) to that from the Sun which it receives. So will it stay at 330K because of this? I suggest the IPCC’s energy diagram concepts might “prove” that it would in fact do so, because they imply that the temperature would be a function of the number of photons received..

    No. It will cool more slowly than before, but how does it “know” to do so? It still “wants” to cool faster. The slower cooling cannot be caused by a transfer of thermal energy from the cooler Earth system. The only other way is for radiation from the Earth to interfere with the plate’s radiation, and this can only happen with a standing wave.

    It will take longer to cool down to 280K and it will stop cooling when in equilibrium with the Earth.

    But only some of the Earth’s radiation will have any effect. Its rate of cooling will be slowed down and gradually come to a halt at whatever is the weighted mean temperature of the Earth and atmosphere – perhaps around 255K.

    So at equilibrium the Earth’s radiation is split into two “sections” – some which is below its mean temperature and thus has no effect because it forms standing waves with the plate at 255K, and some above that 255K “cut-off” which is maintaining the plate’s temperature also at 255K.

    It was the UV, visible and maybe some near-IR in the Solar radiation that was able to heat it to 330K.

    So heat is not automatically transferred wherever “photons” roam. Heat cannot be transferred by radiation from a cooler atmosphere to a warmer surface. All that happens is that standing waves of radiation send a message to the warmer surface to slow down its rate of cooling each evening, so our evenings are more pleasant for our night life.

    But each molecule of carbon dioxide cannot play a greater role in producing standing waves than each molecule of water vapour which outnumbers it by at least 25:1 when water vapour is 1% of the atmosphere. So doubling carbon dioxide is like increasing water vapour from 1.00% to 1.04% of the atmosphere. Given that it can already get up around 4% without too much of a problem, let’s sleep comfortably in the not-much-warmer nights. Actually, carbon dioxide cools us more anyway by sending some of the Sun’s IR back to space.

  116. Ulric Lyons says:

    Stefan–Boltzmann could be valid if applied in context. Firstly, why not allow the albedo effect of clouds to be cancelled by the warming that night clouds give. The oceans are keeping a higher thermal equilibrium than a black body and should be largely responsible for keeping Earth well above the 5.85°C black body value, but as TSI is pretty flat, short term (e.g. ENSO) and long term variations in OHC and SST`s must be driven by the other heat input from the Sun, the solar wind.
    As the geomagnetic indexes follow long term temperature changes, and things like the AO, it makes good sense.
    The plamsa heat and the EMR compliment each other in a beautiful manner, as EMR heats the equator more, while the plasma heat works on the higher latitudes, as we see on Saturn:
    http://www.nature.com/nature/journal/v445/n7126/full/nature05518.html
    The link above says that the polar warming is accompanied by cooling at lower latitudes, we can see this type of effect happen at a Sudden Stratospheric Warming, with a synchronous cooling at the equatorial stratosphere.

  117. Great post. Here are some links to people who support what you are saying starting with one who has already commented:
    Harry Dale Huffman:- http://www.lulu.com/spotlight/hdhsciences
    Counting Cats in Zanzibar:- http://www.countingcats.com/?p=4745
    Nikolov & Zeller: http://wattsupwiththat.com/2011/12/29/unified-theory-of-climate/
    Robert G. Brown:- http://wattsupwiththat.com/2012/01/12/earths-baseline-black-body-model-a-damn-hard-problem/
    Leonard Weinstein:- http://scienceofdoom.com/2010/06/12/venusian-mysteries/#comment-2949
    Steve Goddard:- http://wattsupwiththat.com/2010/05/08/venus-envy/
    gallopingcamel:- http://scienceofdoom.com/2010/06/12/venusian-mysteries/#comment-2953

    I have plenty more but by now everyone gets my drift. Probably more intellectual horsepower here than the entire IPCC staff.

  118. Magic Turtle,
    You are getting your knickers in a twist because you have not understood Steven Wilde’s message.

    Steven is saying that surface temperature can be explained using Stephan-Boltzman, Newtonian gravitation and the gas laws. RTEs (Radiative Transfer Equations) can be neglected because in the troposphere convection and baroclynic eddies are the dominant heat transfer processes.

    The chemical composition of a planet’s atmosphere therefore has a very minor importance. The major factors are TSI (Total Solar Irradiance) and surface pressure. All the other factors such as gas composition, ocean currents and so on have effects that are of second or third order importance.

  119. BenAW says:

    Can someone explain to me why we are discussing mechanisms in which the atmosphere “warms” the earth, given that the total heat capacity of the WHOLE atmosphere is equal to ~3,2 METER of ocean water?

  120. jjthom says:

    Stephen Wilde.
    You seem unwillingh to criticise my model of filters and mirrors (which is just about the same as Willis’s steel greenhouse.) I would be very grateful if you would pull apart the model!

    I assume that you subscribe to the N&Z theory:
    Equation (7) allows us to derive a simple yet robust formula for predicting a planet’s mean surface temperature as a function of only two variables – TOA solar irradiance and mean atmospheric surface pressure,

    This seems to suggest that albedo has no effect (they take no account of albedo with equation 8)?

    If one assumes a GHG free atmosphere around a black globe (absorbs 100% of TSI) You can see an equilibrium where TSI in eventually equals LWIR out and the black earth reaches an equilibrium temperature required to do this. The non-GHG will absorb by conduction heat from the surface and will warm to that temperature (or higher if your theory is correct)

    If one then assumes exactly the same system but with a highly polished globe which reflects 100% of the TSI i.e. it is not absorbing energy from the TSI. so if at 0K or 100K it is not receiving energy. Anything above 0K and it will attempt to radiate until it reaches 0K, however it is a 100% reflective globe (surface emissivity=0) so this will not escape. there is no received energy entering the silvered body. The non-GHG atymosphere similarly will not be heated by the TSI. There will be no energy into the atmosphere other than from conduction from the globe.
    The globe and the atmosphere cannot absorb any of the TSI. TSI is the only source of energy. How can pressure heat the non-GHG and heat the globe and itself. Where is the energy coming from to do this. It is not from TSI.
    So in this case we have total energy into system=TSI
    Totally reflected energt out of system = TSI
    Total LW IR out of system =0
    The system is in balance and if one assume an emissivity of 0 then whatever the initial temperature of the system will never change (energy received=energy reflected at all temperatures). The gas (also not receiving any energy input other than by conduction with the globe) will also stay at the initial temperature of the system. You seem to suggest that the gas will still warm the globe despite receiving no energy from outside.

    wiki:
    A true black body would have an ε = 1 while any real object would have ε < 1. Emissivity is a dimensionless quantity.In general, the duller and blacker a material is, the closer its emissivity is to 1. The more reflective a material is, the lower its emissivity. Highly polished silver has an emissivity of about 0.02.

    JJT – the emissivity of materials has to be taken into account when using thermal imaging – A reflective body will show as cold (actually it usually reflects the surroundings temperature back at the camera)

  121. tchannon says:

    Ford, the emissivity is regardless of lamda.

    I don’t think you are going to get anywhere. Notice that in reality what is actual fits pressure very well but exactly what is going on seem to be moot. What is needed is science, a controlled experiment. Taht is a good place to apply inspiration, if it can be done at all.

    TSI (which is not known accurately) is always same out as in, any system unless it is in transient. This poses an interesting balance problem rather like a compressive heat pump where the working fluid adjusts to conditions, odd stuff. Maybe this gives a clue.

    I’m too busy and tired (my English and spelling will be poor) to take part or even read most of this stuff, other than try and keep an eye on things, sorry.

  122. Chris M says:

    That 390 W/m^2 is radiated from the earth’s surface seems to be taken as a given, derived from the SB law and the lower tropospheric average temperature of 15C. But has this level of LWIR actually been measured? What about the fact that the atmosphere is constantly in contact with land and ocean surfaces warmed by the sun?

    “In conduction, the heat flows through the body itself, as opposed to its transfer by the bulk motion of the matter as in convection, and by thermal radiation. In solids, it is due to the combination of vibrations of the molecules in a lattice or phonons with the energy transported by free electrons. In gases and liquids, conduction is due to the collisions and diffusion of the molecules during their random motion.”

    http://en.wikipedia.org/wiki/Conduction_%28heat%29

  123. Magic Turtle says:

    I think all of the people here who are proposing alternatives to the standard greenhouse theory and telling me that I don’t understand them need to ask themselves whether they understand the standard greenhouse theory that they are wanting to overturn and replace, because as far as I can see none of them do.

    Before you can overturn an established scientific theory you first need to show that there is something wrong with it. No-one has done that yet (although I appreciate that some people here think they have).

    And before you can replace an established theory with your theory you first need to show why your theory is better than the old one and any others that are being proposed at the same time. No-one has done that yet either.

    Instead what they seem all to have done on this blog is to try to do everything all at once and simply call attention to their own alternative theories in a general free-for-all competition. So far I have been pressed to consider Claes Johnson’s theory, Harry Dale Huffman’s theory and Doug Cotton’s theory plus their links to Nahle’s theory etc., as well as Stephen Wilde’s theory, all at the same time. I cannot handle all these demands upon my attention. One theory at a time, please!

    Since this particular blog is about Stephen Wilde’s alternative to the standard greenhouse theory, I am concentrating on that to the exclusion of all others. Sorry you other theorists, but this blog-post is his and I think it’s his turn to get my full attention.

    Before trying to assess the merits of Stephen’s theory to see whether it is better than the standard greenhouse theory though, there is still that first stage to go through of determining whether there is anything wrong with the standard theory. I’m not sure what flaws Stephen may still think he has found in it. He has argued against ‘backradiation’ – an accepted essential element of the standard model – on the grounds that it couldn’t work because radiation emanating from the upper atmosphere would all be reabsorbed by the atmosphere before it could reach the surface to warm it. But that objection was answered on grounds of a) the Beer-Lambert law of optics, which deems that some radiance from even the very top of the atmosphere must get through directly to the surface; and b) the cascade effect arising from radiance from a high altitude being in the direction of the surface, then absorbed by the atmosphere and re-emitted again in the same direction. So far I have not seen Stephen attempt to counter those points.

    Other people have also offered objections to the standard theory on the grounds that surface warming by backradiation somehow violates the 2nd law of thermodynamics. This objection hasn’t been answered yet so let me try to answer it now.

    The 2nd law of thermodynamics implies that heat cannot flow spontaneously from a cooler body to a warmer one. But note the operative term ‘heat’. Heat is one form of energy, but there are other forms of energy too and radiation is one of them. Those are not covered by this implication of the 2nd law and so they can flow spontaneously from a cooler body to a warmer one under the right circumstances. Do we have ‘the right circumstances’ here?

    I think we do in fact because the fundamental constraint of the 2nd law that all energy-transactions in a system will cause the entropy of the system to tend towards infinity, only applies to thermodynamically-closed systems. But the Earth is a thermodynamically-open system instead, since it is receiving energy continuously from the Sun and discharging it again to space. The entropy of an open system would also tend towards infinity if it was not for the continuous supply of negentropy from a source – in this case, the Sun’s radiation. Under these conditions, the open system’s overall entropy is kept in a stable state of dynamic equilibrium by the competing rates of spontaneous entropy-increase (from energetic-transactions taking place inside the system) and the inflow of negentropy from the Sun. (These quantities were actually calculated for the Earth as far back as the 1960s.)

    So the Earth-system is not constrained so tightly by the rigours of the 2nd law as those people here who have been objecting to the standard greenhouse theory on 2nd law grounds have generally assumed. The ‘coffee-flask’ analogy that they are fond of citing is an excellent analogy for a closed system in my opinion, but it is a false and flawed analogy for an open one like the Earth! Sorry guys, but the radiation-paradoxes that you’ve rightly perceived in the various closed-system thought-experiments that you’ve been conducting will not necessarily arise in the open global climate-system of the Earth. And if they won’t necessarily arise, we cannot assume that they will. So this objection doesn’t stand up and I think you theorists need to re-think your thought-experiments in order to visualise the energy-flows correctly.

    According to the established physics, it is true that within a closed system a cooler body cannot warm a warmer one even further. But in an open system it can by means of radiation. In order to see how it can do that, imagine first a planet like the Earth whose atmosphere is totally transparent on all wavelengths because it has no GHGs in it than could absorb radiation. In that case, all of the Sun’s radiation that entered the atmosphere would pass straight through the atmosphere and strike the surface, where whatever amount of it was not reflected would be absorbed. The total amount of absorbed insolation would set the global mean temperature of the planet. Let’s say this turned out to be +6ºC for argument’s sake.

    The atmosphere would not be able to absorb any of the in-coming solar radiation, nor any of the outgoing surface radiation directly, so its only possible way of warming would be by conductive heating from the surface. But because of the constraints of the 2nd law, which apply to all conductive heat-flows in open and closed systems alike, the atmosphere cannot be warmed above the temperature of the surface by this means, so we can say: temperature of atmosphere < 6ºC.

    Now let us imagine that some greenhouse gases are added to the atmosphere and we can see that the situation changes dramatically. The GHGs are able to absorb IR-radiation emanating from the planet’s surface and from the incoming insolation directly. This absorption of power by the GHGs causes them to start radiating with an intensity that is generally greater than that of the ambient atmosphere in which they are contained and some of that extra radiance is directed towards the planet’s surface as backradiation. Furthermore, whatever extra power the GHG’s have absorbed and not re-emitted as radiation is translated into kinetic energy and shared with the rest of the atmosphere by intermolecular-collisions, or ‘conduction’. This raises the mean temperature of the atmosphere and causes it to radiate more intensely than it otherwise would, thereby producing more back-radiation that warms the surface.

    Now before the system has settled back into thermodynamic equilibrium after the GHGs were added to the atmosphere, this initial warming from back-radiation may be very slight, but it boosts the surface temperature by a finite amount. This slight surface temperature increase raises the intensity with which the surface is radiating and so the GHGs in the atmosphere have more outgoing radiance to absorb and then they get warmer too, leading to more back-radiation which raises the surface temperature even higher, and so on in progressive cycles. However, because of the S-B law relating temperature to radiant intensity, the temperature increments at the surface get progressively smaller (instead of larger) with each cycle in a convergent series that all adds up eventually to a substantial, but finite surface temperature increase when equilibrium is reached. Consequently the new surface temperature at equilibrium may be well above 6ºC. No violations of the 2nd law are apparent here.

    So as far as I can see no-one has raised any significant scientific objections to the standard greenhouse theory to date. In that case, following the pragmatist’s dictum “If it ain’t broke, don’t fix it,” there seems to be no great need for a replacement theory to explain planetary surface temperatures being generally elevated above the levels set by solar insolation alone. There would only be a point in replacing it if the alternative theory on offer was significantly better than the existing one in some way. So is Stephen’s alternative theory significantly better than the standard greenhouse theory? This is what we need to determine next.

    The purpose of a scientific theory is to explain something in rational, cause-and-effect terms. Firstly, this implies that causes, effects and the interrelationships between them must be made clear in the theory. Secondly, the theory must be consistent with the complete corpus of existing theories that have been established as standard scientific theories, such as the laws of thermodynamics for example. (This does not mean that the established theories are set in stone of course; it just means that the same assessment-process should be applied to any alternative theories that challenge them as is being applied in the present case.) And thirdly, the theory should fit the facts of the real-world situation to which it refers. Let us compare Stephen’s theory against the standard greenhouse theory using these criteria.

    Firstly, to me the standard greenhouse theory is much clearer and more rational than Stephen’s. With the standard theory I can see what is causing what to happen in a rational, orderly way. To be precise, I can see what is causing the mean surface temperatures of planets to be hotter than they otherwise would be if they were being warmed by solar irradiance alone. With Stephen’s theory I can’t do that. I cannot see the causality of atmospheric heating of the surface by the gas laws. Indeed, I cannot see the first link in the chain of Stephen’s logic that leads him to this conclusion. How can the gas laws heat anything? They possess no energy, or power to heat anything with. Where is the power that warms the surface above the absorbed-insolation baseline coming from originally in his theory? It cannot be coming from solar insolation because that has already been used up in getting to the baseline temperature and there isn’t any left over to do any more heating with. This is an insoluble conundrum for me.

    Secondly, whereas the standard greenhouse theory seems perfectly consistent with the existing corpus of established physical theories to me, Stephen’s theory seems to be at loggerheads with a few of them. The S-B law that he says does not apply to the situation inside the atmosphere and shouldn’t be used, is just one example of the apparent arbitrariness of his theory. The S-B law is a universal law that physics says can be applied to all matter in all places, but Stephen’s theory prohibits its use inside the Earth’s atmosphere. Why? Another is his theory’s seeming denial of the 1st law of thermodynamics as applying to the general power-balance at the surface. According to his theory, there is no power-anomaly of about 153 W/sq.m at the surface to be explained, although the 1st law of thermodynamics and the S-B law insist between them that there is one. Then we have his theory’s ignoring of the Beer-Lambert law which says that back-radiation will get through to the surface in some finite quantity wherever it starts from in the atmosphere, but Stephen’s theory says No to that for some reason which I cannot fathom. If I had to find one word to describe how his theory appears to me it would be ‘whimsical’.

    Finally, how well do the two theories fit the facts of the case? Actually it is not possible to carry out this comparison fully because neither theory has been refined sufficiently to the point where it is capable of generating precise predictions that can be checked with real-world observations. However, the essential elements of the greenhouse theory have been verified empirically, of course. We have known since the 19th century that the Earth emits radiant energy on infrared wavelengths and that certain gases like water-vapour and CO2 are strong absorbers of radiation in their respective IR-absorption wavebands. We have known since the early 20th century that the Earth’s global mean surface temperature is elevated significantly above the temperature that incident solar radiation alone could support. So although the theory as a whole has not been empirically verified the basic ingredients of it have, which is a promising start to my mind.

    However, Stephen’s theory does not seem to have even that much empirical support for it as far as I can see. I have never heard of any experiment or observation which demonstrated that material bodies which are warmed solely by radiation can, through purely conductive heat interactions at their interfaces, attain temperatures above those determined by their radiant power-inputs alone. Such an empirical finding seems highly improbable to me anyway, since it would overthrow the 2nd law of thermodynamics at a stroke. Also, I have never heard of any decisive empirical observation of an atmosphere producing warming solely by its static pressure. That also seems highly improbable to me too, since it would overthrow the 1st law of thermodynamics and the combined gas law (what Stephen calls the ‘Ideal Gas Law’) at a single stroke as well.

    Altogether it seems to me that the standard greenhouse theory wins hands down on all counts. Sorry, but it does. It’s up to each person to decide for themselves which theory they prefer and/or choose to believe in of course. But if you want to decide on scientific grounds, I think you will need to apply an analysis that’s very similar, if not identical to the one I have used.

  124. RKS says:

    Magic Turtle says:
    March 3, 2012 at 12:20 am>>>>>>>>>>>>>

    TSI is accepted to be 240Wm^2 mean over the 24 hour cycle, adding energy to the globe at the rate of 240 Joule.sec.m^2 over 24 hours.

    TSI in = TSI out.

    Over the same time period the Earth’s surface is accepted to be dissipating power at the rate of 390Wm^2 – 390 joule.sec.m^2.

    I have a closed electrical circuit, which is subject to the same physical laws as the atmosphere where energy is concerned, which draws a current of 1 amp from a 240V dc supply – 240W.

    I can even include switched capacitors to simulate absorption and emission in GHG’s.

    If you can suggest any way of dissipating 390W within that closed circuit, without the addition of a secondary source of energy, you could well be on the way to a Nobel prize.

    Of course, Nikolov and Zeller offer a perfect way to account for the increased energy level at the surface, but somehow I don’t think that fits in with your views on the subject.

  125. tchannon says:

    MT,
    “Before you can overturn an established scientific theory you first need to show that there is something wrong with it.”

    No, such as the classic; two explanations for the same thing void it.

    Established means nothing unless that means experimentally confirmed, independent, etc. in this case there is nothing, unless you know otherwise.

    I agree about peripatetic. This is partly our fault (a mod writes), although the nature of blog software gives little latitude. I’ve considered various ways of focusing, yet long experience to before the public internet says that is improbable. Are laws and lores, Pot Lid chap for example deals with 80:20 and similar, the natural ways many things pan out. Human attention is in there too. Books over there describing a raft of heuristic laws, tend to be great fun.

    “Heat is one form of energy, but there are other forms of energy too and radiation is one of them.”

    Not sure about that. Radiation is energy flow. Heat is not energy, it is expressed energy, ie. power. Radiation is not a flow of power. It expresses as power if impeded in it’s flow.
    Maybe another way, radiation is flowing at the speed of light (yada yada), can’t stop it or store it.
    Energy can be sucked up by mass and it is stored, depending on properties of matter.
    Maybe that is cross purposes.

    “…the atmosphere cannot be warmed above the temperature of the surface by this means, so we can say: temperature of atmosphere < 6ºC."

    I've already explained how that can happen, is actually somewhat complex. I'm not sure many grocked. (you are not allowed a radiative profile because it cannot exist)

    At that point you are already way out on a limb, my brain refuses.

    I have problems with ght but that is another matter. Back and forward radiation are not an issue but the details and omits are a different matter. I also object to any usage of SB, is a domain change relying on unknowables. You have a theoretic case but not an empirical case. Gets into unknowable flux. This leaves us able to guess and approximate. Tricky when the data is far worse than claimed. (gets too close to my expertise)

    Got other things to do. Try and remember to come back. (get no notifications of anything, have to wade through random order stuff)

  126. Chris M says:

    For my own benefit, and hopefully others’ as well, I have converted Nasa’s somewhat tricksy energy balance figures to (to me) a more readily understandable form. A version of Trenberth’s diagram is brought in at the end, without much explanation. The alleged 390 W/m^2 seems to have something to do with CERES satellite data.

    http://earthobservatory.nasa.gov/Features/EnergyBalance/page2.php

    So here goes:

    Ignoring reflected sunlight, 29% of 340 W/m^2, gives net absorbed 71% = 241W/m^2.

    Of the absorbed portion:
    32% is absorbed in the atmosphere
    68% is absorbed at the surface

    The proportions of outgoing surface energy are via (approximately):

    Evaporation 0.52
    Convection 0.13
    OLWR 0.35

    (Nearly one-third of surface OLWR is reabsorbed in the atmosphere, leaving over two-thirds to escape via the atmospheric window.)

    Which means that 83% of 241W/m^2 returns to space indirectly via the atmosphere. This includes the original 32% of net incoming SWR. (Note that the 59% OLWR from the atmosphere in the NASA diagram is 59/71 net absorbed SWR = 83% of outgoing, the remainder escaping directly through the atmospheric window.)

    Make of that what you will, but it seems to me that the ground, ocean and atmosphere combine effectively to delay the release of energy to space, by acting as heat reservoirs, hence no need for back radiation.

  127. BenAW says:

    Magic Turtle says:
    March 3, 2012 at 12:20 am

    “Before you can overturn an established scientific theory you first need to show that there is something wrong with it. No-one has done that yet (although I appreciate that some people here think they have).”

    The very base of the GHT is the treatment of earth as a blackbody (BB). This is also it’s fatal flaw imo. If you “switch off” the radiation to a BB in space, it’s temp. IMMEDIATELY goes to 0K.(or 3K if you want). This behaviour is more or less appearent on the moon, which has no atmosphere and no oceans. So the moon is a “reasonable imitation” of a BB.
    Not so planet earth, it has 1,3 billion km^3 of water, with a temp. that is already at least 20K ABOVE the temp the sun is supposedly able to achieve on planet earth. During nighttime the oceans do not cool to 0K, actually even their surface temp remains more or less steady to within ~1K.

    Only relevant thing about radiation is how much solar is “absorbed” by system earth (~70% after reflection) and how much leaves system earth at the top of the atmosphere. If the two are more or less equal, system earth is neither cooling nor warming much.

    So the sun is just barely able to prevent further cooling of the earth.
    The fact that we “recently” started to experience ice ages, means the earth was still cooling from higher temps before that.

    If you accept backradiation as real, then backconduction should also be possible.
    Given that the atmosphere has the heatcapacity of ~3,2 meter of ocean, lets replace the atmosphere with 3,2 meter of “magic water”, with an average temp of lets say 280K.
    (this is on the high side imo)
    How would this magic water warm the earth from the BB 255K to the current 288K by backconduction?

  128. mkelly says:

    Mr. Turtle please get your understanding of open and closed systems right before pontificating. Both systems allow energy to cross the boundary. The difference is open also allows mass to cross the boundary.

  129. Stephen Wilde says:

    “Make of that what you will, but it seems to me that the ground, ocean and atmosphere combine effectively to delay the release of energy to space, by acting as heat reservoirs, hence no need for back radiation.”

    It seems that way for me too.

    The Ideal Gas Law provides the relevant equations via PV=nRT without any need to include a term for radiation at all.

    If radiation were relevant then the absence of a term for it would render the Ideal Gas Law invalid.

    The upshot is that surface temperature for a planet with a gravitational field is derived from insolation and atmospheric mass alone.It is the mass of the entire atmosphere, not just GHGs, which gives the pressure and molecular density at the surface to fix the surface temperature at a given level of solar input.

    GHGs can affect the amount of energy whizzing about in the air but not the surface temperature.The expansion of the atmosphere from more GHGs reduces molecular density at the surface without changing surface pressure so there are less molecules per unit of volume at the surface to interact with insolation hence a cooling of the surface exactly proportionate to the warming of the air.

    Furthermore the system response is instant. More energy in the atmosphere causes immediate expansion.

    The effect of more energy in the air from GHGs (when there is no change in pressure or insolation) is simply an increase in the volume of the atmosphere with no change in surface temperature but instead an air circulation change. That circulation change alters the rate at which energy is transferred to space so that the slowing down effect of energy transfer by GHGs is negated by a faster rate of energy transfer from the expanded atmosphere.

  130. Stephen Wilde says:

    “Also, I have never heard of any decisive empirical observation of an atmosphere producing warming solely by its static pressure.”

    MT, you produced a huge amount of verbiage based on a simple failure to understand what was well established long ago and which led to the Ideal Gas Law in the first place. You are not alone, that ignorance is rife.

    The point that I will now reiterate has been set out many times here but it still does not sink in for you.

    The atmosphere does NOT produce warming from static pressure.

    It will only produce warming when irradiated by an energy source.No energy source and the atmospheric molecules will freeze to the surface.

    Gravity places most molecules and maximum density at the surface. Incoming solar energy gets more molecules to interact with for each unit of volume at the surface where that density is higher.

    The more molecules per unit volume the higher the temperature and the less molecules per unit volume the lower the temperature.

    Thus there is a temperature gradient from surface upward which is entirely a result of the density gradient but only for so long as energy is being supplied.

    That is what the Ideal Gas Law defines and quantifies and it provides a specific rate of temperature decline with height that is dependent on pressure and insolation alone for any planet with a given strength of gravitational field.

    There is no magic production of additional energy from anywhere.

    Now if one introduces GHGs (or any other compositional changes) they do not affect the pressure or density at the surface or the total amount of incoming energy available so they cannot affect surface temperature.

    Instead they add more energy to the air but because of the instant expansion of the air the molecular density at the surface declines and there are then less molecvules per unit area at the surface to react with incoming solar so the surface cools.

    The net effect at the surface is zero but there is a fraction more circulatory action in the air.

    But that change in the air from more GHGs is magnitudes smaller than natural circulatory changes caused by variations from sun and oceans.

    So, compositional changes in the atmosphere will only ever alter the volume of the atmosphere and never the surface temperature.

    The circulatory changes that result will always cause a reconfiguration of the ACTUAL lapse rate so as to ensure that at the top of the atmosphere energy in will equal energy out as predicted from the THEORETICAL lapse rate derived from pressure and insolation alonre.

    If the reconfiguration of the ACTUAL lapse rates (often multiple such lapse rates at different levels according to the varying composition of the vertical column) were ever to fail to net out to match that THEORETICAL lapse rate the planet would lose its atmosphere either from it boiling off to space or freezing to the surface.

  131. Magic Turtle says:

    RKS (Today 1:35 am)

    “TSI in = TSI out.”

    Yes, but that is the basic power-balance equation that applies at the top of the atmosphere (TOA) when the planet is in thermal equilibrium with its environment. At the planet’s surface another power-balance is struck (at equilibrium again) and this time it is simply:

    Total power In = Total power Out.

    But that’s only if the 1st law of thermodynamics applies, of course.

    “If you can suggest any way of dissipating 390W within that closed circuit, without the addition of a secondary source of energy, you could well be on the way to a Nobel prize.”

    Ah, if only we puny humans could do, on our microbial level with our technology, all the things that nature is able to do on planetary, solar, galactic and universal scales with her technology… Just because we cannot, for example, create a self-sustaining thermonuclear reaction it doesn’t mean that nature can’t do it either, as she appears to be doing already inside the hearts of stars. And what about consciousness – how is nature able to produce that? Likewise, just because we cannot do power recycling in electrical circuits, it doesn’t mean that nature can’t recycle power either, as she appears to be doing on a planetary scale with GHGs. But if we can learn from nature how she is doing it, the day may come when we can do it in electrical circuits too.

    But I think your electrical analogy is a false one here. In greenhouse theory, the power that the GHGs absorb and the atmosphere recycles back to the surface is not subtracted from the incoming power-supply from primary insolation. The GHGs enable the atmosphere to capture what would otherwise be waste-power and recover it for use at the surface. If we could find a way of mimicking this waste-power recycling process in an electrical circuit, perhaps we could power our industries and our whole civilization far more efficiently than we presently can. I know of no theoretical reason why we should not be able to recycle the waste IR-radiation from electrical circuits if we want to, or have need to.

    But I am not an electronics expert. If you are, that Nobel Prize could be yours for the taking.

    “Of course, Nikolov and Zeller offer a perfect way to account for the increased energy level at the surface, but somehow I don’t think that fits in with your views on the subject.”

    I am not attached to my existing views on the subject. If you think these people have a superior argument, why don’t you put it to me and let me decide whether I agree with it or not?

  132. RKS says:

    Magic Turtle says:
    March 3, 2012 at 6:54 pm……

    I am not attached to my existing views on the subject. If you think these people have a superior argument, why don’t you put it to me and let me decide whether I agree with it or not?>>>>>

    Link below:-

    Click to access unified_theory_of_climate_poster_nikolov_zeller.pdf

  133. Magic Turtle says:

    tchannon (Today 1:41 am)

    I hope that by the time you read this you will have had a chance to rest up and are feeling recovered. I sympathise with you in the overstretched condition of which you complain. In view of that, I am especially grateful to you for taking the trouble to reply to my long post. It was beyond the call of duty and I appreciate it. I shall try to be brief in this one!

    “ ‘Before you can overturn an established scientific theory you first need to show that there is something wrong with it.’

    No, such as the classic; two explanations for the same thing void it.”

    Actually, I don’t think they do. Occam’s Razor applies.

    “Established means nothing unless that means experimentally confirmed, independent, etc. in this case there is nothing, unless you know otherwise.”

    I meant “established” in the sense of “accepted as standard provisionally by the scientific community”. The term ‘provisionally’ is crucial there. It doesn’t mean that an ‘established’ theory has finally been positively confirmed by experiment and proven beyond all doubt; it just means that it hasn’t been refuted, or ‘falsified’ so far. I think of it as being a practical convention in science that enables scientists to distinguish between completely novel theories and those which have been considered to some degree already. Without having some such novelty-filter like this in place the scientific community would be at constant risk of being overwhelmed by purportedly ‘new’ theories that were really just rehashes of old theories, perhaps in new guises.

    “ ‘…the atmosphere cannot be warmed above the temperature of the surface by this means, so we can say: temperature of atmosphere < 6ºC.’

    I've already explained how that can happen, is actually somewhat complex…”

    Sorry, but I missed seeing your explanation of that somehow. Could you point me to it please? Anyway, I feel I can only point you to the 2nd law of thermodynamics which forbids net conductive heat-transfer in opposition to the temperature gradient.

    “ ‘Heat is one form of energy, but there are other forms of energy too and radiation is one of them.’

    Not sure about that. Radiation is energy flow. Heat is not energy, it is expressed energy, ie. power. Radiation is not a flow of power. It expresses as power if impeded in it’s flow.”

    The fine distinctions between all these technical terms are confusing, especially to non-specialists and I was trying to speak to them as well as to those readers who may already know their thermodynamic onions. To clarify for aficionados like yourself though, by ‘heat’ I really meant ‘heat content’, or that amount of energy which a body contains by virtue of its absolute temperature. (I was trying to avoid using ‘enthalpy’.)

    According to my ‘Dictionary of Physics’, ‘power’ is ‘The rate at which energy is expended or work is done’. Unless someone wants to consider the work being done in this situation, only the former definition applies and we can treat power simply as being the rate of energy-flow.

    “I also object to any usage of SB, is a domain change relying on unknowables.”

    Then your mind is labouring under an unnecessary artificial handicap. I really don’t understand why there is this taboo against the S-B law on this blog. The whole of modern civilization has been using it to good effect and thermodynamicists have been considering it an indispensible tool of their trade for more than 100 years right down here on the Earth’s surface from where you guys seem to want to ban it! Industry uses it daily to ascertain high gas-temperatures which would otherwise melt normal thermometers (although I suppose it’s only the Chinese who do that nowadays!). What’s the problem with it?

    I don’t know what you mean by ‘is a domain change’ but it is not ‘relying on unknowables’ any more than any other mathematical law of physics does. It is perfectly applicable to the case in hand and there is no honest reason that I can see for not using it.

    “You have a theoretic case but not an empirical case. Gets into unknowable flux. This leaves us able to guess and approximate. Tricky when the data is far worse than claimed.”

    Yes, I quite agree. I think it is the vast amount of ‘wiggle-room’ that exists over the empirical implications of the theory that has enabled the man-made climate change lobby to build their alarmist political cult upon it. However, just because they have hijacked the theory and are making false claims for it, that doesn’t mean that the theory itself is necessarily false. I think the present scientific status of the theory is that it remains unfalsified on both empirical and theoretical grounds.

  134. Stephen Wilde says:

    MT said

    “Industry uses it (the S-B Law) daily to ascertain high gas-temperatures which would otherwise melt normal thermometers”

    Now I see why you took objection to me saying that it doesn’t apply within an atmosphere.

    What I meant (obviously) is that it doesn’t apply within an atmosphere for the pupose of calculating the surface temperature of the planet.

    It would help if you think about the context of any particular assertion before objecting to it.

  135. tchannon says:

    MT,
    Sorry about leaking.

    We probably agree on most of the what is accepted, it’s more about my not accepting what many others do, likely from long experience, perhaps from knowledge that all is not well.

    Abnormal temperature.
    I went looking for asymmetric atmospheric temperature processes, it is the asymmetry which is critical. This is the last instance
    Conceptually dayside surface heating mixes into the atmosphere mechanically, nightside the process does not reverse with the effect of leaving the atmosphere hotter than expected. There is a good deal of evidence this happens to some degree.
    Reality is more complex but there is a means. I still think there is much more.

    Most of the warmists have argued a linear process, perhaps without the actual effect of gravity. Sure, fine, except now stick around and actually consider the rest, they run off.

    I wrestle with energy… and energy usually wins. (nicely ambiguous)
    Expert on that stuff, no, too many tricky situations. There again I’ve had to deal with practical stuff.
    I ought to have said latent energy, arriving from flux dissipating power.

    Very often a simplified SB is used. I am pointing out this is unsafe in real situations. We need to know the temperature of the real radiating item, a surface not the insides. This tangles with needing to know the exact surface properties. All we can get is an approximation where there are people arguing about balances of 0.1% Worries me.

    Temperature domain.
    Flux domain.

    Earth at xC mixed with flux y, ought to be T vs T, F vs. F. One is a proxy for the other via SB.
    Why is this done? ‘cos they can’t measure, as you say a plasma at 10kK kind of melts things, similarly it’s difficult pointing a flux sensor at the ground at ground level over a huge area, so the guessed T of the body is used.

    That is the kind of thing I am getting at, probably poorly put.

    “… that doesn’t mean that the theory itself is necessarily false. I think the present scientific status of the theory is that it remains unfalsified on both empirical and theoretical grounds.”

    Yep except I don’t see a complete theory and it is untestable.
    Point out a problem, all I get are hissy fits back about denial of what is social belief. No-one has yet shown me a satisfactory explanation, always jumps, always hand waving, always omissions and usually so poorly done it would take a book to pick it apart. I tend to stay silent.

    N&Z? Don’t know but pressure in a broad sense explains much of what we see, radiative doesn’t without special cases. How exactly this works seems unclear. I want this stuff the get a good hearing without fear, good talk about it, explore. Doesn’t matter if there are mistakes and if just one good thing appears, great.

    I’m writing too much.

  136. Magic Turtle says:

    mkelly (3rd Mar. 12:51 pm)

    “Mr. Turtle please get your understanding of open and closed systems right before pontificating. Both systems allow energy to cross the boundary. The difference is open also allows mass to cross the boundary.”

    Mr Kelly, please get your eyesight fixed before pontificating to me. I specifically referred to ‘thermodynamically-open’ and ‘thermodynamically-closed’ systems, for which the question of the transfer of mass across system-boundaries does not arise and is completely irrelevant.

  137. BenAW says:

    tchannon says:
    March 4, 2012 at 12:55 am

    Abnormal temperature.
    I went looking for asymmetric atmospheric temperature processes, it is the asymmetry which is critical. This is the last instance
    Conceptually dayside surface heating mixes into the atmosphere mechanically, nightside the process does not reverse with the effect of leaving the atmosphere hotter than expected. There is a good deal of evidence this happens to some degree.

    Tim, did you actually think through my simple little theory?
    The OCEANS provide this stability, see http://www.terrapub.co.jp/journals/JO/pdf/6305/63050721.pdf
    It explains why the SURFACE temps of the oceans only vary ~1K between day and night.
    It’s the continents and islands that heat up and cool down during the day/night cycle, and thus create most of the weather.
    On an oceans only planet the weather would be pretty dull.

    Forget about the atmosphere for a while. It’s heat capacity is equal to only 3,2 METER of ocean.
    The base temp of the oceans is 275-277K, leftover from much higher temps in ancient history.
    The sun keeps a very shallow layer warm (prevents further cooling) and this layer warms the atmosphere.

    The atmosphere slows the energy flow from ocean to space somewhat, so at TOA radiation IN = radiation OUT.

  138. PaulK says:

    Stephen: “I don’t see that the gravity induced lapse rate for a theoretical ststic atmosphere is affected by composition. Can you provide evidence that it is ?”

    The formula for the dry adiabatic lapse rate is -dT/dz = g/c_p. Because gases have different c_p, the c_p of air changes with composition.

    Stephen: “According to the Ideal Gas Law the surface temperature is in part dependent on gas density at the surface which is itself a function of molecules per unit of volume.

    The decline in density will be directly related to the amount of expansion so that the thermal effect of the two changes(density and volume) cancels out.
    … and …
    Now if one introduces GHGs (or any other compositional changes) they do not affect the pressure or density at the surface or the total amount of incoming energy available so they cannot affect surface temperature.

    Instead they add more energy to the air but because of the instant expansion of the air the molecular density at the surface declines and there are then less molecvules per unit area at the surface to react with incoming solar so the surface cools.

    The net effect at the surface is zero but there is a fraction more circulatory action in the air.

    But that change in the air from more GHGs is magnitudes smaller than natural circulatory changes caused by variations from sun and oceans.

    So, compositional changes in the atmosphere will only ever alter the volume of the atmosphere and never the surface temperature.

    The circulatory changes that result will always cause a reconfiguration of the ACTUAL lapse rate so as to ensure that at the top of the atmosphere energy in will equal energy out as predicted from the THEORETICAL lapse rate derived from pressure and insolation alonre.”

    It is widely accepted that the doubling of CO2 in the troposhere increases the surface temperature 1 C. In “preindustrial” times there were 280 ppm of CO2 in the air. It’s so small amout that the composition of the air doesn’t change in practise. Only the absorbtion and emission of the air in a narrow range near 15 μm increase which is the cause of the 1 C temperature rise.

    The Ideal Gas Law PV = nRT gives the relative expansion of the air V1/V0 = 1 + ΔT/T0 when pressure is the same. At the surface T0 = 288 K and V1/V0 = 1.0035 and the relative density change is ρ1/ρ0 = 0.9965.

    But you say that the temperature at the surface doesn’t actually change, because the real lapse rate changes due to increased circulation in the air. How much must the lapse rate change? The average lapse rate in the troposphere is 6.5 C/km. The effective back body temperature of the out radiation is 255 K. So the effective radiating height is 5.077 km. If we use it, the lapse rate that keeps the surface temperature constant is 6.697 C/km. That is a 3% increase.

    There is a hole in your theory. Why should the surface temperature stay constant? The standard GHE theory can at least say that the theoretical lapse rate stays constant.

  139. mkelly says:

    Mr Turtle I gave you the thermodynamic difference between an open and closed system. My eyesight does grow dim with age but my thermo text book has not changed in years.

  140. Magic Turtle says:

    I have only just read jjthom’s post (2nd Mar.2:36 pm) above. His reasoning appears sound to me and his argument good.

    Last night I conceived another thought experiment similar to his thought experiments with mirrors only using internally silvered bell-jars instead, which points out the critical differences between open and closed thermodynamic systems and the semi-open/closed nature of the Earth’s system. It goes like this:

    First take a small, 100 Watt electric heating-coil and place it under a bell-jar that is transparent to IR as well as visible wavelengths of e-m radiation. Evacuate the air from inside the bell-jar and switch on the current to the coil. What will happen?

    Since the bell-jar is transparent to IR and shortwave radiation, the system is a thermodynamically open one in which the heating-coil is able to radiate its energy away into the surrounding space freely, although not by conduction or convection because it is contained in a vacuum. Therefore when the current is switched on, the coil will warm up until it is radiating energy at the rate of 100 Watts in total to match the 100 Watts of in-coming power supplied by the electric current. At this point, Power In = Power Out and the coil is said to be ‘in thermodynamic equilibrium’ at its ‘equilibrium temperature’. (This scenario is analogous to the situation of an air-less world like the Moon, which is receiving a constant supply of power from the Sun, reflecting some of it straight back out into space and emitting the rest back out to space after having absorbed it.)

    Now let us repeat this procedure, only using a different bell-jar whose internal and external surfaces have been completely silvered so as to be perfectly reflective of internal and external radiation on all wavelengths. What happens when we switch on the current this time?

    Since the coil’s radiance is prevented from leaving the space inside the bell-jar, the coil is unable to radiate its received power of 100 Watts away and so the energy accumulates inside it and since the coil is the only absorber inside the bell-jar it absorbs all of the radiation that is reflected off the internal walls of the bell-jar that strikes it. Thus the coil has 100 Watts of power coming in and none going out, so its temperature climbs up and up until the coil fuses and this cuts off the electrical current. No equilibrium temperature is possible in this unbalanced situation. (This could be analogous to a stellar nova, in which the products of the star’s core nuclear reactions are prevented from leaving the star whereby its internal energy mounts until it explodes. Fortunately the Earth is not likely to get into that catastrophic predicament in the foreseeable future!)

    Now let us repeat the procedure a third time with another bell-jar again, whose internal surface has been partially silvered so that it is partially reflecting on all wavelengths. When the current is switched on this time, some of the heating-coil’s radiance is able to escape to cool it and some is reflected off of the internal walls of the bell-jar to warm it. In this case, since the coil is receiving energy at the rate of 100 Watts from the electric current, it will warm until its radiation is leaving the bell-jar at the same rate of 100 Watts and at this point it will again be in thermodynamic equilibrium with its environment. However, because some of the coil’s radiance is being recycled back to it by reflection off the semi-silvered internal walls of the bell-jar, it is receiving energy at a greater rate than the 100 Watts from the electric current alone and so it now has a higher equilibrium temperature than it had in the first scenario accordingly. (This situation is analogous to that of a planet with a ‘greenhouse atmosphere’ which acts like a partially-silvered mirror to re-radiate back to the surface some of the outgoing surface radiance, which warms the surface to a higher equilibrium temperature than it would otherwise have with a perfectly transparent atmosphere as in the first scenario.)

  141. Stephen Wilde says:

    “Why should the surface temperature stay constant? The standard GHE theory can at least say that the theoretical lapse rate stays constant.”

    Because the surface temperature depends on density at the surface plus solar input. Density being a function of gravity and atmospheric mass.

    The more molecules per unit volume the greater the interaction with solar input and the higher the temperature.

    If the atmosphere expands there will be less molecules per unit of volume at the surface. The reduction in density at the surface is proportionate to the expansion which is in turn proportionate to the increase in temperature of the air.

    The net effect is warmer expanded air but no temperature change at the surface unless one also increases pressure (and density) or solar input.

    That is the natural implication of The N & Z findings and the Ideal Gas Law.

    If greater density at the surface gives a higher temperature then reducing that density must give a lower temperature all else remaining equal.

    Note that the theoretical lapse rate derived from gravity remains the same but the actual lapse rate changes.

    If ALL the atmosphere were of the same composition then the mass of the atmosphere would be governed by the composition and the actual and theoretical lapse rates would be the same.

    It is the varying compositions at different levels that causes the actual and theoretical lapse rates to diverge at different levels.

    Whatever the atmosphere is composed of the surface temperature is controlled by pressure and solar input and not by that composition.

    “The formula for the dry adiabatic lapse rate is -dT/dz = g/c_p. Because gases have different c_p, the c_p of air changes with composition.”

    On further investigation I found that the theoretical lapse rate set by gravity is not the same as the dry adiabatic lapse rate although I had been using it as such following the lead of others here and elsewhere.

    Both the dry and moist adiabatic lapse rates appear to be ‘actual’ lapse rates which can differ from the lapse rate imposed by gravity and therefore would vary according to composition.

    http://www.britannica.com/EBchecked/topic/330402/lapse-rate

    “The lapse rate of nonrising air—commonly referred to as the normal, or environmental, lapse rate—is highly variable, being affected by radiation, convection, and condensation; it averages about 6.5 °C per kilometre (18.8 °F per mile) in the lower atmosphere (troposphere). It differs from the adiabatic lapse rate, which involves temperature changes due to the rising or sinking of an air parcel. Adiabatic lapse rates are usually differentiated as dry or moist.”

    I have found much confusion as regards the term ‘environmental’ lapse rate. Some sources use it as the actual observed lapse rate and others as the theoretical or ‘normal’ lapse rate.

    Even in the above source there is room for confusion because once affected by radiation convection or condensation it is no longer the environmental lapse rate.

    To get round that I have been using the terms ‘theoretical’ and ‘actual’.

  142. Magic Turtle says:

    Stephen Wilde (3rd Mar. 9:53pm)

    “Now I see why you took objection to me saying that (the S-B law) doesn’t apply within an atmosphere.

    What I meant (obviously) is that it doesn’t apply within an atmosphere for the pupose of calculating the surface temperature of the planet.”

    I already knew that. The problem is that as far as I am aware you haven’t explained why you believe the S-B law doesn’t apply in this perfectly routine way that scientists the world over normally do apply it. What is the reason? I wait with bated breath.

  143. Stephen Wilde says:

    “you haven’t explained why you believe the S-B law doesn’t apply in this perfectly routine way that scientists the world over normally do apply it”

    To apply it correctly one must do so from outside the atmosphere to avoid interference from non radiative processes.

    As far as I know, it is only a small number of climate scientists who have tried to apply it from a point within an atmosphere.

    I’m sure that any other scientists outside that little group would say that for the purposes of S-B the planet and the atmosphere should be treated as a single unit and so the surface should be regarded as being at the top of the atmosphere and not at some point within the atmosphere.

  144. tchannon says:

    Magic Turtle says:
    March 4, 2012 at 3:34 pm (Edit)

    Yes that is the situation, not a problem.

    Now change to heating the element from outside the bell jar.

  145. tchannon says:

    Ben,
    I don’t follow enough to usefully comment.

  146. Magic Turtle says:

    Does anyone know how I can format my comments on this blog? Will the regular style of formatting-tags work?

    tchannon (4th Mar. 12:55 am):

    “I went looking for asymmetric atmospheric temperature processes, it is the asymmetry which is critical.”

    I don’t think it has to be critical; it depends on what problems you need to solve. For example if you just want to establish that there is a substantial difference between the actual, measured global mean temperature and a theoretical baseline-temperature determined by insolation alone, it will usually suffice simply to average the total amount of absorbed solar irradiance over the whole planetary surface and then use the S-B law straightforwardly to convert that quantity of power/unit area into a figure for absolute temperature. That theoretical baseline temperature may not have any real examples that demonstrate it anywhere in the known universe, but it doesn’t need to have any because it is just a conceptual device used for reference purposes like the origin of a graph or like the concept of ‘sea-level’. Have you ever seen, or even heard of a sea that is actually level? Nevertheless, we use the theoretical abstraction of ‘sea-level’ as a basic frame of reference from which to measure comparative heights on land and depths under water. In our determination of an object’s height above ‘sea-level’, we don’t usually need to take global asymmetry into account unless it’s for a special purpose. Likewise, we don’t need to take the asymmetry of other planetary processes into account unless that’s for a special purpose too.

    “Most of the warmists have argued a linear process, perhaps without the actual effect of gravity. Sure, fine, except now stick around and actually consider the rest, they run off.”

    This is also a bone of contention that I have with them. I have challenged them many times to demonstrate that the physics on which they claim their predictions are based is real and they have all tried to fob me off with rhetoric before ducking out of it, every time without exception. I see them as sadly deluded people who are in denial of the depths of their own ignorance.

    “Very often a simplified SB is used.”

    Is it? I didn’t know the S-B law could be simplified without destroying its meaning. Oh, hang on though – you mean like saying that the global warming effect of a doubling of atmospheric CO2 content is ~1ºC before feedbacks have ‘enhanced’ it? I think that proves my point.

    “Earth at xC mixed with flux y, ought to be T vs T, F vs. F. One is a proxy for the other via SB.
    Why is this done? ‘cos they can’t measure, as you say a plasma at 10kK kind of melts things, similarly it’s difficult pointing a flux sensor at the ground at ground level over a huge area, so the guessed T of the body is used.”

    Ah yes, there’s a lot of guesstimating going on unfortunately. However the S-B law can be as precise as we want it to be and it can enable us to determine power-flows, temperatures and emissivities throughout the system wherever the other two variables are known.

    “Yep except I don’t see a complete (greenhouse) theory and it is untestable.”

    In principle it’s not. In practice though it is impossible to test it empirically at the moment because we don’t know of two suitable planets (one with a pure GHG-atmosphere and one with a non-GHG atmosphere for comparison) to use as empirical test-beds. So for now we can only test bits of it empirically, but given the prevailing political situation on the planet one really cannot trust the results of even those in my opinion. There are also tests of logic, rationality and consistency at the theoretical level though and anyone who has the necessary critical reasoning skills should be able to apply such tests with rigour. So far the basic greenhouse theory seems to me to have come through with flying colours on that level.

    “N&Z? Don’t know but pressure in a broad sense explains much of what we see, radiative doesn’t without special cases. How exactly this works seems unclear. I want this stuff the get a good hearing without fear, good talk about it, explore. Doesn’t matter if there are mistakes and if just one good thing appears, great.”

    Thanks for giving me the link to the N&Z paper, which I have now read. I’m afraid you were right to think that it ‘doesn’t fit with my way of thinking’. That is one of the sloppiest, most unscientific, illogical and unself-criticised papers that I have ever read. It is riddled with factual and logical errors that make it worse than useless in my eyes because it is so misleading. I was tempted to do a thorough critique of it until I realised that there is so much that is wrong with it that there is not enough free time in my life to permit that. It is an awful paper and if I was one of its authors I would be hanging my head in shame, but its authors are titled professors so I can only think that they are unaware of the travesty that they have produced and that no-one around them has had the wit and the courage to tell them about it. How can two people of such low intellectual calibre have attained such eminent positions? They’re like Laurel and Hardy!

    “How exactly this works seems unclear.” You can say that again! LOL.

    If we put aside the imperative demands of the 1st law of thermodynamics for a moment and allow the bizarre possibility which the authors claim as a fact, that mere gravitational atmospheric pressure alone can somehow heat planetary surfaces in the complete absence of the requisite power-source to provide necessary heat-energy, the fact remains that they have not tested that idea empirically in the slightest degree although they pretend to have done so and appear to be quite sincere in saying that they have. They present their theory as an alternative to the greenhouse theory but they only ‘test’ their predictions of planetary temperatures on planets which have greenhouse atmospheres, where they have atmospheres at all! This is not real science. It’s computer-games and make-believe – just like the warmists are doing.

    I’m sorry that I can’t be more positive about it but I think it is better for me to be clear, open and honest about what I am seeing than to try to hide the truth to save some people from having the discomfort of facing it. Either we are intent on having with truth or we are not. If we really want the truth then we cannot afford to compromise with make-believe. That’s the way I see it.

  147. adolfogiurfa says:

    The IPCC’s judgement that the solar factor is negligible is based on satellite observations available since 1978 which show that the Sun’s total irradiance, though not being constant, changes only by about 0.1 percent during the course of the 11-year sunspot cycle. This argument, however, does not take into account that the Sun’s eruptional activity (energetic flares, coronal mass ejections, eruptive prominences), heavily affecting the solar wind, as well as softer solar wind contributions by coronal holes have a much stronger effect than total irradiance. The total magnetic flux leaving the Sun, dragged out by the solar wind, has risen by a factor of 2.3 since 1901 (Lockwood et al., 1999), while global temperature on earth increased by about 0.6°C. The energy in the solar flux is transferred to the near-Earth environment by magnetic reconnection and directly into the atmosphere by charged particles. Energetic flares increase the Sun’s ultraviolet radiation by at least 16 percent. Ozone in the stratosphere absorbs this excess energy which causes local warming and circulation disturbances. General circulation models developed by Haigh (1996), Shindell et al. (1999), and Balachandran et al. (1999) confirm that circulation changes, initially induced in the stratosphere, can penetrate into the troposphere and influence temperature, air pressure, Hadley circulation, and storm tracks by changing the distribution of large amounts of energy already present in the atmosphere.
    New Little Ice Age Instead of Global Warming? by Dr. Theodor Landscheidt:
    http://bourabai.narod.ru/landscheidt/new-e.htm

  148. Stephen Wilde says:

    “the authors claim as a fact, that mere gravitational atmospheric pressure alone can somehow heat planetary surfaces in the complete absence of the requisite power-source to provide necessary heat-energy”

    MT, the N & Z paper clearly says that continuing solar input is necessary to produce the observed result.

    I and others have told you that, several times, but you keep making that incorrect assertion.

    Unless you get your head around that point all your lengthy verbiage is a waste of everyone’s time.

  149. BenAW says:

    Stephen, pse give your definition of the temperature lapse rates you think exist and why they are relevant. Every post you make, another slightly different lapse rate seems to pop up.
    At least define:
    – Environmental Lapse Rate (ELR)
    – Dry Adiabatic lapse rate (DALR)
    – actual lapse rate
    – theoretical lapse rate.

    And try to keep the definitions short pse.

  150. Magic Turtle says:

    Tchannon (4th Mar. 8:31 pm):

    “Now change to heating the element from outside the bell jar.”

    Nah. It’s your idea; you do it.

  151. tchannon says:

    MT,

    Simplified SB omits the surface properties and/or wavelength part. Doesn’t matter for a simplistic body.

    N&Z is not about static pressure as many have superficially assumed, is more an observation of what is. I think the critical thing is a flow of energy through an atmosphere under pressure.

    Pressure broadening is likely to come into it.

    Evening out of heat distribution is another, at an extreme the poles and day/night sides are much the same. (the case with Venus, Marov ’78 paper will do on that)

    Exactly how so? That I don’t get. Yet.

    IMO a key thing to investigate in detail is the Jupiter excess heat, very careful checking. There is no known surface so it was excluded from N&Z but the temperature/pressure profile would fit. Now is the supposed excess heat generation from within with no definite causal real or not? Without it ghg effect is dead.

    Straight talking?
    Tempted but a sermon here, no.

  152. tchannon says:

    The U so jar? It rang tolled you so.

    In essence the half silvered cancels to unity. Internal temperature is constant.

    That leaves fiddling with the silvering to try and get a selective effect. Hello ghg arguments, ‘cept none seem to want to know about the whole system, preferring out only.

  153. Magic Turtle says:

    Stephen Wilde (4th Mar. 6:06 pm)

    “you haven’t explained why you believe the S-B law doesn’t apply in this perfectly routine way that scientists the world over normally do apply it”

    To apply it correctly one must do so from outside the atmosphere to avoid interference from non radiative processes.

    Oh? What specific kinds of interference would they be? It takes less than a second to measure the radiation emanating from the ground with a suitably shielded instrument. And even if it was difficult, I don’t see how that is any reason for saying that it should not, or cannot be done.

    But we don’t need to apply the S-B law to the surface for practical measurement of the temperature there anyway because we’ve already got simpler and easier methods of doing it. Our main need to apply it to the surface is theoretical and arises from our prior needs to a) calculate the amount of power that would be radiated from the surface at any given temperature of interest, and b) calculate the amount of planetary surface warming that any given quantity of absorbed insolation by itself could be expected to achieve. The S-B law makes those calculations possible. What is your prejudice against it?

    “As far as I know, it is only a small number of climate scientists who have tried to apply it from a point within an atmosphere.”

    I don’t see what numbers of scientists have to do with it. As Einstein said when he was presented with a petition signed by a hundred eminent scientists protesting that his theory was wrong: “If I was wrong, one would do.”

    “I’m sure that any other scientists outside that little group would say that for the purposes of S-B the planet and the atmosphere should be treated as a single unit and so the surface should be regarded as being at the top of the atmosphere and not at some point within the atmosphere.”

    I can’t imagine why they would say that. There is still no objective reason that I can see for the S-B law not to be used wherever anyone wants to use it. As I have pointed out before, it is a universal law and there are no intrinsic restrictions on its use or applicability.

  154. Magic Turtle says:

    Stephen Wilde (4th Mar. 9:48 pm)

    “the authors claim as a fact, that mere gravitational atmospheric pressure alone can somehow heat planetary surfaces in the complete absence of the requisite power-source to provide necessary heat-energy”

    MT, the N & Z paper clearly says that continuing solar input is necessary to produce the observed result.

    I and others have told you that, several times, but you keep making that incorrect assertion.

    Unless you get your head around that point all your lengthy verbiage is a waste of everyone’s time.

    I have not made an ‘incorrect assertion’.

    Look, from the Abstract of N&Z’s paper:
    “We show via a novel analysis of planetary climates in the solar system that the physical nature of the so-called Greenhouse Effect is in fact a Pressure-induced Thermal Enhancement (PTE), which is independent of the atmospheric chemical composition. Hence, the down-welling infrared radiation (a.k.a. greenhouse- or back-radiation) is a product of the atmospheric temperature (maintained by solar heating and air pressure) rather than a cause for it.”

    Clearly, they are saying that the planet’s atmospheric temperature is maintained by solar heating and air pressure together regardless of the atmosphere’s chemical composition.

    I understood this was what they were saying when I read their paper for the first time yesterday, but I also understood it as being what you were saying from the moment I finished reading your blog-post on the day that you posted it! You know, I’m getting a bit fed up with reading your lectures about what I haven’t understood when I have understood it. In any case, I thought you had ‘given up on me’ and had stopped trying to ‘correct’ my views. Yet here you are, still at it!

    Kindly stop laying your reluctance to understand onto me.

  155. PaulK says:

    Stephen: “Because the surface temperature depends on density at the surface plus solar input.

    The reduction in density at the surface is proportionate to the expansion which is in turn proportionate to the increase in temperature of the air.

    The net effect is warmer expanded air but no temperature change at the surface

    If greater density at the surface gives a higher temperature then reducing that density must give a lower temperature all else remaining equal.

    Whatever the atmosphere is composed of the surface temperature is controlled by pressure and solar input and not by that composition.”

    You are going round and round a circle. It doesn’t bring anyhting new into the table. You have to show the mechanism how your theory works. Please, present the calculations how it goes when CO2 doubles in the troposphere.

    Stephen: “That is the natural implication of The N & Z findings and the Ideal Gas Law.”

    Unfortunately I have read N&Z. You get no help from them. If you disagree, please, show the calculations.

    Stephen: “I have found much confusion as regards the term ‘environmental’ lapse rate. Some sources use it as the actual observed lapse rate and others as the theoretical or ‘normal’ lapse rate.”

    I think in these discussions it suffices to use “the lapse rate” or “the average lapse rate” when referring to the atmospheric/environmenta/actual/real/observed/… lapse rate. When you refer to a specific theoretically derived lapse rate like the dry adiabatic lapse rate, you should state that.

  156. Stephen Wilde says:

    Ben AW

    As a result of the confusion found in the various sources I suggest the following:

    Theoretical lapse rate – The lapse rate that would exist due to gravity alone in a homogenous atmosphere with no compositional variations.

    Actual lapse rate – The rate which actually exists within an atmosphere of mixed composition.

  157. Stephen Wilde says:

    MT,

    PLease define how the ‘requisite’ power source exceeds the actual power source from the sun.

  158. Stephen Wilde says:

    “Oh? What specific kinds of interference would they be?”

    Anything that alters the rate of energy flow through an atmosphere such as convection, conduction, evaporation,condensation, density and pressure.

    For example, ozone in the stratosphere reverses the lapse rate there which slows down the rate of energy loss to space to a rate less than would be expected from radiation alone.

    Every component of an atmosphere slows down radiative energy loss to space to some degree or another with the result that the S-B Law fails to provide an accurate result unless one applies it from a point beyond such disruptive influences.

    And as I said, the point only applies to the obtaining of an accurate result for the temperature of the planet. You can still use S-B within the atmosphere for other purposes.

  159. Stephen Wilde says:

    Ben AW

    I forgot about mass so it should be:

    Theoretical lapse rate – The lapse rate that would exist due to gravity alone in a homogenous atmosphere of a given mass with no compositional variations.

    Actual lapse rate – The rate which actually exists within an atmosphere of a given mass but of mixed composition.

  160. Stephen Wilde says:

    Going a step further one could query whether a homogenous atmosphere of Ozone would have the same theoretical lapse rate as a homogenous atmosphere of Nitrogen.

    The actual lapse rate would be different but not the theoretical lapse rate determined by gravity and atmospheric mass.

    What happens instead is that the Ozone atmosphere becomes warmer than would be allowed by the theoretical lapse rate BUT it expands and reconfigures the circulation so that on average from surface to top of atmosphere the theoretical lapse rate is achieved.

    The expansion would reduce density at the surface (reducing surface temperature from what it otherwise would have been) and alter the effective radiating height to offset the warming effect from the thermal characteristics of Ozone.That would bring the atmosphere back into balance with the theoretical lapse rate.

    If the atmosphere were to fail to reconfigure itself to achive the theoretical lapse rate then there would be a long term energy gain leading to the atmosphere being lost to space or a long term energy loss leading eventually to the atmosphere freezing to the planetary surface.

    What happens in practice though is that if any planet does have an atmosphere whereby the composition cannot allow the actual lapse rate to be brought into line with the theoretical lapse rate then just so much of the atmosphere as necesssary is either boiled off to space or frozen to the surface in order to restore the necessary balance once more.

    Water being the primary GHG on Earth the coming and going of ice caps could be explained on that basis.

    All the above assumes a constant level of solar input because solar input also alters atmospheric volume and thus the actual as opposed to theoretical lapse rate.

    Thus a decrease in insolation would shrink the volume of the atmosphere and cause the actual lapse rate to diverge from the theoretical lapse rate.

    The system stability would then be maintained by an increase in the portion of water vapour as an atmospheric gas being locked away in ice.

    The subsequent meltring of the ice caps would then maintain system stability when the level of solar input increases again.

    All that follows naturally from the Ideal Gas Law and fits the observations of N & Z.

  161. Stephen Wilde says:

    Gravity determines the slope of the theoretical lapse rate.

    Atmospheric mass (and therefore pressure) determines the temperature from which the theoretical lapse rate begins at the surface.

    The level of solar input affects the height and therefore the volume of the atmosphere that the theoretical and actual lapse rates must traverse.

    The composition of the atmosphere only affects the actual lapse rate which must then be brought into line with the theoretical lapse rate by circulatory changes.

    The effect of any given level of solar input is affected by the composition of the atmospheric molecules with which it must interact so that also affects the actual lapse rate with consequent circulatory changes.The reversed lapse rate from sun/ozone reactions in the stratosphere is a prime example.

    Likewise, any variations in energy release by the oceans will affect atmospheric composition (water vapour availability) which must also be countered by circulatory changes.

    So, the theoretical lapse rate is always there in the background as a constant for any given gravitational field and atmospheric mass.

    Everything other than those two variables only changes the actual lapse rates (usually multiple such lapse rates on a single planet) and circulatory changes then reconcile the difference to maintain system stability.

  162. BenAW says:

    Stephen Wilde says:
    March 5, 2012 at 9:48 am

    “The composition of the atmosphere only affects the actual lapse rate which must then be brought into line with the theoretical lapse rate by circulatory changes.”

    You sure have a talent to confuse people. Were are you heading now?

    See https://tallbloke.wordpress.com/2012/02/23/ben-wouters-how-the-earths-surface-maintains-its-temperature/#comment-18889

    I think this is simple explanation what the ELR would be WITHOUT disturbances.
    Since a world without disturbances doesn’t seem to exist, lets accept that the ELR at every place and every moment is (slightly) different.
    I assume that the AVERAGE ELR, as given in the International Standard Atmosphere ICAO gave us will look like the simple model I linked to above.

  163. Stephen Wilde says:

    BenAW

    I am finding the sources confusing since no one seems to have provided a distinct name to the component of any given lapse rate that results from gravity and atmospheric mass alone.

    There are ambiguous and contradictory usages all over the place.

    Your simple explanation of an ELR without disturbances is as good as I have seen anywhere but still doesn’t quite cut it because ANY atmosphere seems to cause a divergence from what I think we should call the theoretical lapse rate caused by gravity and atmospheric mass alone.

    Some atmospheres cause only a very minor divergence and others a larger divergence but in the end the circulation has to cancel out any such divergence if the atmosphere is to be retained.

    The only way we can get the system to work as observed is to say that there is some component to ANY lapse rate which is set by gravity, mass and the consequent pressure and that is a baseline which cannot be overcome. Any thermal characteristics arising from atmospheric composition can then only affect the temperature and VOLUME of the atmosphere at any given level of solar input and NOT surface temperature.

    That appears to be implicit in the concepts behind the Ideal Gas Law and the outcomes which that Law predicts but I have not yet found a source defining it any better than my continuing current efforts.

    The International Standard Atmosphere works just fine for Earthly gravity and atmospheric mass. Changing atmospheric mass or the strength of the Earth’s gravitational field are the only things that will alter it. Changes in composition fail to disturb the Standard Atmosphere which makes the point for us.

    If changes in composition from place to place could affect the Standard Atmosphere then aviation would be in some difficulty. Astronautics in particular relies on very precise parameters which would be upset if atmospheric composition could interfere with the predictions derived from the Standard Atmosphere.

  164. BenAW says:

    Stephen Wilde says:
    March 5, 2012 at 1:55 pm

    The ISA is afaik an average of many observations, which gave the atmosphere as the ISA describes.
    It’s not something that the atmosphere has to return to.

    “if changes in composition from place to place could affect the Standard Atmosphere then aviation would be in some difficulty.”
    Not really. The ISA is used to calibrate altimeters, make calculations etc.
    In real live, when flying at a fixed pressure altitude (eg. 33.000ft / FL330) you may actually be climbing and descending very slowly when flying to and from high and low pressure areas.

  165. Tenuc says:

    Meanwhile, back in the real world, the Arctic sea ice is defying climate change science once again…

    http://ocean.dmi.dk/arctic/icecover.uk.php

    I wonder how much the theoretical back radiation from that little lot will increase global temperature? Not much, I fear… 🙂

  166. Stephen Wilde says:

    You know more about the ISA than I do so I’ll defer to you on that.

    The problem we seem to have is in describing the limits that gravity and atmospheric mass place on the lapse rate that the system is capable of achieving between surface and top of atmosphere when any sort of atmosphere will have an effect on the actual lapse rate. Often the atmospheric composition varies vertically resulting in multiple actual lapse rates acting together within a single atmosphere.

    One way or another the netted out effect of all lapse rates in the vertical column has to result in energy out equalling energy in at the top of the atmosphere for system equilibrium so there needs to be a separate lapse rate which the system always works back to.

    That equilibrium, according to the Ideal Gas Law is a function of pressure at the surface below the atmosphere (from gravity and atmospheric mass) plus solar input. Anything that seeks to disturb that established relationship (such as variable composition) seems to be negated by a volume increase.

    Otherwise the N & Z observations and the Ideal Gas Law cannot hold true.

    If pressure and solar input remain the same when composition changes then atmospheric volume and temperature rise but not surface temperature.

    Gravity sets the slope of the basic lapse rate, pressure/molecular density sets the starting temperature at the surface for a given level of solar input, solar input sets the total height of the atmosphere and circulation changes prevent composition changes from altering the height of the atmosphere set by solar input .In the process, one seens an actual lapse rate or rates which differ from the basic lapse rate but in the end it all nets out to energy in = energy out at equilibrium.

    If that is wrong then it isn’t just me and a clear statement as to why it is wrong would be appreciated so that I don’t waste more of my time and that of others.

  167. Magic Turtle says:

    tchannon (5th Mar. 1:14 am)

    “Simplified SB omits the surface properties and/or wavelength part.”

    I’m not sure what you mean by ‘Simplified SB’. The Stefan-Boltzmann law is already simplified to its essence and states that the intensity of electromagnetic radiation emitted by a body is proportional to the body’s emissivity and the 4th power of its absolute temperature. ‘Simplify’ it any more and you lose the meaning of the law, I imagine.

    The S-B law operates independently of other properties like wavelength and is unaffected by them. This means that when applying it we don’t necessarily need to take the way in which the radiation is emitted into account. So if a star, say, was to emit all its radiation on just one wavelength, the S-B law would still apply to it and you could use it to determine the star’s mean effective surface temperature just by knowing the intensity of the radiation at the star’s surface. Well, we can do the same thing on Earth, regardless of what other properties and behaviours the climate system may have.

    Complications may arise in extreme cases when, say, a composite body has different emissivities at different wavelengths and you need to take those differences into account for some reason, perhaps if you were trying to ascertain the overall temperature of a laser for example. But the Earth’s surface is not such an extreme case because we already know that its emissivity on all infrared wavelengths is almost 1.0 as nearly as makes no practical difference to most climate calculations.

    “N&Z is not about static pressure as many have superficially assumed, is more an observation of what is.”

    By my reading of their paper, they definitely are proposing that static pressure is the primary cause of planetary surface temperatures being raised above the levels supported by insolation alone. They also say it themselves in their ‘Abstract’ which I quoted to Stephen earlier.

    “IMO a key thing to investigate in detail is the Jupiter excess heat, very careful checking. There is no known surface so it was excluded from N&Z but the temperature/pressure profile would fit. Now is the supposed excess heat generation from within with no definite causal real or not? Without it ghg effect is dead.”

    The excess outgoing radiation from Jupiter’s atmosphere could not be an effect of GHGs according to the greenhouse theory, which holds that GHGs only recycle radiation back to the surface without affecting the In-Out radiation-balance at the top of the atmosphere. So whatever is causing the In-Out imbalance at the Jupiterian TOA, I think it can’t be GHGs and the GHG effect is not at risk from it.

    “Straight talking?
    Tempted but a sermon here, no.”

    I have not come here to give sermons either. However, I cannot prevent other people from projecting their disowned inner-sermonisers (if they have them) onto me of course. I can only point it out when they do so.

  168. BenAW says:

    Stephen Wilde says:
    March 5, 2012 at 4:41 pm

    Often the atmospheric composition varies vertically resulting in multiple actual lapse rates acting together within a single atmosphere.

    And the problem with that would be ???

  169. tchannon says:

    I am referring to the missing e of a real body.

  170. Magic Turtle says:

    tchannon (5th Mar. 1:22)

    “The U so jar? It rang tolled you so.”

    That’s very p/funny. But what does it mean?

    “In essence the half silvered cancels to unity. Internal temperature is constant.”

    I had a feeling you were going to say that. It’s another reason why I wanted you to run the experiment instead of me. 😉

    “That leaves fiddling with the silvering to try and get a selective effect. Hello ghg arguments, ‘cept none seem to want to know about the whole system, preferring out only.”

    I suggest you try some of my patent magic silvering with which you can alter the percentages of in-coming and out-going radiances differently to reflect the fact that the IR-absorption wavebands of GHGs occupy a smaller proportion of the in-coming solar energy spectrum than they do of the out-going surface radiance spectrum. The maths are a little more complicated in this situation but since you have equilibrium conditions at both the planet’s surface (object inside the bell jar) and the TOA (walls of the bell jar), you just need to write two simple energy-flow equations to describe the two different balances at the two sites and then solve them simultaneously.

    This gives you the most basic, elementary and simplistic model of the greenhouse principle that it’s possible to conceive – the ‘essential model’, so to speak. Once you’ve got that you can modify it to take other factors and features of the climate system into account (naturally making the maths more complicated at every stage of course) and gradually build up a more comprehensive model of the detailed power-flows throughout the whole system. (I am using the term ‘model’ in its original scientific sense as a representative image of a real system here and not in the sense that ‘climate modellers’ like Hansen use it to mean a sort of computerised numerology that has no particular internal relationships with the real climate system.)

  171. Stephen Wilde says:

    “Often the atmospheric composition varies vertically resulting in multiple actual lapse rates acting together within a single atmosphere.

    And the problem with that would be ???”

    That isn’t the problem. The ‘problem’ is defining the netted out consequence as a separate lapse rate governed by pressure derived from the strength of the gravitational field and atmospheric mass.

    I’ve tried one definition but I’m not entirely happy with it.

    What is your suggestion ?

  172. tchannon says:

    The knell of word playing.

    Profiles on effect vs wavelength is a large part of the problem, where asymmetric effect is not allowed. This is one thing I see is missing from many arguments, want to have the effect in one direction only. Sun is heating in that band via ghg too but less than it would without it.

  173. PaulK says:

    (Stephen, you probably did not see this message, because it was waiting for moderation while you wrote several messages.)

    Stephen: “Because the surface temperature depends on density at the surface plus solar input.

    The reduction in density at the surface is proportionate to the expansion which is in turn proportionate to the increase in temperature of the air.

    The net effect is warmer expanded air but no temperature change at the surface

    If greater density at the surface gives a higher temperature then reducing that density must give a lower temperature all else remaining equal.

    Whatever the atmosphere is composed of the surface temperature is controlled by pressure and solar input and not by that composition.”

    You are going round and round a circle. It doesn’t bring anyhting new into the table. You have to show the mechanism how your theory works. Please, present the calculations how it goes when CO2 doubles in the troposphere.

    Stephen: “That is the natural implication of The N & Z findings and the Ideal Gas Law.”

    Unfortunately I have read N&Z. You get no help from them. If you disagree, please, show the calculations.

    Stephen: “I have found much confusion as regards the term ‘environmental’ lapse rate. Some sources use it as the actual observed lapse rate and others as the theoretical or ‘normal’ lapse rate.”

    I think in these discussions it suffices to use “the lapse rate” or “the average lapse rate” when referring to the atmospheric/environmenta/actual/real/observed/… lapse rate. When you refer to a specific theoretically derived lapse rate like the dry adiabatic lapse rate, you should state that.

  174. Stephen Wilde says:

    PaulK said:

    “You have to show the mechanism how your theory works.”

    It isn’t my theory. It is established science that maximum temperature is generated at the surface where incoming solar energy has more molecules per unit of volume to interact with.

    My difficulty is in formulating the implications of that in a way that is more widely acceptable to those who never knew it. I agree that in the process I am going round in circles because there is no recognised term for the lapse rate determined by gravity and atmospheric mass alone. The terms you suggest are already in use for the actual lapse rate which varies greatly for a variety of reasons.

    I think the task may well be impossible and will need to await more real world data showing that changes in CO2 or indeed in GHGs make not a jot of difference to surface temperature but only to the general air circulation.

    Such is implied by the Ideal Gas Law which contains no term for radiation yet it works well enough to be recognised as a Law.

    I think this thread has run its course unless someone can bring in something new.

  175. BenAW says:

    Stephen Wilde says:
    March 5, 2012 at 10:40 pm

    “It isn’t my theory. It is established science that maximum temperature is generated at the surface where incoming solar energy has more molecules per unit of volume to interact with.”

    Imo the maximum surface temps will be reached WITHOUT atmosphere. In that case the surface can only cool by radiation. With atmosphere conduction and convection are also available, better means of transporting energy than radiation alone.

  176. Magic Turtle says:

    Stephen Wilde (5th Mar. 7:29 am)

    “ MT,

    PLease define how the ‘requisite’ power source exceeds the actual power source from the sun.”

    Stephen,

    That you should need to ask me this question tells me that you do not understand basic thermodynamics. It is elementary, but since you appear not to know the elements of the subject I think I should probably preface my statement of the definition which you request with a brief explanation of the fundamental relationship between ‘power’ and ‘temperature’.

    Power is defined in physics as the ‘rate of expressing energy’ – not just ‘energy’. When energy is static, it is unexpressed energy, but when energy flows it becomes power. So power is the term that is used generically for ‘energy-flux’, ie. the rate of ‘energy-flow’.

    The absolute temperature of a body is defined as the average kinetic energy of the individual constituents that make it up. (In the case of the Earth’s atmosphere those are its molecules.) So if the average KE of the constituents increases, its temperature increases; if their average KE decreases, so does the body’s temperature.

    If, then, a body abides at a constant temperature, the average KE of its constituents is constant and this means that energy is flowing into it at the same rate as energy is flowing out of it. In other words, at equilibrium Total Power In = Total Power Out.

    But it is the amount of power flowing through the body that determines the average KE of its constituents and its temperature accordingly. And since Power In = Power Out, we can calculate a body’s temperature from either our knowledge of the amount of its total Power Input or from that of its total Power Output. It makes no difference to the end result, whichever one we take. The total Power Input to a planet warmed by a star is the total amount of the star’s radiance that the planet absorbs. This sets the equilibrium temperature of the planet at a certain level which I shall call the ‘standard insolation temperature’ (SIT).

    However, we find from observation that the surfaces of most planets are at more or less constant global mean temperatures that are significantly higher than their respective SITs, which means that their Total Power Inflows and Outflows at the surface must also be significantly greater than the total amounts of power that they have absorbed from the insolation which set their SITs. This is anomalous and it requires a quantity of power to be supplied to the surface in addition to that which is supplied by total insolation alone in order to explain the observed equilibrium temperature at the surface being higher than the SIT.

    We may now define the power-anomaly at the surface of a planet as the difference between its global mean absorbed insolation and the global mean power that is required to maintain the surface at its observed equilibrium temperature.

    I hope that this meets your request but if there is anything that you do not understand about it then by all means please ask freely.

  177. tchannon says:

    If this thread is talked out I’ll take the opportunity to raise something.

    The only experimental method which comes to mind which might be able to answer some questions would be based on the gas centrifuge, similar to those used for isotope enrichment.

    These turn out to have a history but not surprisingly hard information is not readily available, nor I suspect is it wise to research too hard given the sensitivity of the subject.

    History.
    When the idea of using artificial gravity to produce a mass profile in a centrifuge was first tried it didn’t work and dropped out of usage.

    Sometime I think in the 1930s the reason for the problem was guessed, correct, and from then on development continued, on what proved to be a very difficult engineering problem.

    The problem was heat on the outside of the centrifuge; it worked if the centrifuge drum operated in a vacuum. This tripped a nasty problem, the drum had to be driven at high speed through vacuum seals, the seals are the problem.

    I gather these drums for isotope usage are long and various internal gas flows are used, necessary as a process, a flow of gas.

    In the case I have in mind no flow is desirable and a vacuum forming on the axis of the drum is desirable. We are looking for a pressure gradient in free space .

    Is there anything which seems viable?

  178. Magic Turtle,

    Over 100 years ago Svante Arrhenius made this statement:
    “If the quantity of carbonic acid in the air should sink to one-half its present percentage, the temperature would fall by about 4°; a diminution to one-quarter would reduce the temperature by 8°. On the other hand, any doubling of the percentage of carbon dioxide in the air would raise the temperature of the earth’s surface by 4°; and if the carbon dioxide were increased fourfold, the temperature would rise by 8°.”

    The atmospheric CO2 concentration has risen by 40% since Arrhenius made his prediction so we should have experiencd a temperature increase of 1.95 Kelvin. That is not even close so it makes sense to look for other explanations.

    You don’t have to look far because contemporaries of Arrhenius including Angstrom refuted his theory. Take a look at what Niels Bohr had to say. The link can be found on the website of one of the people who has already commented here:

    Click to access gh_experiments.pdf

    This is no big mystery. The derivation of the adiabatic lapse rate is taught in high schools and provides an explanation of the observed temperature gradient in Earth’s atmosphere. The same theory works for Venus and Titan as Nikolov and Zeller have shown. I believe it also works for Jupiter. I recommend this explanation by Rodrigo Caballero (University College Dublin). Start at page 133:

    Click to access PhysMetLectNotes.pdf

    Please note that there is no mention of “Radiative Transfer Equations”. In order to establish the “Adiabatic Lapse Rate” a method of heat transfer is required. In the troposphere the dominent heat transfer processes are convective so RTEs can safely be ignored.

    James Hansen with his “Runaway Greenhouse Effect” is to blame for much of the sensationalist nonsense that passes for “Climate Science”.

  179. Stephen Wilde says:

    “Imo the maximum surface temps will be reached WITHOUT atmosphere”

    Of course, that is so.

    But where there is an atmosphere of ANY composition the highest temperature within that atmosphere is at the surface.

  180. Stephen Wilde says:

    “This is anomalous and it requires a quantity of power to be supplied to the surface in addition to that which is supplied by total insolation alone in order to explain the observed equilibrium temperature at the surface being higher than the SIT”

    Alternatively it requires no additional power but merely a delay in transmission of incoming energy between arrival at the surface and radiation to space.

    Thus energy accumulates at the surface to produce that higher temperature.

    You have correctly stated that what happens is an ongoing energy interchange by way of recirculation of energy between surface and air which accounts for the discrepancy.

    You have said that the explanation is downward IR from GHGs.

    I and others say that it is a dynamic energy exchange between the surface and the air molecules in contact with or just above the surface involving ALL atmospheric molecules in that location and not just the GHGs.

    The Ideal Gas Law supports that latter proposition.

    We cannot resolve that disagreement, merely note each other’s respective points of view and await a resolution in due course.

  181. BenAW says:

    gallopingcamel says:
    March 6, 2012 at 5:12 am

    The derivation of the adiabatic lapse rate is taught in high schools and provides an explanation of the observed temperature gradient in Earth’s atmosphere.

    No, it does not. The adiabatic lapse rates are ONLY used to calculate the temp change for rising or sinking parcels of air. The observed, environmental, actual or whatever lapse rate is just a description of the temp. profile of the atmosphere.

  182. BenAW says:

    Stephen Wilde says:
    March 6, 2012 at 5:45 am

    “But where there is an atmosphere of ANY composition the highest temperature within that atmosphere is at the surface.”

    Only true as long the surface is heated. On Antarctica during winter there is a huge part of the atmosphere that is warmer than the surface.

    See http://weather.uwyo.edu/upperair/sounding.html
    select Antarctica, Gif to 10mb, 2011, juli and select the station in the middle of Antarctica.

  183. PaulK says:

    Stephen says: “… there is no recognised term for the lapse rate determined by gravity and atmospheric mass alone.

    I think the task may well be impossible and will need to await more real world data showing that changes in CO2 or indeed in GHGs make not a jot of difference to surface temperature but only to the general air circulation.”

    It is not the terms. It is the lack of clear mechanism and formulas. The big problem in the circulation theory is that the amount of air, that rises in one place decreasing the lapse rate, also goes down in another place, increasing the lapse rate, and the average lapse rate doesn’t change.

  184. Stephen Wilde says:

    BenAW said:

    “Only true as long the surface is heated”

    Of course, taken as read. The whole scenario of an atmosphere not frozen to the surface is dependent on solar input.

    PaulK said:

    “The big problem in the circulation theory is that the amount of air, that rises in one place decreasing the lapse rate, also goes down in another place, increasing the lapse rate, and the average lapse rate doesn’t change.”

    That isn’t a problem. That is the secret of its efficiency. Faster circulation or a deeper circulation or a reconfigured circulation moves energy at different speeds between surface and space so as to negate any changes other than from pressure or solar input.

    The problem is convincing those unfamiliar with the Ideal Gas Law as to the different consequences on surface temperature when one increases surface pressure or solar input so as to change the energy available to the system as a whole compared to changing atmospheric composition which merely redistributes the same amount of energy within the system.

    I don’t think I can convince such people on the basis of the data currently available. A simple appeal to logic appears to be insufficient.

    Some way of describing the lapse rate set by gravity/atmospheric mass as separate from that set by atmospheric composition is what we need but none of the usages or definitions that I have seen achieve that. The gravitational component is always jumbled up in the various other types of lapse rates and even those other types have acquired multiple names for the same thing.

    Someone else will have to unravel it but in the meantime if one accepts the Ideal Gas Law then the logic should be clear and radiation becomes irrelevant for surface temperature unless that Law is falsified.

    Radiation is only relevant at the Earth’s ‘surface’ which must be top of atmosphere where the Earth system meets space if Earth and atmosphere are treated as a single linked unit, as they are, and that is where the S-B equations are satisfied.

  185. PaulK says:

    PaulK: “The big problem in the circulation theory is that the amount of air, that rises in one place decreasing the lapse rate, also goes down in another place, increasing the lapse rate, and the average lapse rate doesn’t change.”

    Stephen: “That isn’t a problem. That is the secret of its efficiency. Faster circulation or a deeper circulation or a reconfigured circulation moves energy at different speeds between surface and space so as to negate any changes other than from pressure or solar input.”

    I wanted to point out that the average lapse rate does not change with circulation, fast or slow.

    Stephen: “The problem is convincing those unfamiliar with the Ideal Gas Law …”

    Please!

    Stephen: “I don’t think I can convince such people on the basis of the data currently available. A simple appeal to logic appears to be insufficient.”

    Your logic is just not enough. We need formulas.

  186. Stephen Wilde says:

    PV = nRT

    No provision for radiation, yet it works.

    There is your formula.

    If solar input stays the same (T) and pressure at the surface stays the same (P) then any change in internal energy distribution resulting from a change in atmospheric composition can only affect volume ( V) which is freely variable.

    Since an increase in V then reduces n which is the number of molecules per unit volume at the surface then that offsets any increase in surface temperature that would otherwise have occurred from the change in atmospheric composition.

    The reason being that for the same level of solar input more molecules per unit volume at the surface (n) will generate a higher temperature and less molecules per unit volume at the surface will generate a lower temperature.

    Believe it or not, as you choose, but the Ideal Gas Law would be invalidated if the surface temperature could rise without an increase in surface pressure or solar input.

    There seems a marked reluctance to accept that more molecules in a given volume will generate a higher temperature when irradiated than would less molecules in the same volume.

    That is the mechanism one needs to account for observations.

  187. RKS says:

    PaulK says:
    March 6, 2012 at 11:04 am

    PaulK: “The big problem in the circulation theory is that the amount of air, that rises in one place decreasing the lapse rate, also goes down in another place, increasing the lapse rate, and the average lapse rate doesn’t change.”

    Stephen: “That isn’t a problem. That is the secret of its efficiency. Faster circulation or a deeper circulation or a reconfigured circulation moves energy at different speeds between surface and space so as to negate any changes other than from pressure or solar input.”

    I wanted to point out that the average lapse rate does not change with circulation, fast or slow.

    Stephen: “The problem is convincing those unfamiliar with the Ideal Gas Law …”

    Please!

    Stephen: “I don’t think I can convince such people on the basis of the data currently available. A simple appeal to logic appears to be insufficient.”

    Your logic is just not enough. We need formulas.>>>>>>>>>>>>>>

    Have you looked at the non-linear function in N&Z’s equ. 7 – derived from empirical planetary data?

  188. Stephen Wilde says:

    “Have you looked at the non-linear function in N&Z’s equ. 7 – derived from empirical planetary data?”

    How do you interpret that ?

  189. RKS says:

    Stephen Wilde says:
    March 6, 2012 at 12:46 pm

    “Have you looked at the non-linear function in N&Z’s equ. 7 – derived from empirical planetary data?”

    How do you interpret that ?>>>>>

    It’s a key part of their theory which allows actual computation of surface temperature form insolation and atmospheric pressure alone via their equ. 8.

    As a proponent of N&Z’s work you already knew that!

    My post was in response to PaulK’s request for formulae to uphold the use of the ideal gas laws.

  190. Stephen Wilde says:

    RKS,

    Thanks.

  191. PaulK says:

    Stephen: “PV = nRT

    No provision for radiation, yet it works.

    There is your formula.”

    I have no problem with that. It works fine also with radiation.

    Stephen: “If solar input stays the same (T) and pressure at the surface stays the same (P) then any change in internal energy distribution resulting from a change in atmospheric composition can only affect volume ( V) which is freely variable.

    Since an increase in V then reduces n which is the number of molecules per unit volume at the surface then that offsets any increase in surface temperature that would otherwise have occurred from the change in atmospheric composition.”

    You do this:

    1) P0*V0 = n0*R*T0

    The temperature increases (T1 > T0)

    2) P0*V1 = n0*R*T1

    Volume V0 has expanded to V1. You change the game by taking a new volume of the original size V0 with less air molecules (n1 n1 = n0*(T0/T1)

    5) 3+4 => P0*V0 = n0*R*T0

    The equation (5) is the same as (1). And that’s all it is. It has nothing to do with the increased temperature. See corrected post March 6, 2012 at 2:21 pm

  192. PaulK says:

    RKS: “Have you looked at the non-linear function in N&Z’s equ. 7 – derived from empirical planetary data?”

    Yes. It is a fitted function that has nothing to do with GHE.

  193. PaulK says:

    (Sorry again. I hope moderator removes extras [original strike, intermediate trashed –Tim])

    Stephen: “PV = nRT

    No provision for radiation, yet it works.

    There is your formula.”

    I have no problem with that. It works fine also with radiation.

    Stephen: “If solar input stays the same (T) and pressure at the surface stays the same (P) then any change in internal energy distribution resulting from a change in atmospheric composition can only affect volume ( V) which is freely variable.

    Since an increase in V then reduces n which is the number of molecules per unit volume at the surface then that offsets any increase in surface temperature that would otherwise have occurred from the change in atmospheric composition.”

    You do this:

    1) P0*V0 = n0*R*T0

    The temperature increases (T1 > T0)

    2) P0*V1 = n0*R*T1

    Volume V0 has expanded to V1. You change the game by taking a new volume of the original size V0 with less air molecules

    3) P0*V0 = n1*R*T1

    4) 1+3 => n1 = n0*(T0/T1)

    5) 3+4 => P0*V0 = n0*R*T0

    The equation (5) is the same as (1). And that’s all it is. It has nothing to do with the increased temperature.

  194. tchannon says:

    The formation of the linear vs. altitude lapse rate seems to cause headscratching but the pure radiative lapse rate is aphysical, cannot happen on a planet in a thermal flux with gravity. This is the case for all the solar system.

    I showed some evidence and pointed at two explanations in the casual post I did 25th Dec. Context was get something up after the police raid and before the N&Z mushroom, which came by chance from that. https://tallbloke.wordpress.com/2011/12/25/palestine-sagan-and-atmospheric-physics/

    This shows the lapse rate for Venus and Jupiter. The double plain language referenced is here, science forum

    In essence gas heated at ground level becomes more buoyant but might not be heated strongly enough to actually rise much, hence there is an energy limit before the process gets going. This in practice produces the effect we see, The maths is around, seems rather hairy, kind of thing best approximated.

  195. Stephen Wilde says:

    To get T0 to rise to T1 at the surface you need solar input to increase or surface pressure to increase.

    If you just get more radiative or conductive energy zipping about in the air T0 at the surface does not increase. V increases instead.

    How does that affect your equations ?

  196. BenAW says:

    Let’s see were we are:

    -we have GHE claiming the earth warms from avg 255K to avg 288K due greenhouse gasses by backradiation. 33K warming.
    -we have N&Z claiming the earth warms from avg 155K to avg 288K due gravity. 133K warming.

    We have my little theory explaining the earth is ALREADY at 275K from cooling since ancient times.
    We only need to explain 13K warming using the SUN, GHE and gravity.
    Since no one has dismissed my ideas, I assume they are at least reasonable.
    Any reason why this idea isn’t exploited to it’s fullest?

    So why are you trying to sqeeze energy out off lapse rates by bending them around in the fifth dimension or something similar?????

    It’s pretty simple: sun warms earth, earth warms atmosphere, atmosphere radiates at the top with the same avg energy as the sun dumps on the surface. Lapse rate explained.

  197. Stephen Wilde says:

    “It’s pretty simple: sun warms earth, earth warms atmosphere, atmosphere radiates at the top with the same avg energy as the sun dumps on the surface. Lapse rate explained.”

    Exactly as per my contention (and that of many others).Though there is probably some direct solar warming of the atmosphere too.

    The trouble is that we have no definition for that simple baseline lapse rate.

    We know from the Ideal Gas Law that it is a function only of solar input plus surface pressure.

    It is a sort of fantasy lapse rate that never exists between surface and TOA but on every planet with an atmosphere it underlies the entire structure of the atmosphere.

    All the definitions I have seen are variations on the variable lapse rate(s) of one type or another that we see between surface and TOA.

    Whatever the surface emits in excess of solar input is absorbed and recycled back to the surface by the atmosphere but it is nothing to do with GHGs. It involves pressure acting on the entire atmosphere whilst it is subjected to irradiation from the sun.

    And that baseline lapse rate always ensures that solar energy in equals radiated energy out but only at TOA which is the only place that one can correctly apply S-B if one is trying to determine the temperature of the whole Earth system i.e. Earth and atmosphere as a combined unit.

  198. Stephen Wilde says:

    Perhaps the absence of a clear definition is why N & Z introduce the term ‘Atmospheric Thermal Effect’.

  199. BenAW says:

    Stephen Wilde says:
    March 6, 2012 at 5:01 pm

    The trouble is that we have no definition for that simple baseline lapse rate.

    We know from the Ideal Gas Law that it is a function only of solar input plus surface pressure.

    The lapse rate is just a list of temperatures against increasing altitude. Every square mm on the earth has it’s own lapse rate. We only measure a limited number at meteo stations worlwide by letting balloons rise twice a day.

    How do you “know” that the ideal gas law is valid for a whole atmosphere. The atmosphere doesn’t look like an ideal gas to me.

  200. RKS says:

    “Stephen Wilde says:
    March 6, 2012 at 5:03 pm

    Perhaps the absence of a clear definition is why N & Z introduce the term ‘Atmospheric Thermal Effect’.”>>>>>>>>>>>>>>>>

    Sorry to sound pedantic Stephen, but the correct term is Atmospheric Thermal Enhancement, derived from their equations 7 & 8.

    See below from their paper:-

    “3. The Atmospheric Thermal Enhancement
    Previous studies have noted that the term Greenhouse Effect is a misnomer when applied to the
    atmosphere, since real greenhouses retain heat through an entirely different mechanism compared to
    the free atmosphere, i.e. by physically trapping air mass and restricting convective heat exchange.
    Figure 4. According to observations, the
    Earth-Atmosphere System absorbs on
    average a net solar flux of 239 W m-2, while
    the lower troposphere alone emits 343 W
    m-2 thermal radiation toward the surface.
    Figure 3. Solution to the two-layer model in Eq. (4) for Ts and Ta as a function of
    atmospheric emissivity assuming a convective atmosphere (gbH = 0.075 m/s). Also
    shown is the predicted down-welling LW flux (Ld). Note that Ld ≤ 239 W m-2.
    6
    Hence, we propose a new term instead, Near-surface Atmospheric Thermal Enhancement (ATE) defined as a non-dimensional ratio (NTE) of the planet actual mean surface air temperature (Ts , K) to the average temperature of a Standard”

  201. Magic Turtle says:

    gallopingcamel (6th Mar. 5:12 am)

    “The atmospheric CO2 concentration has risen by 40% since Arrhenius made his prediction so we should have experiencd a temperature increase of 1.95 Kelvin. That is not even close so it makes sense to look for other explanations.”

    As far as I can make out the ‘Arrhenius greenhouse law’ is the claimed theoretical justification for the IPCC’s infamous logarithmic formula for the forcing from CO2 alone, around which so much climate mythology has been woven. However the IPCC has modified it and has invented a suitable value for pre-industrial CO2-concentration so that their calculation of the ‘forcing’ from the hypothetical 40% increase in atmospheric CO2 since the pre-industrial epoch fits neatly onto the trend-line of the well-massaged temperature-record, which shows warming at the rate of about 0.7ºC/century. So the disparity between predicted and ‘observed’ temperature-rise no longer exists and the warmists’ version of the greenhouse theory remains smugly unfalsified.

    There is so much fudging, mystery-mongering and downright fraud going on in climate science these days that I think it is doubtful whether any sincere climate theory can be properly tested empirically and that means we are restricted to critiquing each one at the theoretical level only for the time being. On that level the basic greenhouse theory appears to remain unscathed to me.

    Thanks for the link to the quote of Bohr’s observation that CO2-molecules do not warm, but only become excited when they absorb an IR-photon. I suspect that quote may have been taken out of context because there doesn’t seem to me to be any obvious reason why the energy of excitation should not be transferred to other molecules in the atmosphere and back to the ground by photon exchanges and inter-molecular collisions in the normal manner.

    “This is no big mystery. The derivation of the adiabatic lapse rate is taught in high schools and provides an explanation of the observed temperature gradient in Earth’s atmosphere. The same theory works for Venus and Titan as Nikolov and Zeller have shown. I believe it also works for Jupiter.”

    The derivation of the lapse rate may explain the observed temperature gradient. I think what it does not explain though is the actual temperature at the bottom of the atmosphere. That does seem anomalous to me, as I have pointed out.

    I think it is ‘anomalous’ in the sense that at the surfaces of all of the planets you mention, more power is coming out of them than is going into them from solar energy alone. Where is this extra power coming from? N&Z and apparently Stephen too, are saying that it is coming originally from ‘atmospheric pressure’. But that proposition is in conflict with the 1st law of thermodynamics (ie. the ‘law of conservation of energy’) and with the gas laws that are being cited as proof of it.

    Ask any thermodynamic engineer who actually works with heat engines and the gas laws every day. If you could show him a way to get more power out of a pressurised gas than he has had to put into it by pressurising it in the first place, he would fall in love with you and want to have your babies! But of course that’s not going to happen because no-one could show him how to do it, since according to current physics it is impossible to get more power out of something – even an atmosphere – continuously than is being put into it continuously by the power-source.

    “James Hansen with his ‘Runaway Greenhouse Effect’ is to blame for much of the sensationalist nonsense that passes for ‘Climate Science’.”

    Has Hansen been promoting the Runaway Greenhouse Effect too? I thought his fanciful ‘positive feedback from water vapour’ ended in a finite quantity. Oh well, perhaps you are right. I find his pronouncements so absurd that I barely take notice of anything he says these days.

  202. PaulK says:

    Stephen: “If you just get more radiative or conductive energy zipping about in the air T0 at the surface does not increase. V increases instead.”

    P does not change. T does not increase. You locked also V.

    Let’s compare two situations P0*V0 = n0*R*T0 and P1*V1 = n1*R*T1 =>

    (P1/P0)*(V1/V0) = (n1/n0)*(T1/T0)

    When we compare the same set of molecules n1 = n0 =>

    (P1/P0)*(V1/V0) = T1/T0

    Because the pressure does not change on surface P1 = P0 =>

    V1/V0 = T1/T0

    When the temperature does not change T1 = T0 =>

    V1 = V0

    You have locked also the volume.

  203. PaulK says:

    What is it in N&Z that makes you think their result is somehow significant?

    They simply fit a function to the data of some planets. The Earth is among the fitting set. So, there is no prediction or anything with the values the function gives for the Earth. It just gives out the values that were used when fitting the function.

    And anyway, fitted has no explanatory power.

  204. Stephen Wilde says:

    “T does not increase. You locked also V.”

    T of the air changes but not T of the surface.

    Changing T of the air unlocks V and prevents warming of the surface because there is then less density at the surface and the power of N & Z’s ATE declines proportionately to the increase in T of the air.

    So the warmer the air the greater the volume and the less ATE at the surface.

  205. Stephen Wilde says:

    “Where is this extra power coming from? N&Z and apparently Stephen too, are saying that it is coming originally from ‘atmospheric pressure’. ”

    No it isn’t. It is coming from the sun because more molecules per unit volume will produce more energy from a given amount of solar irradiation than do less molecules per unit of volume.

    You keep repeating that incorrect canard.

  206. Stephen Wilde says:

    “How do you “know” that the ideal gas law is valid for a whole atmosphere”

    Because PV = nRT works.

    I know there is infinite variability on the smaller scale in three dimensions but it still works.

    I’ll have to let you all chunner amongst yourselves for a few days because I’ll be away.

  207. Magic Turtle says:

    Stephen Wilde (6th Mar. 5:54am)

    “Alternatively it requires no additional power but merely a delay in transmission of incoming energy between arrival at the surface and radiation to space.

    Thus energy accumulates at the surface to produce that higher temperature.”

    I’m afraid that explanation doesn’t work. You see, as I have explained, power is not just energy; it is the rate at which energy is flowing. The anomaly arises because energy is flowing out of the surface faster than the energy of solar irradiance is pouring into it, yet it remains at a constant temperature and keeps on radiating energy away faster than the Sun is supplying it. Delaying the release of energy from the surface can have no effect on the disparity between the rate of solar energy-inflow and the rate of surface energy-outflow in the long term.

    “You have correctly stated that what happens is an ongoing energy interchange by way of recirculation of energy between surface and air which accounts for the discrepancy.

    You have said that the explanation is downward IR from GHGs.

    I and others say that it is a dynamic energy exchange between the surface and the air molecules in contact with or just above the surface involving ALL atmospheric molecules in that location and not just the GHGs.”

    I can confirm that you have stated my position correctly. However, although we agree that the required extra power-input to the surface is coming from ‘recirculation of energy between surface and air’, ‘my’ greenhouse theory specifies how this energy-recycling is achieved whereas your alternative theory does not. Your explanation of the ‘dynamic energy exchange between the surface and the air molecules in contact with or just above it’ is vague and indeterminate. It also seems to me to rely on the invention of new physics which somehow overcomes the hitherto inescapable constraints of the 1st and 2nd laws of thermodynamics. The gas laws provide no release from these laws’ constraints since they are fundamental while the gas laws are derivative and based upon them.

    “The Ideal Gas Law supports that latter proposition.”

    I don’t see how it does. The Ideal Gas law is not an original power-source.

  208. Magic Turtle says:

    Stephen Wilde (6th Mar. 6:56 pm)

    “Where is this extra power coming from? N&Z and apparently Stephen too, are saying that it is coming originally from ‘atmospheric pressure’. ”

    No it isn’t. It is coming from the sun because more molecules per unit volume will produce more energy from a given amount of solar irradiation than do less molecules per unit of volume.

    You keep repeating that incorrect canard.

    If the warming of the planet causes the atmosphere to expand, then its volume (V) will increase and the atmospheric density (n) will decrease. That means we have fewer molecules per unit volume, not more.

    Furthermore, only GHG-molecules can absorb incoming solar radiation directly in significant amounts. Their special radiation-absorbent property is what makes them special ‘greenhouse’ gases instead of just ordinary gases that don’t absorb radiation particularly well.

    Just remind me again, who is it that is repeating incorrect canards?

  209. BenAW says:

    Magic Turtle says:
    March 6, 2012 at 9:07 pm

    Thus energy accumulates at the surface to produce that higher temperature.”

    I’m afraid that explanation doesn’t work. You see, as I have explained, power is not just energy; it is the rate at which energy is flowing. The anomaly arises because energy is flowing out of the surface faster than the energy of solar irradiance is pouring into it, yet it remains at a constant temperature and keeps on radiating energy away faster than the Sun is supplying it. Delaying the release of energy from the surface can have no effect on the disparity between the rate of solar energy-inflow and the rate of surface energy-outflow in the long term.

    As long as at the top of atmosphere the energy in = energy out there isn’t a problem. System earth isn’t loosing more energy to space than it receives from the sun.

  210. Stephen Wilde says:

    i) “If the warming of the planet causes the atmosphere to expand, then its volume (V) will increase and the atmospheric density (n) will decrease. That means we have fewer molecules per unit volume, not more.”

    The planet doesn’t warm from GHGs. The air warms and expands whilst the surface remains at the tempersature set by insolation and pressure.

    ii) “ordinary gases that don’t absorb radiation particularly well.”

    They don’t need to absorb or emit radiatively. We are dealing with events at or just above the surface where conduction and convection dominate thus involving all the ‘ordinary’ gases.

    iii) “The anomaly arises because energy is flowing out of the surface faster than the energy of solar irradiance is pouring into it, yet it remains at a constant temperature and keeps on radiating energy away faster than the Sun is supplying it”

    The atmosphere holds onto energy a little longer than the surface would in the absence of an atmosphere. That energy is then constantly recycled to and fro between surface and air thus maintaining a constant temperature as long as solar input and pressure do not change.
    The important region is at or just above the surface where conduction and convection dominate. Radiation becomes irrelevant.

  211. Magic Turtle says:

    Stephen

    A businessman is being sued for tax-evasion. In court the prosecuting barrister explains his case to the Judge and the Jury.

    Prosecutor: “M’lud, this case is being brought because the Department of Inland Revenue has investigated the Defendant’s personal accounts and has found them to be anomalous. The Defendant’s accounts show that his expenditures over the past twenty years have been at the constant rate of £390,000 per year although he has disclosed a regular income of only £237,000 per year while his savings in his offshore bank account have remained constant at a colossal £15 million. The Crown alleges that he must have another source of income in the region of at least £153,000 per year that he has not disclosed in order to avoid paying the tax on it.”

    The Prosecution then calls in the Tax Officer who investigated the Defendant’s accounts and he confirms all that the Prosecution has alleged. The Defendant does not call any witnesses and simply appeals to the Court’s sense of reason and justice when explaining his side of the case.

    Defendant: “Sir, I am an honest businessman simply trying to make an honest living in these hard times. I do not understand high finance and I leave all that to my accountant. He is the one who should be standing here in the dock, not I. From the day that I employed him he told me, indeed, he insisted that I give him all of the money that comes to me and he would make it grow for me. I don’t know how he does it, but I do as he has told me and when he gives my money back to me there’s more of it! So I spend it since I know there was plenty more where that comes from.”

    Prosecutor: “Do you really expect the Court to believe that you were so naive as not to question your accountant about where the extra money was coming from? Surely you did.”

    Defendant: “Yes, of course I did!”

    Prosecutor: “Well, what did he say? How did he explain it?”

    Defendant: “He said the delay entailed in his keeping hold of my money after I’d given it to him enabled it to accumulate. He tried to tell me in detail how it happened and I remember him saying something about putting my money under pressure, but it was all way above my head and too technical for me to understand, so I gave up in the end and just accepted it. Because the money was plentiful I haven’t needed to touch my £15 millions of savings and because I have spent all the money that he’s given me I haven’t put any more away in savings either, so that’s why the £15 million in my savings account overseas has stayed the same for all these years.”

    Well, I don’t know about you but he’s convinced me. Good luck to him, I say! 😉

  212. tchannon,

    Thanks for mentioning Jupiter. The Galileo probe measured pressure and temperature to -132 km where “Zero” is the level where the pressure is one bar.

    The measured lapse rate is consistent with an atmosphere that is mostly hydrogen and helium. Absolutely no need to involve “Radiative Transfer Equations” and trace gasses to explain the temperature gradient.

    One might have expected something strange because there is no “Surface” as is the case with rocky planets. Also, Jupiter radiates more than twice the energy it receives from the sun, indicating that there is an internal heat source.

    Nevertheless, the adiabatic lapse rate in Jupiter’s troposphere appears to be consistent with what is being taught at the University College of Dublin.

  213. Magic Turtle @ March 6, 2012 at 5:52 pm,

    It seems that we are much closer than I thought. Somehow I got the impression that you believed that traces of CO2 were somehow a dominant driver of Earth’s surface temperature in spite abundant evidence falsifying that hypothesis.

  214. PaulK says:

    gallopingcamel: “The measured lapse rate is consistent with an atmosphere that is mostly hydrogen and helium. Absolutely no need to involve “Radiative Transfer Equations” and trace gasses to explain the temperature gradient.”

    I have never seen anybody claiming that radiative transfer and trace gases explain the lapse rate. Can you give me an example?

  215. Stephen Wilde says:

    Income £237,000 and Expenditure £237,000

    Capital sum invested of £15 million in reserve.

    £153,000 moving into and out of reserves as necessary to maintain cash flow in the face of varying payment and receipt times.

    That is all that the Earth’s atmosphere does.

    It invoves all atmospheric molecules and not just GHGs.

    The error in MT’s post above is that there is no outgoing £390,000 because he has only counted one side of a two way flow. Only 247 Wm2 gets out to space.

    The taxman cannot claim that the £153,000 coming in is additional income when it is just one side of short term transfers to and from reserves.

    The Earth only radiates 237Wm2 to space and not 390Wm2 to space which is why the example is wrong.

    Off on a break now.

  216. tallbloke says:

    Wow! 215 comments. Well done Stephen and thanks to all for the high standard of discussion.

  217. Magic Turtle says:

    Hi tallbloke,

    ’Pleased to make your acquaintance. I am a newcomer to your blog and have found it very engaging. I was attracted by the high standard of discussion here, which seems notable to me for its freedom from the abusive troll-activity that infests many other blogs and makes sensible discourse impossible on them. No doubt the moderators here must be given credit for that.

    If you have been following the comments under Stephen’s blog you will already know that I am not a warmist and I do not buy the IPCC’s and the Hockey Team’s alarmist technobabble with which they pretend to have demonstrated that human emissions of GHGs are endangering the planet and making drastic mitigation measures urgently necessary. To my mind, that pitch has got ‘megascam’ written all over it – an impression that has only been reinforced by the Climategates 1 & 2 emails, for which I want to take the opportunity to thank you for playing a significant part in helping to make available to us all. In my view your close encounter of the nasty kind with the Norfolk Constabulary was also salutary in showing us how close this country has come to being a secret-police state and how far the warmist cabal are prepared to go to stop the truth of what they are really doing from coming out.

    However, that is all about politics and I am less concerned about that than I am about what is being done to science in this brouhaha about global warming. Today’s political battles are transient and in a hundred years time they will be half-remembered history for schoolchildren to study, but I think science is one of the central pillars of civilization that needs to be preserved and continually perfected throughout time if civilization is to flourish. But it appears to have become something of a political football in the global warming debate and I think that has to be bad for science and civilization. The warmists have perverted it quite deliberately in order to make their argument persuasive, it seems to me. Their so-called ‘climate science’ is no science at all in my eyes and looks more like an exercise in manufacturing and maintaining a grand illusion. Their ‘science’ can easily be refuted and shown to be bogus. Using the IPCC’s own data, I can prove quite simply, easily and straightforwardly that man-made global warming from human society’s CO2-emissions cannot be proceeding any faster than about 0.03ºC per century at the present time. That is not merely uncatastrophic; it is undetectable!

    I think the warmist threat to honest science is an insidious one because most people are unaware of it. And because warmist ‘climate science’ is now being taught in our schools and universities as standard, a whole generation of young people is growing up thinking that it has been educated in the ways of science when in fact it has been miseducated in them and has been given a mass of false impressions and illusions about science that it will need to unlearn before it can start to learn real science. Naturally people who become aware of this travesty that the warmist establishment has made of real science tend to become sceptical of warmist theory and to reject it wholesale. But in doing that, I think they are in danger of throwing the baby out with bathwater because the essential greenhouse principle that the warmists have hijacked and perverted to their political advantage is sound and well-supported by modern physics. I believe it is for precisely this reason that they have hijacked it!

    So now another threat to honest science emerges in the form of those anti-warmists who reject not only the warmist superstition and propaganda dressed up as ‘climate science’ but who also reject the scientifically-sound foundation of the greenhouse theory that the warmists are claiming as their justification. I think these anti-greenhouse theorists may be sincere but the position that they have adopted places them in denial of scientific laws and principles that support the essential greenhouse theory and automatically in conflict with real science generally as a result.

    Of course real science must always be open to challenges from new theories and that is as necessary in the field of climate science as anywhere else. But when a new theory is presented I think it needs to be debated fully and brought to a decisive head before the debate is wound up. And in regard to Stephen’s new theory of planetary heating, that doesn’t seem to have happened yet as far as I can see, although I do think we have got close to it. So if it’s OK with you, I for one would like to continue the discussion here in his absence and try to thrash the matter out. However, I won’t attempt to do that without your permission.

    Best wishes,
    MT.

  218. tallbloke says:

    Hi MT and thanks for this comment. I think the best thing to do in Stephen’s absence is post your comment as a new thread, which it is in my opinion worthy of, and discuss its merits in its own right. That way, Stephen doesn’t lose out on being up to date with his own thread, and we can also discuss the wider issues your comment raises.

    That OK with you?

  219. Magic Turtle says:

    That’s fine by me, tallbloke. How should I go about doing that? (It’ll take me a few days as I’m pretty tied up at the moment.)

    [Reply] No problem, tidy up the comment in your own time as you wish and repost it here. I’ll post it as a new thread and delete it from this one.

  220. PaulK, March 7, 2012 at 10:13 am

    The point of this post is that planetary surface temperatures are to a first order determined by the gas laws, Newtonian gravity and thermodynamics. There is no need to involve RTEs and absorptive trace gasses. The idea that CO2 is the dominant driver of surface temperature is rejected outright.

    Whatever influence CO2 does have is too small to be determined empirically with any accuracy. My point is that radiative heat transfer is overwhelmed by convective processes in the troposphere and may safely be neglected.

  221. Roger Clague says:

    Magic Turtle please give some details and references to what you believe to be the ‘scientifically sound foundation of the greenhouse theory’.

    And an explanation and defence of the theory by yourself or someone else

    [Reply] This will be dealt with on a new thread as I said above. Not here. TB

  222. PaulK says:

    gallopingcamel: “The point of this post is that planetary surface temperatures are to a first order determined by the gas laws, Newtonian gravity and thermodynamics. There is no need to involve RTEs and absorptive trace gasses.”

    Last time you talked about the lapse rate, not the surface temperature.

    gallopingcamel: “Whatever influence CO2 does have is too small to be determined empirically with any accuracy.”

    It is impossible to determine empirically, because the Earth is too complex system.

    gallopingcamel: “My point is that radiative heat transfer is overwhelmed by convective processes in the troposphere and may safely be neglected.”

    Radiative heat transfer out to space is the only way there is to keep the Earth cool. So, you cannot neglect it.

  223. […] Comments Michele on X-class, nice try, missedBrian H on The Sound of SilencePaulK on Stephen Wilde: The Myth of…James on Doug Proctor: A geologist on s…Roger Clague on Stephen Wilde: The Myth […]

  224. PaulK,
    You are correct to say that radiative transfer of heat has an important role in emitting heat into space.

    It also is the dominant heat transfer process in the stratosphere which accounts for the fact that temperature usually rises with altitude in the upper atmosphere.

    In the convective part of the atmosphere convection and baroclynic eddies move the heat around with a tiny contribution from radiative processes.

  225. Magic Turtle says:

    Thanks Tallbloke. Please feel free to delete my last comment above now.

    Thanks also for adding those kind words of introduction – and for the cartoon too, which lightens it nicely to my mind.

  226. Samoht says:

    Sorry Stephen, but your theory is plainly nonsense. You completely ignore the fact that molecules of CO2, H2O, CH4 and others have very strong interaction if IR radiation due to their molecular structure. Their vibration modes coincide with significant parts of Earth black body radiation spectrum. CO2 IR lasers are using this fact.

    You can also arm yourself with a cheap IR thermometer gun and point it skywards. It measures the temperature of what you see – the IR back radiation. Measure on clear day, even in middle of a sunny day, and the clear sky will read some -50 Deg C or even less. Aim at a cloud and you will see the temperature of the cloud, perhaps 0 Deg or whatever your cloud temperature may be. That is why in a clear night your car will get so clod that condensation forms while it will not on a cloudy night.

    The chemical composition of the air above you will significantly alter its IR transmission spectrum.

  227. Stephen Wilde says:

    There is considerable doubt as to whether any current measuring devices are able to adequately separate out the energy flux from surrounding molecules.

    Clouds simply block upward radiation and thereby put a stop to continued radiative cooling.

    Furthermore, clouds usually drift in from warmer regions bringing advected warmer air along with them.

  228. Stephen Wilde says:

    “the clear sky will read some -50 Deg C or even less.”

    You expect to get significant DWIR from that ?

    Even if there is downward IR the air circulation just reconfigures to remove it so as to restore the surface temperature set by pressure and insolation.

  229. wayne says:

    Stephen, I think your question of “back radiation” lies within this explanation below. Seems by this all atmospheres and all atmospheres have it in the same proportion ruled by equipartition.

    Sorry it has taken me six months to reply to this thread Stephen but it has taken a LOT of digging to get myself a handle on what really matters in planetary atmosphere temperature profiles. I have picked up a lot of clues from many including gallopingcamel, Nikolov & Zeller, Miskolczi, Ramanathan and the CERES team, even Huffman, but none of them seemed to give the complete, 100%, answer but each had hints in their texts.

    gallopingcamel said:
    “The derivation of the adiabatic lapse rate is taught in high schools and provides an explanation of the observed temperature gradient in Earth’s atmosphere.”

    Absolutely. I have come to realize that a lapse rate is a terrible metric to use if you are striving to describe an atmosphere, it is the pressure ratio between two layers and the mean heat capacity ratio (λ=Cp/Cv) adjusted for the natural greenhouse function (1 minus 1/3) that exactly describes each atmosphere. The GH adjustment is therefore G=2/3 for optically thick atmospheres.

    This line of thought derives from some topics: heat capacity ratio (λ), degrees of freedom of molecules (DOF), heat capacities (Cp & Cv), speed of sound in gases, potential temperature ( Φ=T(P0/P)^(R/Cp) ), and equipartition of energy per mode bi-directionally, always.

    Take earth by the ’76 Standard Atmosphere or the ISA, at 11020 meters the temperature there and upward is 216.65K. The pressure here is 22629 Pa. Our atmosphere is predominately composed of linear di and tri atomic molecules setting the Cp/Cv to be right at 1.4 decreased very slightly for the IR active components and most importantly at their concentration.

    G=2/3

    On Earth:

    Mean DOF (degrees of freedom) = 5.008
    λ = 1+2/DOF
    (note: DOF = 2/( λ-1)
    P0 = 22629
    216.65‹K› * (101325 / P0)^(G * (λ-1)/λ)
    = 288.156‹K›

    On Venus:

    Mean DOF = 5.77
    λ = 1+2/DOF
    P0 = 1‹atm›
    338.3‹K› * (92‹atm› / P0)^(G * (λ-1)/λ)
    = 735.002‹K›

    ( By the way, the 5.77 DOF was from some actual lab experiments to measure CO2’s effective Cp. This derived value came from a Cp/Cv of approximately within error of 1.3466. Remember, CO2 is linear and has no permanent moment dipole)

    You can plug in any pressure level for either atmosphere and you will find the temperature at that pressure level will always very closely match (+/- 0.1°C) either Earth’s standard atmosphere or Venus’s International Reference Atmosphere (VIRA) very closely, too close to ignore. I digitized a plot of the VIRA atmosphere from a peer reviewed paper and the values are close to 735.1K at surface at 92‹atm› and the other end at 60 km and 23093 Pa where the temperature is 262.5‹K›.

    Caveat One: It seems logical to me that this relation with a G of 2/3 should only hold for atmospheres where the per-line tau or optical thickness is greater than one, maybe even much greater than one. Earth’s mean tau is 1.87 and it seems to hold by this relationship so that seems to give us a somewhat scale of magnitude.

    Caveat Two: Why the G=2/3? In one respect that seems logical also. The surface has an energy pressing upward and at most lines are all absorbed at some level, matters not where, and that absorption likewise presses both downward and upward to space. You get two pressures within the atmosphere and one pressure to space giving you the 2/3. That’s a bit simplistic but that’s one explanation. The other might be that horizontal radiation within the atmosphere itself is always bound and guaranteed absorption since horizontally all strong radiation lines are homogenous. That would give 4/6th or 2/3rd, six being possible directions in three dimensional space and all other radiation not included in this 4/6th is destined to escape upward, the 1/3rd portion. All of this would require proper differential analysis of course but each in its own right seems logical macroscopically viewing the atmosphere as a single element.

    Caveat Three: Does this hold for each and every thick planetary atmosphere?
    Don’t know about Saturn and Jupiter yet, and here is why you don’t want to get drug into thinking that lapse rates are anything but descriptive (not causal) in nature. Both of those gas giants have a much lower environmental lapse rate but due to there huge size and gravitational pull the pressure ratios across levels are much further apart in kilometers that on the Earth of Venus and so even though pressure level to pressure level ratios are close to the Earth and Venus, looking at temperatures, the lapse rates are far apart in value. Watch out.

    Another thing I have learned is that just by knowing the speed of sound and the mean molar mass of an atmosphere you can, along with Van der Waals adjustments, get the empirical λ which is much better than relying on idealistic theoretical values. Maybe one day NASA can lower an explosive charge to explode on the surface and actually measure Venus’s mean speed of sound vertically across its atmosphere so we can get as actual mean λ and DOF which would add credence to this.

    And that brings me to why CO2 levels have but a miniscule influence on Earth’s mean surface temperature, the 5.008 might increase to say something like 5.018 due to the doubling of CO2 from 0.039% but that only budges the surface temperature, and get this, downward by less than 0.1°C. This assumes the isothermal portion of the lower stratosphere maintains the same temperature.

    Something seems amiss; something besides CO2 increased the temperature in the last few decades. Could it have been the sun after all in some manner?

    Maybe this might give someone the push to read and help me understand this process a bit deeper for I’m rather new to meteorology, thermodynamics and climatology.

    If someone knows why this relationship exists please let me know.

    But, I am still digging along this line of evidence.

    [Reply] Wayne, you’ve been busy! I’ll post this as a new thread and see if we can get some input. Hans will be very interested I’m certain.

  230. wayne says:

    One more note on G=2/3. Ramanathan(1987?) list the g at 0.332, Miskolczi (2007) lists the g of 0.333. I just took the more than evident of g=exactly 1/3 to give the G as 1-1/3 or 2/3. Assuming here a simple geometric explanation which may not be really true.

  231. wayne says:

    Stephen, one more thing. You might wonder how the exponent R/Cp in the potential temperature equation ended up as (λ-1)/ λ.

    Under http://en.wikipedia.org/wiki/Heat_capacity_ratio
    Cp = λnR/(λ-1)
    We are dealing molar so n=1, drop it, and therefore flip Cp:
    1/Cp = (λ-1)/(λR)
    therefore:
    R/Cp = (λ-1)/λ

    Add the G and it at least matches the entire profiles for both Earth and Venus with correct heat capacity ratios applied and that is where I became VERY interested in this relationship, very curious as to why. I thought you and gallopingcamel might be curious too.

  232. tallbloke says:

    Wayne: Brilliant stuff. I’ve copied your last three comments and kicked off a new thread to discuss it here:

    Wayne Jackson: New Identity Linking Meteorological Phenomena

  233. Samoht says:

    Stephen:

    There is considerable doubt as to whether any current measuring devices are able to adequately separate out the energy flux from surrounding molecules.

    Uh, so you have an opinion on GW but don’t believe that science has the ability to differentiate the radiation physics of the molecules in the atmosphere?

    I guess this highlights where you guys miss significant bits of the science.

    Have a look at this: http://en.wikipedia.org/wiki/File:Atmosfaerisk_spredning.gif
    Its the IR transmission transmission spectrum of the Earth’s Atmosphere.

    Also this is a good article: http://geosci.uchicago.edu/~rtp1/papers/PhysTodayRT2011.pdf

    Perhaps looking at this will start the process rolling of digging a bit deeper into radiation heat transfer.

  234. tallbloke says:

    Samoht: “Its the IR transmission transmission spectrum of the Earth’s Atmosphere.”

    Isn’t that the theoretical IR transmission transmission spectrum of the Earth’s Atmosphere?

    What Stephen is pointing out is that there are severe calibration issues with pyrgeometers. This was discussed here recently, starting from this comment:

    The Fraud of the Atmospheric Greenhouse Effect Part 2: Moving to Reality

    “Also this is a good article:”

    Pierrehumberts radiative model is unphysical because it takes insufficient account of the dominant processes of convection and latent heat transfer in the troposphere. The Stefan Boltzmann equation applies to radiating surfaces with a vacuum between them.

    I guess this highlights where you guys miss significant bits of the science.

  235. Stephen Wilde says:

    Thanks to wayne for making a serious effort to tease out the truth.

    Although my contentions seem obvious from first principles (to me at least) it appears to require quite some intellectual effort and detailed work to demonstrate it to those who have somehow developed a very unphysical interpretation of the science.

    I look forward to seeing how his thread develops.

  236. Samoht says:

    Stephen, read this please. The IR transmission spectrum of the Earth atmosphere is based on actual measurements.
    And the change in the spectrum over time due to the rise in GG can be seen using satellite monitoring. http://www.skepticalscience.com/print.php?r=35

  237. Samoht says:

    Oh and I really think you should go outside (and anybody else who is doubting the importance of the radiative properties of the atmosphere above us) with an IR thermometer gun and point it skywards in various conditions. This is a really instructive exercise.
    The radiative heat transfer is very important, especially at night, in determining heat loss from the ground, the ocean surface etc.
    The Stefan Boltzmann equation applies to radiating surfaces with a vacuum between them.. Indeed, however in Earths case we have a mix of gases with some highly efficient IR absorbers in-between. That is the whole point.

  238. Samoht says:

    Stephen, the radiative forcing of anthropogenic GG is about 2W/m2 on average, not 300W/m2
    300W/m2 is roughly the entire outgoing – and incoming for that matter – average global radiation exchange.

    Your friend in Sweden has no chance to see 2W/m2 in his anecdotal temperature data with the equipment he is using. So much must be obvious to you!

    The 2W/m2 applied over the entire globe is however enough to upset the apple cart for Earth’s climate and set the changes in motion that we are witnessing.
    We also see the back radiation clearly in the spectra that are recorded:

    The image above shows the spectrum of the back radiation measured at the surface. The contributions of the various IR active molecules are noted. CO2 is the dominant component.
    Source:
    http://ams.confex.com/ams/Annual2006/techprogram/paper_100737.htm

    Do you still think we don’t measure the IR components attributable to various molecules taking part in the Earth’s radiative heat exchange?

    Also: In order to be active absorbers and re-emitters of IR in the Black Body bands of the Earths spectrum molecules have to show vibrational modes that fall into this part of the IR spectrum.
    Trilateral molecules such as CO2, O3, H20 or quadrilateral molecules such as CH4 have this capacity especially.
    For an introduction: http://www.elmhurst.edu/~chm/vchembook/globalwarmA5.html

    [Reply] The ~2W/m^2 is the theoretical TOA imbalance. However, the uncertainty on the measurements is around +/-4W/m^2. The radiative flux can notionally be split into upward and downward components, but the net flux averaged over the ocean is ~65W/m^2 upwards. – TB

  239. Stephen Wilde says:

    “What they found was a drop in outgoing radiation at the wavelength bands that greenhouse gases such as carbon dioxide (CO2) and methane (CH4) absorb energy”

    Wasn’t there also evidence of a similar scale of INCREASE in OLR at other wavelengths ?

  240. Entropic man says:

    Stephen Wilde

    I am having trouble visualising how the energy budget for the Earth could produce the temperatures we observe without back radiation from GHGs.
    Could you put up figures for an energy budget diagram which operates as you describe.

  241. Stephen Wilde says:

    Just take the alleged figure for DWIR and show it as a dynamic energy exchange between the surface and the warm air molecules directly above it.

    Exactly the same amount goes from ground to air as goes from air to ground.

    That squares the circle just as well as proposing DWIR.

  242. Samoht says:

    Stephen, “Exactly the same amount goes from ground to air as goes from air to ground.” that is not true. The net radiation balance – the amount going out minus the amount coming back down – is what counts. The net balance is not zero.

    And even an atmosphere considerably colder than the ground will still radiate back to the ground, reducing the net amount that is going out in the process.

    The reduction in outgoing radiation together with its spectral resolution showing which gases are involved is measured, not just calculated from theory. You seem to completely ignore all this.

    Btw. 2W/m2 in radiative forcing attributed to GG may not seem much but over the 510 million Km2 of the Earth surface it equates the energy of about 55,000 Hiroshima bombs per hour, every hour, every day….
    This is the extra energy that remains in our Earth system at the moment and continues to raise our temperature.
    Of worry are the positive further feed-backs such as albedo decline over areas where black ocean now replaces ice, or rock replaces glacier, the release of large amounts of further GG from melting permafrost and others.

    The observed temperature change and ocean heat content change matches well with the observed changes in the Earth’s radiation physics due to the GG increase, mostly CO2, which we are in the process of doubling….

    I recommend http://www.skepticalscience.com/ to anybody who is seriously mystified by Science involved. It contains well developed teaching material.

  243. tallbloke says:

    Samoht says:
    The reduction in outgoing radiation together with its spectral resolution showing which gases are involved is measured, not just calculated from theory. You seem to completely ignore all this.

    Please could you idicate where on this graph of global outgoing radiation where the ‘measured reduction’ showing ‘which gases are involved’ is.

    Thanks

    If you think this graph is wrong, please supply a link to the one you think is correct.

  244. Entropic man says:

    Stephen Wilde

    “Just take the alleged figure for DWIR and show it as a dynamic energy exchange between the surface and the warm air molecules directly above it.

    Exactly the same amount goes from ground to air as goes from air to ground.

    That squares the circle just as well as proposing DWIR.”

    Way too vague. Are we talking about conduction? Convection? Radiation? All three, and if so, in what proportions? With the low windspeeds at the surface, what part does turbulence in the boundary layer play? You can’t avoid the 733C measured surface temperature and the consequent 16,100W/M^2 surface radiation without completely rewriting thermodynamics, so you have to show what mechanisms recycle most of the energy back to the surface.. It’s not science until you can say it with numbers.
    If you want your alternative hypothesis to be taken seriously you’ll have to show that it works at least as well as back radiation.

    [reply] We’re working on it – see Waynes thread. Don’t start wittering about LWIR on it though. 😉

  245. Stephen Wilde says:

    “The net balance is not zero. ”

    It is at the surface / air interface averaged globally.

    Otherwise energy in could not match energy out and we know that it does over time otherwise there could be no gaseous atmosphere.

    Why should the surface / air energy exchange NOT be a net zero over time ?

    Doesn’t warm air warm a cooler surface and a warm surface heat a cooler air primarily by non radiative processes ? Conduction is quite sufficient at the actual interface.

    You say that the surface / air energy exchange is not zero, without any evidence of that proposition, then you have to invent DWIR to equalise the energy flows.

    Not necessary.

    Now it is true that imbalances can occur as a result of system variability in air and oceans but when that happens the air circulation changes so as to eliminate it.

    If a temperature differential opens up between ground surface and the air above it then convection will decline or increase appropriately.

    Likewise over water where the equalisation process is far more rapid than over land due to the power of evaporation.

    Look at tallbloke’s chart of the OLR history.

    OLR decreased from 1975 to 2000 which was a period of high solar activity, poleward climate zone shifting, decreasing cloudiness and increasing ocean heat content.

    OLR has been increasing since 2000 with lower solar activity, equatorward shifting climate zones, increasing global cloudiness and ocean heat content now falling.

    There is no match to the increase in CO2 during the period.

    There is a match to global cloudiness changes which therefore appear to be solar induced.An active sun reduces cloudiness and allows more energy into the oceans. An inactive sun increases cloudiness and allows less energy into the oceans.

    That is consistent with the OLR observations. AGW theory is not.

    Now I do accept that in theory more CO2 might also shift the climate zones but we can see that thei contribution from CO2 counts as nothing compared to the natural shifts which correlate much better with solar activity.

    The dynamic energy exchange between surface and air accounts for the discrepancy from the S-B equation.In the case of Earth about 33C.

    That equation cannot validly be applied to a surface beneath an atmosphere.

    The more massive the atmosphere or the greater the gravitational field the larger will be the discrepancy from S-B expectations at any given level of insolation.

    That discrepancy is always accounted for by the dynamic energy exchange between surface and atmosphere operating primarily via non radiative processes at the molecular level at the interface and not by magical radiative beams of energy from colder air higher up.

  246. Entropic man says:

    tallbloke

    Actually I’m whittering about the energy transfer formulation of the 1st Law of thermodynamics. Radiation is the only one of the three applicable energy transfer methods which can transfer energy against the temperature gradient.

  247. tallbloke says:

    Entropic, you’re not confusing the transfer of energy with heat are you?

    On average the net radiation flux is upwards, away from the surface towards space. Downwelling infrared cannot on average heat the surface of the Earth.

    Agreed?

  248. Stephen Wilde says:

    “Radiation is the only one of the three applicable energy transfer methods which can transfer energy against the temperature gradient”

    Radiation is, however, affected by the intervention of matter as regards net speed of transmission.

    The other energy transfer methods can then freely vary their speeds of energy transfer within an atmosphere.

    That similarity is a critical factor.

    If radiation tries to work against the temperature gradient the other processes speed up to eliminate the effect of any radiation that is going against the gradient.

    The reason being that surface pressure and atmospheric mass put a top limit on the amount of energy that can be retained within the atmosphere at a given level of solar input.

    If the composition of the atmosphere changes so as to alter the rate of radiative transmission then other non radiative processes change their speeds to offset the effect.

    Otherwise planets could not retain atmospheres for billions of years because any change in the speed of radiative transmission resulting from any change in composition would build up a permanent net gain or deficit until the atmosphere were blown off to space or congealed on the surface.

    Atmospheres change their compositions all the time. All such changes have a bearing on the rate of radiative transmission through the atmosphere which is why we see lapse rates from level to level that differ from the ideal lapse rate.

    Something must always offset the effect of compositional changes otherwise atmospheres could not be retained.

    The key is to completely detach the surface / air energy exchange from the rest of the global energy budget.

    If one does that then the solar energy in and longwave energy out budget must always balance as per S-B.

    All the current global energy budget illustrations make the mistake of merging everything together but that should not be done.

    For any planet that retains an atmosphere the top of atmosphere energy exchange will always be in balance.

    However the surface / air energy balance can be pretty much anything that the atmospheric composition and the non radiative processes dictate.

    However, the top of atmosphere radiative balance can never be upset by anything that goes on between the surface and the air above. If it ever were to be upset then the atmosphere would be lost.

    At its simplest it boils down to this:

    i) Solar energy in 255 units

    ii) Longwave energy out 255 units.

    iii) Surface to air 33 units.

    iv) Air to surface 33 units.

    And iv) is greatly assisted by the warming of descending air as it is compressed by gravity.

    No need for DWIR at all.

    If one changes composition but not solar input the surface / air change will still be 33 units because any slowdown in radiative transmission within the atmosphere from the compositional changes is offset by a speeding up of radiative transmission from the top of the atmosphere due to non radiative processes moving energy upward faster.

    The air circulation gets altered but not surface temperature.

  249. Entropic man says:

    tallbloke

    “Entropic, you’re not confusing the transfer of energy with heat are you?

    On average the net radiation flux is upwards, away from the surface towards space. Downwelling infrared cannot on average heat the surface of the Earth.

    Agreed?”

    Not at all. Radiation transfers energy. Heat is the energy stored in a material.

    DWIR cannot produce a direct heating effect. That would create energy from nothing!

    DWIR redirects a proportion of the outgoing energy by re-radiation back towards the surface, effectively slowing the surface cooling rate.

    This increment of extra downward radiation raises the stored heat in the surface, increasing surface temperature. This leads to increased outward radiation. The system reaches equilibrium at the temperature at which the surface pruduces enough extra outward radiation to cancel out the insulating effect of the GHG producing the DWIR. The outgoing radiation at the top of the atmosphere remains the same as the insolation once equilibrium has been reached.

    My point addressed the problem of transferring energy from a cooler atmosphere to a hotter surface, as on Venus. Convection or conduction can only transfer heat from a hotter to a colder medium, from surface to atmosphere.
    You need a way to transfer the radiated energy from the surface back to the surface. Radiation can transfer energy in both directions.

  250. Stephen Wilde says:

    “DWIR redirects a proportion of the outgoing energy by re-radiation back towards the surface, effectively slowing the surface cooling rate”

    In the absence of GHGs ALL energy in the atmosphere must get back to the surface before it can be radiated to space.The reason being that non GHGs cannot radiate ANYTHING (or very little) out to space.

    Therefore the net effect of GHGs must be to send LESS back to the surface given that a portion is radiated out to space directly INSTEAD of having to be routed back to the surface and thence to space.

    “You need a way to transfer the radiated energy from the surface back to the surface”

    No, you need a way to get all the incoming solar energy back out to space.

    GHGs make it easier and faster by adding upward radiation from the atmosphere to the suite of available methods.

    The primary means of returning energy to the surface is via compression of descending air. If GHGs have radiated a slice directly out to space then less energy needs to be transferred back to the surface and the cycle of decompression on the day side and compression on the night side can slow down.

  251. Samoht says:

    Steven: This is the graph form the article I linked:

    Caption: Change in spectrum from 1970 to 1996 due to trace gases. ‘Brightness temperature’ indicates equivalent blackbody temperature (Harries 2001).
    Source: http://www.nature.com/nature/journal/v410/n6826/abs/410355a0.html

    What they found was a drop in outgoing radiation at the wavelength bands that greenhouse gases such as carbon dioxide (CO2) and methane (CH4) absorb energy. The change in outgoing radiation is consistent with theoretical expectations. Thus the paper found “direct experimental evidence for a significant increase in the Earth’s greenhouse effect”.

    And further: http://www.skepticalscience.com/images/Greenhouse_Spectrum.gif
    Source: http://ams.confex.com/ams/Annual2006/techprogram/paper_100737.htm
    This is the spectrum that is measuring the Backradiation arriving at our surface.
    It shows the attribution of the various GG in the backradiation. The H2O backradiation has been subtracted to show the contribution of the GG that remains.

    These two measured spectra show a) that energy is missing in the outgoing radiation and b) that is is consistent with the spectra bands arriving at the surface.

    I can’t comment on your graph above (the PNG file) as you provided no source references for it.

  252. Samoht says:

    Stephen: you said above “In the absence of GHGs ALL energy in the atmosphere must get back to the surface before it can be radiated to space.The reason being that non GHGs cannot radiate ANYTHING (or very little) out to space.”

    [snip]If you had no GHGs in the atmosphere the Earth’s surface would radiate its energy very efficiently to space and the average temperatures on the planet would be too cold to support life as we know it. There would be very little heat energy from the atmosphere to worry about.
    [snip]

    The fact that we have CHGs prevents the Earth surface to radiate efficiently to space as a significant amount of the outgoing radiation is reflected back. The more GHGs you add the stronger the effect becomes.

    [Moderation note] Condescension removed. – TB

    [Reply] The 9% increase in airborne co2 since ~1996 hasn’t had any effect. Why is that?

  253. Samoht says:

    Stephen, 9 out of the 10 hottest years ever recorded where in the last decade. The arctic ice cap is close to vanishing in summer and the ocean heat content is rising relentlessly. So how can you say there was no effect since 1996.
    Again for the benefit of your readers: http://www.skepticalscience.com/

    [Moderation note] The linked site contains cooked up plots. Watch the Y axes labelling.
    No statistically significant warming since 1996 is the observation of the MET office (and Prof Phil Jones of UEA CRU).

  254. Stephen Wilde says:

    “If you had no GHGs in the atmosphere the Earth’s surface would radiate its energy very efficiently to space ”

    What about energy conducted from surface to air and whisked upward by convection and in the process converted to gravitational potential energy ?

    Whilst in the atmosphere it cannot be radiated out to space or to the surface.

    It has to come back to the surface at some later time by some mechanism doesn’t it ?

    So even with a non GHG atmosphere the energy returning to the surface via reconversion of gravitational potential energy to kinetic energy will keep the surface warmer than it would be in the absence of an atmosphere.

    The presence of GHGs makes it easier not harder for energy to radiate out but that doesn’t matter either way.

    Whether GHGs make it easier or harder, there is simply a change in the air circulation and volume of the atmosphere instead of a rise in surface temperature.

    It is the kinetic energy / gravitational potential energy loop that keeps the atmosphere in place and makes the surface warmer than it otherwise would be if there were no atmosphere.

  255. Stephen Wilde says:

    You could say that gravitational potential energy is what supports the atmosphere above the surface.

    Such energy is unavailable to the radiative energy exchange until it is reconverted to kinetic energy.

  256. Samoht says:

    Stephen: There is no net transfer of potential gravitational energy in our atmosphere. For every Kg of air going up, there is another Kg of gas going down somewhere else and no NET energy is moved this way at all. The atmosphere is “supported” by the pressure gradient. Pressure is a function of the Atmospheres temperature.
    Convection currents do move heat upwards and contribute to the heat transfer in the atmosphere but only to about 5% of the energy flow from the Earth’s surface. A large part is transferred as latent heat of evaporation to the cloud layer. The bulk of Earths outgoing IR radiation is generated at around -18C or 255K, which is the black body equivalent average temperature of the Earth. You can see that most radiation to space is therefore generated at temperatures found in regions higher up in the atmosphere. Only a small percentage of our energy flow is directly passing through the atmosphere in form of IR transmission.
    A good read for you and your readers is this: http://www.eoearth.org/article/Energy_balance_of_Earth

    As far as the http://www.skepticalscience.com/ site, it is doing a good job at pointing out the cooking of evidence and data committed by others. You will find all their analysis is backed up with citations of sources of actual scientists work.

    As far as the “Earth has not warmed since 1996” story, have a look at this:
    http://www.skepticalscience.com/print.php?n=895

    [Reply]From the link: “According to the University of East Anglia (UEA) data (HadCRUT), the average surface temperature has warmed 0.12°C since 1996 – it may not be statistically significant, but…”
    But it’s been falling since 2005.

  257. Samoht says:

    Stephen: What people believe or not is their own responsibility. Some choose to actually look at the evidence. Others cherry pick results to support their beliefs.
    [URL snipped]

    And here is a concise taxonomy of peoples beliefs, addressed by science:
    [URL snipped]

    [Reply] This is not armchair psychology corner. I’m sorry, but we’re busy here and don’t have time to deal with dullards. Thanks for playing, now goodbye.

  258. Eco-geek says:

    I cannot see anything in the gas laws or the first law of thermodynamics that prevent greenhouse gasses from absorbing radiation at one wavelength and re-radiating at another typically longer wavelength. As some of this radiation is radiated downwards (i.e. is back radiation) then it is entirely reasonable that any gas might play a role, secondary to the sun, in setting the temperature of the Earth’s surface and atmosphere by physics which lies outside these laws. Here comes the “but”:

    But two things we must take into consideration are:

    1) That GHGs do not just “back radiate” they are also better than non-GHGs and the Earth’s surface at converting thermal energy into radiation.
    2) The gas laws and the first law of thermodynamics do fix the lapse rate.

    Now the energy interactions at the Earth’s surface and lower troposphere due to radiation transfers, conduction, convection and latent heat of evaporation, condensation etc. are very complicated and it would be very easy to get swamped by the complexity of what is happening and end up taking a warmist or skeptical position.

    The way to get a handle on all of this is to look simply at the outgoing radiation budget of the Earth which must on long term average equal the incoming radiation budget from the sun. It is much simper this way as radiation is the only way the planet can lose thermal energy to balance the books and there are no feedbacks from space. Now:

    We know from (1) above that GHGs will add significantly to this outgoing radiation so that the more GHGs we have in the atmosphere the more outgoing radiation there will be for a given temperature. However as the incoming and outgoing radiation budgets must balance the outgoing radiation must be reduced in the event that more GHGs are injected into the atmosphere and this can only be achieved by reducing the temperature of those parts of the atmosphere radiating out into space. In fact outgoing radiation from both GHGs and non-GHGs will decrease as temperatures fall.

    Now because of multiple absorption and emission events lower down in the atmosphere dominate most of the radiation leaving the planet originates from emission events in the upper atmosphere and so GHGs make the upper atmosphere cool – even the IPCC say this. Now by (2) above, as the thermal gradient is constant a cooler upper atmosphere means a cooler lower atmosphere (and because of the interactions with the Earth’s surface a cooler surface to the Earth) – something the IPCC doesn’t say.

    So while I believe in the existence of back radiation and likewise the ability of both GHGs and non-GHGs to radiate into space and thus cool the planet at different rates it does seem to me entirely clear that GHGs produce cooling of the Earth and not warming. Small changes in CO2 concentrations make little difference to global temperatures which is fortunate as we are moving into cooler climes.

    Anyway, here we have someone who not only believes in “back radiation” but also believes that GHGs cool the planet. Imagine that!

  259. Eco-geek says:

    A little addition to my last post:

    Note that while I believe in “back radiation” which is as I understand it is due to the Earth’s surface being heated by shorter wavelength radiation coming in from the sun and then being re-emitted at longer wavelengths which are in turn absorbed and re-emitted by GHGs, some in a downwards direction, it is interesting to consider what would happen if this single back radiation mechanism were absent.

    First of all we mutate our black body Earth so that it absorbs but does not re-emit. In this case the Earth’s surface is cooled by convection and conduction and latent heat of evaporation to the lower atmosphere. In fact this is the predominant method by which cooling of the Earth’s surface happens anyway. I have seen figures of between 80% and 99% in various places.

    But what happens then? Well ignoring the non-radiation thermal equilibrium dynamics between the surface and the atmosphere we know that GHGs cool quickly as they are good at converting thermal energy into radiation so hey presto! What do we get? More “back-radiation”! I do not know which is the most potent form of back-radiation, but from the Lapse Rate Law as discussed above I do not need to. GHGs cool the planet.

    So you see back radiation does exists but in the warmist world only the fully radiation based form. Similarly by denying the existence of back radiation (and therefore “forward radiation”) the skeptics will never be able to falsify the AGW hypothesis.

    Stay cool!

    (Sorry about the ‘d’ word)

  260. Eco-geek says:

    Sometimes I am a complete idiot! This should be my last post..

    In the last addition I said “I do not know which is the most potent form of back-radiation”… but on reflection I think they are both exactly the same. After all we are talking about equal amounts of energy in equal amounts of time (albeit with a temporal phase lag). Both are re-radiated isotropicaly by gasses so both produce exactly the same amount of back radiation.

    What this means is that the back radiation of the Warmists is not an additional thermal input to the Earth’s surface simply an alternate route! For every additional Watt of warmist back radiation the Earth’s surface receives one less Watt of Coolist (lets call it) back radiation! The two cancel exactly!

    What an idiot I am.

    Ye Gods. I have accidentally falsified the AGW hypothesis in the coolist possible way and all before dinner.

    I am going to allow myself an extra beer tonight.

    Cheers everybody and….

    Stay Cool!

    Eco-geek

  261. Stephen Wilde says:

    Samoht said:

    “There is no net transfer of potential gravitational energy in our atmosphere. For every Kg of air going up, there is another Kg of gas going down somewhere else and no NET energy is moved this way at all. The atmosphere is “supported” by the pressure gradient. Pressure is a function of the Atmospheres temperature.”

    Of course there is no net transfer.

    The kinetic energy converted to potential gravitational energy is the same on the way up as it is on the way down.

    However the going up and coming down takes TIME and during that time an amount of kinetic energy is in the form of potential gravitational energy tied up in the atmospheric loop and therefore unavailable for radiative (or any other) transfer.

    Pressure under an open sky is a function only of mass and gravity. For it to be a function of temperature too one needs an enclosed container.

    The atmosphere is supported by solar energy. No solar energy and it falls to the ground. Less solar energy and it reduces in height.

    There is always ever present in the atmosphere a quantity of potential gravitational energy which is invisible to radiative physics.

    It is the failure to properly account for the swapping of energy between kinetic and potential gravitational energy that has caused the problem.

    AGW theory takes the surface temperature BEFORE the kinetic energy at the surface has been converted by rising air to potential gravitational energy but takes the top of atmosphere temperature AFTER kinetic energy at the top of the atmosphere has been converted by rising air to potential gravitational energy.

    That leads to an accounting error.

    If one takes the potential gravitational energy out of the loop on both sides of the equation instead of adding it to one side and deducting it from the other then the numbers balance and S-B is satisfied.

    You cannot mix and match solar energy in and longwave energy out with kinetic to potential gravitational.

    They must be treated as separate energy processing loops.

  262. Eco-geek says:

    Ok, so it wasn’t quite my last post. There is something else I should explain…

    It would be possible to interpret my last post as just saying that GHGs produce additional back radiation by either mechanism which is true . OK so what happens if we remove GHGs from the atmosphere? Well the back radiation rate goes down obviously but then so does the forward radiation rate and the outgoing radiation budget falls short. This of course will mean the Earth’s surface and atmosphere will warm up until equilibrium is restored with more back radiation and more forward radiation.

    You see back radiation doesn’t cause global warming, global warming causes back radiation.

    Stay cool!

  263. Stephen Wilde says:

    Eco.

    It is the conversion of gravitational potential energy to kinetic energy as air descends through the atmospheric column that heats the surface. Not backradiation.

    So called back radiation is simply the temperature of the molecules at the surface.

  264. Eco-geek says:

    Stephen,

    I am not sue whether you are questioning my definition or saying that gasses do not radiate in a downward direction.

    If my definition is wrong then I am lost otherwise I believe gasses do radiate in a downward direction (as well as up) but as discussed in my posts the there is no net warming of the Earth as a result. Indeed my equalising of the two methods of producing back radiation is not quite right as GHGs do not intercept all wavelengths and so what I understand to be “warmist back radiation” is not as potent as “coolist back radiation” which if I drivel on means that GHG fully radiation based feedback is less effective at warming a warmist world than that convectively sourced back radiation. In other words what I take to be the warmist position is self-defeated.

    Anyway I’m going for a beer but would appreciate a better understanding of the definitions.

    Stay cool!

  265. Entropic man says:

    “No statistically significant warming since 1996 is the observation of the MET office (and Prof Phil Jones of UEA CRU).”

    Tallbloke.

    At the warming rate shown between 1960 and 2003, and with the level of short term variability shown by the Hadcrut data, it takes a minimum of 17 years of data to show with 95% confidence that a change has occured.
    The Met Office probably mean that the period since 1996 is too short to see statistically significant change.
    You have chosen to interpret it as saying that no warming has taken place.
    The temperature graph has shown a flatter slope since 2003, but it will take until at least 2020 before it can be demonstrated to be statistically significant, even assuming it continues to stay flat.

    [Reply] Why speculate to obfuscate Entropic? Lucia crunched the numbers to prove the ‘not significant’ finding in 2010, and it has cooled since… Give it up, you are on a loser with this one.
    http://rankexploits.com/musings/2011/statistical-significance-since-1995-not-with-hadcrut

  266. Entropic man says:

    tallbloke

    “Why speculate?”

    Lucia quotes her t calculation for Hadcrut4 1995-2009 as giving a probability of 0.513. That’s a probability of 94.87% . For Hadcrut3, given the same assumptions she gets p<0.05 (exact figure unspecified) and a probability over 95%.

    My basic point stands. At latter 20th century rates of change and variability you need a couple of decades of data before you can reliably say much about the statistical significance of any pattern in the global temperature record. The 15 years used in her calculation are a marginal timespan, with very small changes in the data or the assumptions being enough to push the p value above or below 0.05.

    That, stripped of the subleties, is what Dr. Jones was saying:-

    “The trend over the period 1995-2009 was significant at the 90% level, but wasn’t significant at the standard 95% level that people use,” Professor Jones told BBC News.

    “Basically what’s changed is one more year [of data]. That period 1995-2009 was just 15 years – and because of the uncertainty in estimating trends over short periods, an extra year has made that trend significant at the 95% level which is the traditional threshold that statisticians have used for many years.

    “It just shows the difficulty of achieving significance with a short time series, and that’s why longer series – 20 or 30 years – would be a much better way of estimating trends and getting significance on a consistent basis.”

    [Reply] If you’re right, then we shouldn’t treat the warming from 1980 to 2000 as being particularly significant either. 😉

  267. Entropic man says:

    Take a look at the Hadcrut3 graph here.

    http://www.metoffice.gov.uk/research/monitoring/climate/surface-temperature

    The 5-year mean graph has 95% confidence limits shown. You probably know the inspection method for estimating significance in such data. Pick the two years of interest and compare the 95% confidence limits for the 5-year means. If they do not overlap, the difference between the means ls likely to be significant.

    You would need to do the maths to be sure, but the 1980 and 2000 means look significantly different by this quick and dirty test.